Sei sulla pagina 1di 222

AIIMS Q&A MAY 2015

By Dr Manish B Mandal
Copyright 2015 Dr Manish B Mandal
Smashwords Edition

AIIMS MAY 2015 Q&A

BY Dr Manish B Mandal,MO(BHEL),ExHCMS-2,Ex-Resident AIIMS,SJH.NEWDELHI

1)Muscle not inserted on greater tuberosity


a) Teres minor
b) Supraspinatus
c) Infraspinatus
d) Subscapularis
D
The supraspinatus, infraspinatus, teres minor and subscapularis muscles comprise the rotator
cuff muscle group. The main role of these muscles is stabilization of the humeral head in the
glenoid fossa. Tendons of teres minor, supraspinatus, and infraspinatus insert on the greater
tuberosity of the humerus, and subscapularis tendon inserts on lesser humeral tuberosity.
Actions of these muscles are internal rotation (subscapularis), external rotation (teres minor and
7
infraspinatus) and early abduction from 0 to 30 (supraspinatus) . The subacromial bursa lies
between supraspinatus tendon and the acromion.

http://morphopedics.wikidot.com/supraspinatus-tendinitis
2) Card test done for
a) Lumbricals
b) Palmar interossei
c) Dorsal interossei
d) Adductor pollicis
B

Ulnar Nerve (C7,8,T1) injury :


M.C. site - Medial epicondyle, or a little more
distally where the nerve enters the
forearm b/w the two heads of flexor
carpi ulnaris
Causes: - # of medial condyle
- During anesthesia
S/S:
- Radial deviation of wrist on flexion
- Claw-hand deformity
- Paresthesia and sensory loss on hand
Page 21
21
A- Egawas Test Dorsal Interossei
B- Card Test Palmer interossei
C- Book Test - Adductor pollicis
(Froment,s sign)

Page 22
22
:Median nerve (C5,6,7,8,T1) injury:
CAUSES - Crutch compression
- Sleep paralysis
- Penetrating trauma
- Shoulder dislocation
S/S:
- Atrophy of the thenar
eminence
- Simian or ape hand
(d/t opponens pollicis)
- Benediction hand
Page 23
23
- Paresis of forearm pronation
- Paresis of distal flexion of the thumb
- Paresis of radial wrist flexion
- Impaired opposition of the thumb
- Paresis of flexion of the second and
to a lesser extent, the third fingers
Pen Test For Abductor pollicis brevis
http://www.aiimsnets.org/NeurosurgeryEducation/NeurosurgicalSpecialties/Peripheralnerve/Ner
ve%20Injuries%20Diagnosis,%20Evaluation%20And%20Management_new/Nerve%20Injuries
%20Diagnosis,%20Evaluation%20And%20Management.pdf

3) Which nerve doesnt have a general visceral efferent


a) Olfactory
b) Facial
c) Oculomotor
d) Glossopharyngeal
A
It is possible to describe a cranial nerve in terms of its function and embryological origin, initially cranial
nerves can be subdivided into being either:

Motor (efferent)
Sensory (afferent)

And from there further categorization can occur.


--------------------------------------------------------------------------------------------------Motor (efferent) Cranial nerves
-Somatic motor (general somatic efferent)
(III, IV, VI, XII)
These cranial nerves are so called because they innervate muscles derived from the occipital somites,
such as the extra ocular and extrinsic tongue muscles.
-Branchial motor (special visceral efferent)
(V, VII, IX, X, XI)
These are described as branchial because they specifically innervate muscles which are derived from the
branchial arches during development (muscles of mastication, larynx, facial expression, pharynx and
middle ear)
- Parasympatheic (general visceral efferent)
(III, VII, IX, X)
These nuclei do not innervate striated muscle like the branchial and somatic, they instead provide
preganglionic parasympathetic fibers to innervate glands, smooth muscle and cardiac muscle within the
head, heart, lungs and digestive tract above the splenic flexure.
--------------------------------------------------------------------------------------------------Sensory (afferent) cranial nerves
-Visceral sensory
special visceral afferent- (VII, IX, X)
general visceral afferent- (IX, X)
The name is related to the fact that it detects sensation from visceral organs.
They are divided into special visceral, referring to the rostral portion of the nucleus which contributes to
the special sensation of taste. Whilst the general visceral portion is named as such due to this caudal
portion receiving general sensory impulses such as cardiac, respiratory and GI inputs.
- General somatic sensory (general somatic afferent)
(V, VII, IX, X)
These nuclei detect general sensation, such as touch, pain, vibration from the face, sinuses and
meninges
- Special somatic sensory (special somatic)
(VIII)
This carries information from the special sensation of hearing and balance.

http://www.fastbleep.com/medical-notes/neuro-and-psych/2/95/610
4) Cranial nerve nucleus lying beneath the facial colliculus is
a) Facial
b) Abducent
c) Glossopharyngeal

d) Trigeminal
B
The abducent nucleus: This nucleus lies in the lower part of the pons, deep to the facial
colliculus in the floor of the fourth ventricle. It is situated in the grey matter lining the
floor of the fourth ventricle near the midline. The abducent nucleus is a motor nucleus
and sends its attached nerve to supply the lateral rectus muscle.
https://www.kenhub.com/en/library/anatomy/cranial-nerve-nuclei

Overview of the Brainstem

The brainstem consists (from superior to


inferior) of the midbrain, pons and the medulla oblongata. The midbrain is continuous,
above, with the cerebral hemispheres. The medulla is continuous, below, with the spinal
cord. Posteriorly, the pons and medulla are separated from the cerebellum by the
4th ventricle.
The brainstem has a ventral aspect (anterior surface) and a dorsal or posterior surface.

Posterior surface
The posterior surface of the brainstem is formed by:

the fasciculus gracilis (tracts from the posterior funiculus of the spinal cord) which

ends superiorly as a rounded elevation called gracile tubercle

the fasciculus cuneatus (also tracts from the posterior funiculus of the spinal cord)
which ends superiorly as a rounded elevation called cuneate tubercle
These fasciculi and tubercles are at the lower half of the posterior surface of the medulla
thus the lowest portion of the brainstem (posteriorly).

At the midbrain level, the posterior surface of the brainstem is marked by four rounded
swellings called the superior and inferior colliculi, with one pair (a superior- and an inferior
colliculus) on each side of the midline, as well as a connecting ridge called the brachium.
Most of the remaining part of the posterior surface of the brainstem, which includes the
posterior surfaces of pons and the superior half of the medulla, is formed by the floor of the
4th ventricle.
The 4th ventricle is continuous, below, with the central canal, which traverses the lower part
of the medulla, and becomes continuous with the central canal of the spinal cord. Cranially,
the 4th ventricle is continuous with the cerebral aqueduct (of Sylvius), which passes
through the midbrain to connect the 3rd ventricle [which is a median cavity into which, the
1st and 2nd (lateral) ventricles open, through the interventricular foramen (of Monro).
The fourth ventricle also has a tent-shaped roof, cavity and lateral walls. However, the floor is
the most related part to the cranial nerve nuclei.
The floor of the fourth ventricle, often called the rhomboid fossa because of its shape, is
divisible into an upper triangular part formed by the posterior surface of the pons; a lower
triangular part formed by the upper part of the posterior surface of the medulla; and an
intermediate part at the junction of the pons and medulla. The intermediate part is prolonged
laterally over the inferior cerebellar peduncle as the floor of the lateral recess. Its surface is
marked by the presence of delicate bundles of transeversely running fibres. Those bundles
are the striae medullares.
The entire floor is divided into right and left halves by a median sulcus. Next to the middle
line, there is a longitudinal elevation called the medial eminence. The eminence is bounded
laterally by the sulcus limitans. The region lateral to the sulcus limitans is the vestibular
area which overlies the vestibular nuclei (cranial nerve nuclei responsible for hearing and
balance). The vestibular area lies partly in the pons and partly in the medulla.
The pontine part of the floor shows some features of interest in close relation to the sulcus
limitans and the median eminence. The uppermost part of the sulcus limitans overlies an
area that is bluish in color and this area is called the locus coeruleus. Somewhat lower down,
the sulcus limitans is marked by a depression, the superior fovea. At this level, the medial
eminence shows a swelling, the facial colliculus.
The medullary part of the floor also shows some features of interest in relation to the medial
eminence and the sulcus limitans. The sulcus limitans is marked by a depression, theinferior
fovea. Descending from the fovea, there is a sulcus that runs obliquely towards the middle
line. That sulcus divides the medial eminence into two triangles the hypoglossal
trigone (which houses the cranial nerve nuclei called hypoglossal nuclei), medially; and
thevagal trigone (housing the vagal nuclei), laterally. Between the vagal trigone, superiorly
and the gracile tubercle, inferiorly, there is a small area called the area postrema.
The lowest part of the floor of the fourth ventricle is called the calamus scriptorius, because
of its resemblance to a nib. Each inferolateral margin of the ventricle is marked by a narrow

white ridge or taenia. The right and left taeniae meet at the inferior angle of the floor to form
a small fold called the obex.

Ventral surface
The anterior surface of the brainstem is mainly marked by the left and right crus cerebri at the
midbrain, the middle cerebellar peduncle at the level of the pons, and rounded elevations
called olive and pyramid at the level of the medulla.
Within the brainstem, there is a remarkable number of tracts and grey matter. Along the
entire length of the brainstem, there are areas not occupied by well-defined nuclei or nerve
fibres, but consisted of a network of fibres within which scattered neurons are situated.
Those areas are referred to as the reticular formation of the brainstem. These areas mainly
occupy the dorsal part of the midbrain, pons and medulla. Such areas (reticular formation)
are found at all levels of the central nervous system, and are functionally very important.
At the level of the midbrain and medulla, there is a grey matter region called central grey
matter which surrounds the cerebral aqueduct and gives origin to some cranial nerve nuclei
including the mesencephalic nucleus of the trigeminal nerve. Most of the cranial nerve nuclei
that migrated from their original position (in relation to the floor of fourth ventricle) during
embryonic development are situated in this grey matter region (central grey matter).

Introduction
The cranial nerve nuclei are aggregate of cells (collection of cell bodies). Attached to these
cell bodies are fibres called cranial nerves (bundles of axons). These nuclei are
either sensoryor motor but never both. However, cranial nerves can be sensory, motor or mixed
nerves(when they have both sensory and motor functions).
The cranial nerve motor nuclei are further grouped according to their targets, that is, where
their axons (attached cranial nerves) are sent or structures they innervate. Hence the cranial
nerve nuclei with motor functions can be grouped according to the following functional
components to which their fibres belong:

General Somatic Efferents (GSE)

Special Visceral Efferents (SVE)

General Visceral Efferents (GVE)

Similarly, the cranial nerve sensory nuclei are grouped according to the information they
receive, which constitutes the functional components to which their attached nerves belong.
These functional components are:

General Somatic Afferents (GSA)

Special Somatic Afferents (SSA)

General Visceral Afferents (GVA)

Special Visceral Afferents (SVA)

The first cranial nerve (olfactory nerve CN I) does not originate from a cranial nerve
nucleus. CN 1 originates from the olfactory bulb, which is a structure located in the
forebrain and controls olfaction. The second cranial nerve (optic nerve CN II) originates
from the lateral geniculate nucleus [(lateral geniculate body or lateral geniculate complex)
which is a relay centre in the thalamus for the visual pathway], the pulvinar, and the superior
colliculus [the pulvinar and superior colliculus are also part of the primary visual centres,
and from those three structures, fibres (CN II) connect the ganglionic cells of the retina to
coordinate and interpret visual impulses].
In the brainstem, there are about 18 cranial nerve nuclei comprising of 10 motor cranial
nerve nuclei and 8 sensory cranial nerve nuclei. The functions of those cranial nerves are
suggestive of the functions of the parts of the brainstem they are located. For example, the
midbrain is involved in eye movement control and houses the oculomotor and trochlear
nuclei which also have these functions. The pons control breathing, signal relay, and
contains the trigeminal, abducens and facial nuclei.
In summary therefore, cranial nerve nuclei are either motor (efferent) or sensory (afferent), and
both category can be somatic or visceral.

Development
During embryonic development, the cranial nerve nuclei related to the various components
GSE, SVE, GVE, GSA, SSA, GVA and SVA, are arranged in vertical rows called columns in a
definitive sequence in the grey matter related to the floor of the fourth ventricle. Each half
(along the median sulcus) of the floor of the fourth ventricle is divided into a medial part
called the basal lamina, and a lateral part called the alar lamina by the sulcus limitans.
The Efferent nuclei (motor nuclei) lie in the medial part, while the Afferent nuclei (sensory
nuclei) lie in the lateral part alar lamina. In both the medial and lateral parts, the visceral
nuclei [special visceral (branchial) effernt, general visceral efferent, general visceral afferent,
and special visceral afferent] lie nearer to the sulcus limitans than the somatic nuclei [GSE,
GSA, and SSA]. Therefore, from the midline (median sulcus of the fourth ventricle) to the
lateral aspects, the sequence of nuclear column is:
1. GSE - lying closest to the median sulcus
2. SVE - lying next to the GSE
3. GVE - lying farthest from the median sulcus but next to the sulcus limitans
All of the above listed three nuclear columns are in the medial part (basal lamina) of the floor
of the 4th ventricle.
On the other side of the sulcus limitans that is the lateral part (alar lamina), the sequence
is:

1. GVA - lying next to the sulcus limitans


2. SVA - lying next to GVA, laterally
3. GSA - lying next to the SVA and finally
4. SSA - lying farthest away from the sulcus limitans, laterally
As development continues, parts of those columns disappear from their position in relation
to the floor of the fourth ventricle, so that each of the no longer extends the whole length of
the brainstem, but is represented by one or more discrete nuclei. Some nuclei retain their
original positions in relation to the floor of the fourth ventricle, but some others migrate
deeper into the brainstem.
In the descriptions that follows, the cranial nerve nuclei originating from each of these
columns, as well as their definitive positions in the brainstem are outlined.

Gross Anatomy
General Somatic Efferent (Motor) Nuclei
The general somatic efferent column consists of the following nuclei that supply striated
(skeletal) muscles of somatic origin.

1. The oculomotor nucleus: This nucleus is located in the upper part of the midbrain at the
level of the superior colliculus. The oculomotor nuclei (left and right sides) form a
single complex that lies in the central grey matter, ventral to the cerebral aqueduct (of
Sylvius). The oculomotor nucleus sends fibres (oculomotor nerve) to supply
thesuperior, medial and inferior rectus, inferior oblique and levator palpebrae
superioris muscles.

2. The trochlear nucleus: The trochlear nucleus is located in the lower part of the midbrain
at the level of the inferior colliculus. The nucleus lies anterior to the cerebral aqueduct
in the central grey matter. Since the trochlear nucleus is a motor nucleus, it sends
fibres to innervate the superior oblique muscle.

3. The abducent nucleus: This nucleus lies in the lower part of the pons, deep to the facial
colliculus in the floor of the fourth ventricle. It is situated in the grey matter lining the
floor of the fourth ventricle near the midline. The abducent nucleus is a motor nucleus
and sends its attached nerve to supply the lateral rectus muscle.

4. The hypoglossal nucleus: The hypoglossal nucleus is situated near the midline and
below the hypoglossal trigone (or triangle) in the floor of the fourth ventricle of the
upper medulla. It is an elongated column extending into both the open and closed
parts of the medulla. Fibres from this nucleus are motor, and they innervate
the muscles of the tongue.

Special Visceral Efferent (Motor) Nuclei


These nuclei are also called Branchial Efferent or Branchiomotor nuclei. They supply striated
(skeletal) muscles derived from the branchial arches.

1. The motor nucleus of the trigeminal nerve: This cranial nucleus lies in the upper part of
the pons, in the pons' dorsal part. It is situated in the lateral part of the reticular
formation, medial to the main sensory nucleus of the trigeminal nerve. The motor
nucleus of trigeminal nerve innervates the mastication muscles, mylohyoid
muscle and tensor palati.

2. The nucleus of the facial nerve: This nucleus lies in the lower part of the pons, deep and
lateral to the facial colliculus, and occupies a position similar to that of the motor
nucleus of the trigeminal nerve, because the spinal nucleus and tract of the trigeminal
nerve lie lateral to it, as seen in figure 8 above. The facial nerve nucleus sends its
nerve to the lacrimal gland for lacrimal secretion and salivary secretion.

3. The nucleus ambiguus: The nucleus ambiguus lies in the medulla. It forms an elongated
collumn lying deep in the reticular formation, both in the open and closed parts of the
medulla. Inferiorly, it is continuous with the spinal accessory nucleus. The nucleus
ambiguus is a composite nucleus and contributes fibres to
the glossopharyngeal, vagusand accessory nerves.

General Visceral Efferent (Motor) Nuclei


The nuclei of this column gives origin to preganglionic fibres that contribute to the cranial
parasympathetic outflow. These fibres end in peripheral ganglia. Postganglionic fibres
arising in those ganglia (peripheral ganglia) supply the smooth muscles and glands. The
nuclei from the GVE are as follows:

1. The Edinger-Westphal nucleus (Accessory Oculomotor nucleus): This nucleus lies in the
midbrain. It is closely related to the oculomotor complex. Fibres arising in this
nucleus pass through the oculomotor nerve, and relay in the ciliary ganglion to supply
the sphincter pupillae and the ciliaris muscle (see the Visual Pathway).

2. The Salivary (salivatory) nuclei: The superior and inferior salivary nuclei lie in the dorsal
part of the pons, just above its junction with the medulla. They are located a little
above the upper end of the dorsal nucleus of the vagus nerve. The superior salivatory
nucleus sends fibres into the facial nerve and these fibres relay in the submandibular
ganglion to supply the submandibular and sublingual salivary glands. The inferior
salivary nucleus sends fibres into the glossopharyngeal nerve. Those fibres relay in
the otic ganglion to supply the parotid gland. The parotid gland may also receive
some fibres from the superior salivatory nucleus, through the submandibular
ganglion.

3. The dorsal or motor nucleus of the vagus nerve (dorsal vagal nucleus): This cranial nerve
nucleus is long and it lies vertically in the medulla. Its upper end lies deep to the vagal

trigone in the floor of the fourth ventricle. When traced backwards, it extends into the
closed part of the medulla where it lies in the lateral part of the central grey matter.
Fibres arising fom this nucleus supply
the heart, lungs, bronchi, oesophagus,stomach, small intestine and large
intestine up to the right two-thirds of the transverse colon. Those fibres end in
ganglion (nerve plexuses) closely related to those organs heart, lungs, stomach, etc.

General and Special Visceral Afferent (Sensory) Nuclei


The general and special visceral afferent collumns are represented by:

1. The nucleus of theSOLITARY tract


2. The commissural nucleus of vagus
3. The gustatory nucleus
The nucleus of theSOLITARY tract is present in the medulla. Its cells (nucleus
ofSOLITARY tract) form an elongated column lying deep in the reticular formation. The
upper part of the nucleus of theSOLITARY tract lies ventrolateral to the dorsal nucleus of
the vagus (dorsal vagal nucleus). When traced backwards, it extends into the closed part of
the medulla. There it lies dorsomedial to the vagal nucleus. The lower ends of the nuclei of
the two sides fuse to form the commissural nucleus of the vagus.
The nucleus solitarius receives fibres carrying general visceral sensations through the
vagus and glossopharyngeal nerves. Through those afferents, and through connections
with the reticular formation, the nucleus of solitary tract plays an important role in reflex
control of respiratory and cardiovascular functions.
Fibres of taste special visceral afferent, carried by facial, glossopharyngeal and vagus
nerves end in the upper part of the nucleus of the solitary tract. This upper part of the
nucleus is referred to as the gustatory nucleus.

General Somatic Afferent (Sensory) Nuclei


These nuclei give rise to the following sensory nuclei:

1. The main or superior sensory nucleus: This nucleus lies in the upper part of the pons, in
the lateral part of the reticular formation. It lies lateral to the motor nucleus of the
trigeminal. The superior sensory nucleus is mainly concerned in mediation of
proprioceptive impulses, touch and pressure.

2. The spinal nucleus: The spinal nucleus is another sensory cranial nerve nucleus which
extends from the main nucleus (superior sensory nucleus) in the pons down into the
medulla, and into the upper two segments of the spinal cord. The lower end of the
spinal nucleus is continuous with the substantia gelatinosa of the spinal cord. The
spinal nucleus receives general somatic sensations carried by the facial,
glossopharyngeal and vagus nerves. Functions of the spinal nucleus includes
mediation of pain and thermal sensibility. The spinal nucleus is divisible (craniocaudally) into three sub-nuclei, theoralis, interpolaris, and caudalis.

3. The mesencephalic nucleus: This is also called the mesencephalic nucleus of the
trigeminal nerve. It extends cranially from the upper end of the main sensory nucleus
in the pons into the midbrain. In the midbrain, the mesencephalic nucleus lies in the
central grey matter lateral to the aqueduct. Functionally, this nucleus appears to be
similar to sensory ganglia of the cranial nerves, and to the spinal ganglia, rather than
to afferent nuclei. The processes (dendrites) of the neurons of this nucleus are
believed to carry proprioceptive impulses from muscles of mastication, and possibly
also from muscles of the eyeballs, face, tongue and teeth. The mesencephalic nucleus
is the centre for jaw jerk.

Special Somatic Afferent (Sensory) Nuclei


This column gives rise to:
1. The cochlear nucleus
2. The vestibular nucleus
There are two cochlear nuclei dorsal and ventral cochlear nucleus. They are respectively
positioned dorsal and anterior to the inferior cerebellar peduncle at the level of the junction
between the pons and medulla. The two nuclei are continuous with each other, being
separated only by a layer of nerve fibres.
The vestibular nucleus lies in the grey matter underlying the lateral part of the floor of the
fourth ventricle. The vestibular nucleus is situated partly in the medulla and partly in the
pons. Four distinct nuclei of the vestibular nucleus are recognized. These nuclei are
themedial, lateral, inferior and superior vestibular nuclei. The lateral nucleus is also called
theDeiter's nucleus.

Connections Of Some Cranial Nerve Nuclei


Column of motor nucleus of the trigeminal nerve, nucleus of the facial nerve, nucleus ambiguus,
and oculomotor, trochear, abducent, hypoglossal nuclei
The nuclei of the GSE and SVE connect with the skeletal muscles, to which they send their
nerves. Those nuclei (GSE and SVE) receive sensory fibres through which they also make
connections with the cerebral cortex, the tectum (connections of vision and of hearing),
vestibular nuclei (vestibular impulses), sensory cranial nuclei, the red nucleus and the
reticular formation.
Connections of the cochlear nuclei
The dorsal and ventral cochlear nuclei are major parts of the partway of hearing. They
receive fibres connecting them with the spiral ganglion and sends their attached fibres,
some of which relay in the trapezoid body, before reaching the nucleus of trapezoid body
and the superior olivary nucleus.
Connections of the vestibular nuclei

The vestibular nuclei are connected to the cerebellum to which they send and receive fibres.
These nuclei also have connections with the spinal cord, through fibres, mainly, originating
in the medial vestibular nucleus. These fibres descend in the fasciculus.
Connections of the nucleus of theSOLITARY tract
The nucleus of theSOLITARY tract receives fibres from the sensory ganglia of the
seventh, ninth, and tenth cranial nerves, and mainly send fibres to the hypothalamus
(solitario-hypothalamic tract), the thalamus and cerebral cortex.
Connections of the sensory nuclei of the trigeminal nerve
The main sensory nucleus of the trigeminal nerve and the spinal nucleus of trigeminal nerve,
both receive fibres, through which, they connect with the trigeminal ganglion. These nuclei
(main sensory and spinal nucleus) send fibres to the thalamus and cerebral cortex.

Clinical Aspects
Brain Death
The combined developments of ventilatory machines and transplantation surgery have
underlined the need for defining the criteria for making a correct diagnosis of brain death
the irreversible cessation of brainstem function. In comatose patients with irremediable
structural brain damage due to a disorder that can lead to brain death, brainstem reflexes
must be shown to be absent in order to establish brain death. The following are the criteria
(anatomical basis) normally adopted:

Fixed pupils, not reacting to light testing midbrain function

No corneal reflexes testing pontine connections between trigeminal and facial nerve

nuclei

No vestibulo-ocular reflexes testing the connections between the vestibular nerve

and eye muscle nerves

No gag reflex or response to bronchial stimulation by a catheter passed down the


trachea testing vagal connections in the medulla

No motor responses in any cranial nerves on adequate nociceptive stimulation of any

somatic area testing for facial grimacing from pressure on the supraorbital margins
(connections between trigeminal and facial nerve nuclei) or on the bases of fingernails
(connections between cervical spinal cord and facial nerve nuclei)

5) Mesorectal fascia doesnt contain which of the following

a) Inferior rectal vein


b) Pararectal node
c) Superior rectal vein
A
The mesorectum is an important surgical and radiological structure. It contains the
superior rectal artery and vein, and numerous lymphatics as well as rectal branches of
the inferior hypogastric plexus. It is bounded by the mesorectal fascia, a distinct layer
which separates the rectal fat from other pelvic fat.
http://ozradonc.wikidot.com/anatomy:rectum
6) Which of these is not a support of the uterus
a) Urogenital diaphragm
b) Pelvic diaphragm
c) Perineal body
d) Rectovaginal septum
D
7) Reticular framework is present in all tissues except
a) Thymus
b) Bone marrow
c) Spleen
d) Lymph node
A
Ref= The Anatomical Record >
Vol 190 Issue 3 >
Abstract

Electron microscopy of the normal human thymus demonstrates a supporting framework of epithelial-reticular cells
with long branching cytoplasmic processes joined by desmosomes. In the interstices of the epithelial-reticular cell
processes lie lymphocytes, macrophages, and rare myoid cells. Both small and large lymphocytes are evident. No
desmosomes are observed between the lymphocytes and the epithelial-reticular cells. Macrophages are most
numerous in the cortex where they often contain phagocytized nuclear debris. The possible functional significance of
the above-described fine structural features is discussed.

http://onlinelibrary.wiley.com/doi/10.1002/ar.1091900310/abstract
8) What is the stage present in the oogonia at birth
a) Telophase I
b) Prophase I
B

A wave of proliferation begins that lasts from the 15th week to the 7th

month: primary germ cells arise in the cortical zone via mitosis of oogonia
clones, bound together in cellular bridges, that happens in rapid succession.
The cell bridges are necessary for a synchronous onset of the subsequent
meiosis.

With the onset of the meiosis (earliest onset in the prophase in the 12th week)
the designation of the germ cells changes. They are now called primary
oocytes.
The primary oocytes become arrested in the diplotene stage of prophase I
(the prophase of the first meiotic division). Shortly before birth, all the fetal
oocytes in the female ovary have attained this stage. The meiotic resting
phase that then begins is called the dictyotene and it lasts till puberty, during
which each month (and in each month thereafter until menopause) a pair of
primary oocytes complete the first meiosis. Only a few oocytes (secondary
oocytes plus one polar body), though, reach the second meiosis and the
subsequent ovulation. The remaining oocytes that mature each month
become atretic.
The primary oocytes that remain in the ovaries can stay in the dictyotene
stage up to menopause, in the extreme case, without ever maturing during a
menstrual cycle.

While the oogonia transform into primary oocytes, they become restructured
so that at the end of prophase I (the time of the dictyotene) each one gets
enveloped by a single layer of flat, follicular epithelial cells (descendents of
the coelomic epithelium). (oocyte + follicular epithelium = primordial follicle).

http://www.embryology.ch/anglais/cgametogen/oogenese01.html
9) At which stage of maturation of the spermatocyte the assortment of maternal and
paternal chromosomes takes place
a) Spermatogonia to primary spermatocyte
b) Primary to secondary spermatocyte
c) Secondary spermatocyte to spermatids
d) Spermatids to spermatozoa
B
10) Not a branch of external carotid artery
a) Transverse cervical A.
b) Lingual A.
c) Superior thyroid A.
d) Ascending pharyngeal A.
A
11) All are derived from Mesonephros except
a) Glomerulus
b) Paraoopharan
c) Vas deferans
d) Epididymis

A
The urogenital system arises during the fourth week of development from urogenital ridges in
the intermediate mesoderm on each side of the primitive aorta. The nephrogenic ridgeis the
part of the urogenital ridge that forms the urinary system. Three sets of kidneys develop
sequentially in the embryo: The pronephros is rudimentary and nonfunctional, and regresses
completely. The mesonephros is functional for only a short period of time, and remains as
the mesonephric (Wolffian) duct. The metanephros remains as the permanent adult kidney. It
develops from the uteric bud, an outgrowth of the mesonephric duct, and the metanephric
mesoderm, derived from the caudal part of the nephrogenic ridge.
Urine excreted into the amniotic cavity by the fetus forms a major component of the amniotic
fluid. Urine formation begins towards the end of the first trimester (weeks 11 to 12) and
continues throughout fetal life.
The kidneys develop in the pelvis and ascend during development to their adult anatomical
location at T12-L3. This normally happens by the ninth week.
Table 12 - Adult Derivatives of Embryonic Kidney Structures
Embryonic Structure
Adult Derivative
Ureteric bud (metanephric
Ureter
diverticulum)
Renal pelvis
Major and minor calyces
Collecting tubules
Metanephric mesoderm
Renal glomerulus +
capillaries
Bowmans capsule
Proximal convoluted tubule
Loop of Henle
Distal convoluted tubule
Urinary Bladder

The urinary bladder develops from the upper end of the urogenital sinus, which is
continuous with the allantois. It is lined with endoderm. The lower ends of the
metanephric ducts are incorporated into the wall of the urogenital sinus and form
the trigone of the bladder. The connective tissue and smooth muscle surrounding the
bladder are derived from adjacent splanchnic mesoderm.
The allantois degenerates and remains in the adult as a fibrous cord called the urachus
(median umbilical ligament).
http://www.med.umich.edu/lrc/coursepages/m1/embryology/embryo/11urinarysyste
m.htm

12) Microvilli are not present in


a) Duodenum
b) Gall bladder
c) PCT

d) Collecting duct
D
13) In the following picture, wave B represents [PICTURE] EEG shows high amplitude,
low frequency waves as compared to waves A & C and EOG & EMG are nearly flat
when compared to A. In C also they are flat.
a) REM
b) NREM
c) Quiet wakefulness
d) Awake

B
EEG shows high amplitude, low frequency waves as compared to waves A & C and EOG & EMG
are nearly flat when compared to A. In C also they are flat.

14) In the following diagram left ventricular pressure is nearly equal to diastolic BP at
which number [PICTURE]
a) A
b) B
c) C
d) D
(PIC NOT AVAILABLE)
15) If the interstitial hydrostatic pressure is 2 mm Hg, interstitial oncotic pressure is 7
mmHg and capillary hydrostatic pressure is 25mmHg, what should be the capillary
oncotic pressure to allow a net filtration pressure of 3 mm Hg

a) 21
b) 27
B
GFR
If we disregard any oncotic pressure in the Bowman's capsule, we have in effect, three
pressures to consider: glomerular hydrostatic pressure, glomerular oncotic pressure, and
Bowman's capsule hydrostatic pressure. These can be expressed as a formula that will tell
us the amount of hydrostatic pressure pushing fluid out of the glomerulus:
Net glomerular pressure equals:
Glomerular hydrostatic pressure minus [glomerular concotic pressure + Bowman's capsule
hydrostatic pressure]
In order to measure the glomerular filtration rate, we must add to this measurement an
estimation of glomerular permeability, and surface area that is, how many functioning
nephrons are available in the body for filtration, and how effectively the glomeruli filter fluid. A
normal glomerular filtration rate is around 125mls/min, and this measurement is used to
determine, and to classify, kidney function.

http://www.nottingham.ac.uk/nmp/sonet/rlos/bioproc/gfr/6.html
Net Filtration Pressure (NFP): NFP = HPg - (OPg + HPc)
Glomerular Hydrostatic Pressure: (HPg) is essentially glomerular blood pressure (55
mm Hg)
Filtration Opposing Forces:
Colloid Osmotic Pressure (OPg) of glomerular blood (28 - 30 mm Hg)
Capsular Hydrostatic Pressure (HPc) (15 mm Hg)
NFP = 55 - (30+15)
NFP = 10 mm Hg
Glomerular Filtration Rate - the amount of filtrate formed per minute by the kidneys.
There are generally three factors involved in the GFR:
Page 2
1. total surface area for filtration
2. filtration membrane permeability
3. net filtration pressure
Normal GFR for an adult is 120 - 125 ml/ min. Note a drop in glomerular pressure
of 15% will stop filtration altogether.
Note also that GFR is directly proportional to the net filtration rate.
The GFR must be precisely regulated or many substances normally resorbed will be lost
in the urine, OR wastes that should be expelled may be resorbed.
There are three mechanisms that help to keep the GFR relatively constant:
1. Renal Autoregulation (intrinsic controls)
2. Neural controls
3. Renin-angiotensin system
Renal Autoregulation - the kidney can maintain a relatively constant GFR regardless of
fluctuations in systemic blood pressure. This is done by regulating the diameter of the
afferent and efferent arterioles. This is done in one of two ways:
1. myogenic mechanisms - responds to changes in the pressure in the renal

vessels.
2. tubuloglomerular feedback mechanism - senses changes in the juxtaglomerular
apparatus. Using the macula densa cells: these cells release a vasoconstrictor if
the GFR is too high or permit vasodilation of afferent arterioles if the filtration
rate is too low. This system can work in the range of 80 to 180 mm Hg. It cannot
handle low systemic pressures. Below 45 mm Hg filtration stops.
Neural Control - Sympathetic control. When the sympathetic nervous system is at rest
renal vessels are maximally dilated. Sympathetic stimulation causes constriction of
afferent arterioles thus decreasing filtration rate. This stimulation causes release of
epinephrine from the adrenal medulla. The epinephrine in turn acts on alpha receptors on
vascular smooth muscle. This indirectly starts the renin-angiotensin mechanism by
stimulating macula densa cells.
The sympathetic nervous system also directly stimulates the Juxtaglomerular cells (by
binding NE to beta adrenergic receptors) to release renin, which begins the reninangiotensin mechanism.
Renin-Angiotensin mechanism - begins when juxtaglomerular cells release renin. Renin
acts on angiotensinogen (plasma protein made in the liver) to release angiotensin I which
is then converted to angiotensin II by angiotensin converting enzyme.
Resorption in the kidneys is active (requires ATP) or passive depending on the substance
to be resorbed.
Page 3
Sodium ions are the single most abundant cations in the filtrate. Sodium ion resorption is
always active.
Obligatory water reabsorption: sodium movement establishes a strong osmotic
gradient, and water moves by osmosis into the peritubular capillaries. Here the water is
obliged to follow the salt.
As water leaves the tubules, the relative concentration of the substances still present in
the filtrate increases dramatically and, if able, they will begin to follow their
concentration gradients into the tubule cell. This is called Solvent Drag.
Some substances are not resorbed or are resorbed incompletely. This happens because:
1. they lack carriers
2. they are not lipid soluble
3. they are too large to pass through the plasma membrane pores of the tubule
cells.
The most important of these substances are the nitrogenous end products of
protein and nucleic acid metabolism: urea, creatinine, and uric acid.
Absorptive capabilities of various regions of the renal tubules
Proximal convoluted tubule: is the most active region for reabsorption
All glucose , lactate, and amino acids
65 - 70% of Na+ (linked to cotransport of other solutes or Na+-H+ exchange)
65 - 70% of water
90% of bicarbonate
50% of chloride
>90% of potassium
Almost all uric acid is reabsorbed in the PCT but later is returned to the filtrate.

Of the 125 ml/min of fluid filtered at the glomerulus, only about 40 ml/min
remains after the PCT.
Loop of Henle
Descending limb: Water moves out freely, K+ moves back in.
Ascending limb: Na+, K+, Cl- move out, water cannot leave the ascending limb.
Distal Convoulted Tubule:
Page 4
Na+ and Cl- reabsorbed if needed by the body. This is under hormonal control. If needed
almost ALL of the water and Na+ reaching this area can be reclaimed.
Na+ resorption is under the control of Aldosterone from the adrenal cortex. When
aldosterone is present almost no Na+ leaves the body via the urine. In addition the reninangiotensin mechanism stimulates the release of aldosterone. Aldosterone also promotes
water reabsorption because water follows the Na+.
ADH also plays a role here.
Atrial Natriuretic Peptide: (ANP) is a hormone released from the atrial cardiac cells
when blood volume and/or blood pressure is elevated. ANP inhibits Na+ absorption by
closing Na+ channels. This reduces water reabsorption and therefore blood volume.
Tubular Secretion: is essentially reabsorption in reverse.
We see tubular secretion in the PCT, DCT and cortical collecting ducts, but not in the
loop of Henle.
This is important for:
1. disposing of substances that are not already in the filtrate (drugs)
2. eleminating undesirable substances such as urea and uric acid
3. getting rid of excess K+
4. controlling blood pH
Osmolality: the number of solute particles dissolved in one liter of water. This is
reflected in the solutions ability to cause osmosis.
The kidneys have the important job of keeping the body fluids at a constant 300 mosm
(milliosmoles). This is done by using a countercurrent exchange system.
Some common terms that apply to urinary physiology:
Acidosis: A condition in which the pH of the blood is below 7.35
Alkalosis: A condition in which the pH of the blood is higher than 7.45
Compensation: the physiological response to an acid/base imbalance that acts to
normalize the pH of arterial blood.
Complete compensation: results if the arterial pH is brought to within normal
limits
Partial compensation: is only partially corrected but does not fall within the
normal range
Page 5
If a person has an altered blood pH due to metabolic causes, hyper/hypoventilation may
bring the pH back into the normal range. This would be known as respiratory
compensation.
If a person has an altered blood pH due to respiratory causes then they must use renal

compensation to try to return to normal limits. Renal compensation works by changing


the secretion of H+ and reabsorption of HCO3- by the kidneys.
Metabolic acidosis/alkalosis results from changes in HCO3
- concentrations in the blood.
The normal range for HCO3
- is 22 26 mEq/liter.
Metabolic acidosis is defined as the arterial blood HCO3
- level dropping below 22
mEq/liter. This could result from actual loss of HCO3
- as may be seen with severe
diarrhea or renal disease, or an accumulation of an acid other than HCO3
-, or failure of
the kidneys to excrete H+. If this problem is not too severe we can use respiratory
compensation (through hyperventilation) to bring the blood pH back into the normal
range.
Metabolic alkalosis is defined as an arterial blood HCO3
- level above 26 mEq/liter. A loss
of acid or excessive intake of alkaline drugs can cause the blood pH to rise above 7.45.
The most frequent cause is excessive vomiting which results in a substantial loss of HCl.
Hypoventilation may provide respiratory compensation.
There are basically 4 steps in diagnosing acid/base imbalances:
1. determine whether the pH is high (alkalosis) or low (acidosis).
2. determine which value (PCO2 or HCO3
-) is out of range
3. If the cause is a change in PCO2 the problem is respiratory. If the cause is
HCO3
-, the problem is metabolic.
4. NOW, look at the value that doesnt correspond with the observed pH change.
If it is within its normal range there is no compensation occurring. If it is outside its
normal range compensation is occurring and partially correct the problem.
The partial pressure of carbon dioxide is the single most important indicator of
respiratory function.
When respiratory function is normal PCO2 ranges from 35 - 45 mm Hg.
Respiratory acidosis and alkalosis are both disorders resulting form changes in the partial
pressure of CO2 (PCO2)
Values of PCO2 above 45 mm Hg indicate respiratory acidosis
Values of PCO2 below 35 mm Hg indicate respiratory alkalosis
Page 6
Respiratory acidosis is defined as an abnormally high PCO2 in arterial blood. Inadequate
exhalation of CO2 causes the blood pH to drop. Respiratory acidosis can result from slow
breathing or hampered gas exchange (pneumonia, cystic fibrosis, emphysema). Here CO2
accumulates in the blood. This causes a falling blood pH and rising PCO2. The kidneys
may provide renal compensation by increasing the excretion of H+ and the reabsorption
of HCO3
-.

The goal in treating respiratory acidosis is to increase the blow off (exhalation) of CO2.
Respiratory alkalosis is defined as an abnormally low PCO2 in the arterial blood.. The
cause of this condition is hyperventilation and CO2 is eliminated from the body faster
than it is produced.. Hyperventilation may be caused by several factors such as oxygen
deficiency due to high altitude, stroke, or sever anxiety. Renal compensation may bring
the blood pH into the normal range if the kidneys are able to decrease the excretion of H+
and reabsorption of HCO3
-.
Note that, unlike respiratory acidosis, respiratory alkalosis this is rarely caused by
pathology,.

h
ttp://faculty.etsu.edu/forsman/UrinaryPhysiology.pdf

16) Physiological changes in laparoscopy include all except


a) Increased ?PCWP
b) Increased ICP
c) Decreased FRC
d) Increased pH
D
Summary of Haemodyanamic Changes due to Mechanical pressure of C02 insufation Increased systemic vascular
resistance (SVR) Increased Mean Arterial pressure (MAP) Minimal change in heart rate (HR) Increased cerebral blood
ow (CBF) Increased intracranial pressure (ICP) Decreased renal blood ow (RBF) Decreased portal blood ow
Decreased splanchnic blood flow Decreased pulmonary compliance
C02 absorption Signicant hypercapnia and acidosis may occur duringlaparoscopy due to C02 absorption.
Hypercapnia may cause a decrease in myocardial contractility and lower arrhythmia threshold. The anticipated
direct vascular effect ofhypercapnia, producing arteriolar dilation and decreased SVR,is modulated by mechanical
and neurohumoral responses, including catecholamine release

http://iages.in/pdf/c_67_70.pdf
17) A 50 % increase in radius of vessel will cause rise in blood flow by
a) 10 times
b) 20 times
c) 5 times
d) 100 times
C
There are three primary factors that determine the resistance to blood flow within a single vessel:
vessel diameter (or radius), vessel length, and viscosity of the blood. Of these three factors, the
most important quantitatively and physiologically is vessel diameter. The reason for this is that
vessel diameter changes because of contraction and relaxation of the vascular smooth muscle in the

wall of the blood vessel. Furthermore, as described below, very small changes in vessel diameter
lead to large changes in resistance. Vessel length does not change significantly and blood viscosity
normally stays within a small range (except when hematocrit changes).
Vessel resistance (R) is directly proportional to the length (L) of the vessel and the viscosity () of
the blood, and inversely proportional to the radius to the fourth power (r4). Because changes in
diameter and radius are directly proportional to each other (D = 2r; therefore Dr), diameter can be
substituted for radius in the following expression.

Therefore, a vessel having twice the length of another vessel (and each having the same radius) will
have twice the resistance to flow. Similarly, if the viscosity of the blood increases 2-fold, the
resistance to flow will increase 2-fold. In contrast, an increase in radius will reduce resistance.
Furthermore, the change in radius alters resistance to the fourth power of the change in radius. For
example, a 2-fold increase in radius decreases resistance by 16-fold! Therefore, vessel resistance is
exquisitely sensitive to changes in radius.

The relationship between flow and vessel radius to the fourth power (assuming constant P, L, and
laminar flow conditions) is illustrated in the figure to the right. This figure shows how very small
decreases in radius dramatically reduces flow.
Vessel length does not change appreciably in vivo and, therefore, can generally be considered as a
constant. Blood viscosity normally does not change very much; however, it can be significantly
altered by changes in hematocrit, temperature, and by low flow states.
If the above expression for resistance is combined with the equation describing the relationship
between flow, pressure and resistance (F=P/R), then

This relationship (Poiseuille's equation) was first described by the 19th century French physician
Poiseuille. It is a description of how flow is related to perfusion pressure, radius, length, and
viscosity. The full equation contains a constant of integration and pi, which are not included in the
above proportionality.
In the body, however, flow does not conform exactly to this relationship because this relationship
assumes long, straight tubes (blood vessels), a Newtonian fluid (e.g., water, not blood which is non-

Newtonian), and steady, laminar flowconditions. Nevertheless, the relationship clearly shows the
dominant influence of vessel radius on resistance and flow and therefore serves as an important
concept to understand how physiological (e.g., vascular tone) and pathological (e.g., vascular
stenosis) changes in vessel radius affect pressure and flow, and how changes in heart valve orifice
size (e.g., in valvular stenosis) affect flow and pressure gradients across heart valves.
Although the above discussion is directed toward blood vessels, the factors that determine
resistance across a heart valve are the same as described above except that length becomes
insignificant because path of blood flow across a valve is extremely short compared to a blood
vessel. Therefore, when resistance to flow is described for heart valves, the primary factors
considered are radius and blood viscosity.

http://www.cvphysiology.com/Hemodynamics/H003.htm

18) The clot formed is not stable unless extensive cross linking occurs. This is done by
a) Plasmin
b) Thrombin
c) Factor XIII
d) HMWK
C
19) In a study mannitol 10g is injected iv and the concentration measured after some
time is 50 mg/100ml. In this time 10% of mannitol is excreted. What is the volume of
the ECF
a) 18 L
b) 42 L
A
Volume of ECF=amount/concentration

20) ABO blood group is an example of


a) Co dominance
A
21) What is true regarding alpha and gamma neurons during voluntary movements
a) Alpha acts first
b) Gamma acts first
c) Both act together
d) gamma 1st then aphla
ANS=C

Motor neurons are divided into two groups. Alpha motor neurons innervate extrafusal
fibers, the highly contracting fibers that supply the muscle with its power. Gamma
motor neurons innervate intrafusal fibers, which contract only slightly. The function
of intrafusal fiber contraction is not to provide force to the muscle; rather, gamma
activation of the intrafusal fiber is necessary to keep the muscle spindle taut, and
therefore sensitive to stretch, over a wide range of muscle lengths. This concept is
illustrated in Figure 1.10. If a resting muscle is stretched, the muscle spindle becomes
stretched in parallel, sending signals through the primary and secondary afferents. A
subsequent contraction of the muscle, however, removes the pull on the spindle, and
it becomes slack, causing the spindle afferents to cease firing. If the muscle were to
be stretched again, the muscle spindle would not be able to signal this stretch. Thus,
the spindle is rendered temporarily insensitive to stretch after the muscle has
contracted. Activation of gamma motor neurons prevents this temporary insensitivity
by causing a weak contraction of the intrafusal fibers, in parallel with the contraction
of the muscle. This contraction keeps the spindle taut at all times and maintains its
sensitivity to changes in the length of the muscle. Thus, when the CNS instructs a
muscle to contract, it not only sends the appropriate signals to the alpha motor
neurons, it also instructs gamma motor neurons to contract the intrafusal fibers
appropriately; this coordinated process is referred to as alpha-gamma coactivation.
http://neuroscience.uth.tmc.edu/s3/chapter01.html

22) Which of these is secreted by beta cells of pancreas along with insulin
a) Amylin
b) Glucagon like peptide
A
23) Brown adipose tissue is present in all except
a) Around blood vessels
b) Subcutaneous tissue
c) Adrenal cortex
d) Scapula
ANS=D

Visceral BAT includes the following: 1) Perivascular BAT around the aorta, common
carotid artery, and brachiocephalic artery; in anterior mediastinum (paracardial) fat
(Fig. 1); and around epicardial coronary artery and cardiac veins as well as mediumsized muscular arteries and veins including the internal mammary and the intercostal
artery branches from the subclavian and aorta. The intercostal veins drain blood from
the chest and abdominal walls into the azygous veins, the left joining the main right
azygous vein in the latters thoracic cephalad course closely adjacent to the inferior
vena cava before emptying into the superior vena cava (14). 2) Viscus BAT, defined as
BAT surrounding a hollow muscular organ other than blood vessels, situated in variable
amounts in the epicardium around the heart (Fig. 1) and in the esophago-tracheal
groove, as well as greater omentum and transverse mesocolon in the peritoneal
cavity. 3) BAT around solid organs, namely, kidney, adrenal, pancreas, liver, and
splenic hilum including paravertebral fat, which was not examined in Heatons series
(2) but can be seen on CT scans of the thorax adjacent to periaortic fat (Fig. 1). It lies
next to the intercostal artery from which a spinal branch supplies the spinal cord (14).
To our knowledge, BAT has not been described in the meninges covering the brain and
spinal cord or in the subcutaneous tissue of the scalp.
http://diabetes.diabetesjournals.org/content/62/6/1783.full
--------------------------------------------------------------------------------Brown adipose tissue is sometimes mistaken for a type of gland, which it resembles
more than white adipose tissue. It varies in color from dark red to tan, reflecting lipid
content. Its lipid reserves are depleted when the animal is exposed to a cold
environment, and the color darkens. In contrast to white fat, brown fat is richly
vascularized and has numerous unmyelinated nerves which provide sympathetic

stimulation to the adipocytes.


Brown fat is most prominent in newborn animals. In human infants it comprises up to
5% of body weight, then diminishes with age. Substantial quantities of brown adipose
tissue can be detected in adult humans using positron-emission tomography,
especially when the individuals are exposed to cold temperatures.* Most of this tissue
in adults is located in the lower neck and supraclavicular region. Intriguingly, there is
an inverse correlation between the amount of brown adipose tissue and body mass
index, with obese individuals having significantly less of the tissue than lean
individuals; this suggests that brown fat may be an important factor in maintaining a
lean phenotype.
A good place to observe brown fat is in mice, where it persists into adulthood.*******
Dissection of a mouse will reveal two large, lobulated masses of brown fat on the
dorsal aspect of the thorax, between the scapulae.********* Masses of brown fat are
also to be found around the aorta and in the hilus of the kidney.
Examination of sections of white and brown fat at low magnification reveal dramatic
differences in structure, as seen below in images of mouse tissues.
http://www.vivo.colostate.edu//p/misc_topics/brownfat.html

24) 7 yr old boy with presents with severe abdominal pain. On examination he has
xanthomas. Blood drawn for work up has milky appearance of plasma. Which
lipoprotein is increased
a) Chylomicron
b) Chylomicron remnants
c) LDL
d) HDL
ANS=A
http://www.rxpgonline.com/modules.php?name=usertools&file=redirect&url=http://
usmle.biochemistryformedics.com/milky-plasma-your-diagnosis/

25) If in a person, Total Cholesterol= 300mg/dL, HDL= 25mg/dL and TAG=150mg/dL,


find out the value of LDL.
a) 245
b) 125
c) 95
d) 55
A
Friedewald (1972) Formula: LDL = TC - HDL - TG/5.0 (mg/dL)

http://homepages.slingshot.co.nz/~geoff36/LDL_mg.htm

26) In a person, LDL is highly elevated but the level of LDL receptors is normal. What
could be the cause
a) Phosphorylation of LDL receptors
b) ApoB100 mutation
c) Lipoprotein lipase deficiency
d) Cholesterol Acyl CoA transferased deficiency
B
27) An infant at ?7months with history of vomiting and failure to thrive. Patient
improved with iv glucose. After one month returns with same complaints. On
evaluation found to have high glutamine and uracil. Which is the likely enzyme
defect
a) CPS1
b) Ornithine transcarbamoylase
c) Arginase
d) Arginosuccinase lyase
A
The liver is the only site of the complete urea cycle. Among the six enzymes in the cycle, N-acetylglutamate
synthase (NAGS), carbamyl phosphate synthetase 1 (CPS1) and ornithine transcarbamylase (OTC) are
intramitochondrial whereas arginase, argininosuccinate synthetase (ASS) and argininosuccinate lyase (ASL) are
cytosolic.
Unlike fats and carbohydrates, protein is not stored in the body but rather exists in a balanced state of anabolism
(formation) and catabolism (breakdown). Protein excess beyond normal bodily requirements comes from either
excess dietary protein intake or from protein breakdown through various catabolic processes (stress of the
newborn period, infection, dehydration, injury, or surgery). Amino acids liberated from excess protein are broken
down, releasing nitrogen which circulates in the body as ammonia (NH3). Ammonia is then converted into urea via
the urea cycle and disposed of in the urine. An enzymatic block in the urea cycle defect results in the
accumulation of excess ammonia which has toxic effects, most severe in the central nervous system causing
cerebral edema.
Ammonia also circulates in the body as free ammonia or within glutamine which functions as a temporary
repository for ammonia. Consequently, in a urea cycle defect not only does free ammonia rise
(hyperammonemia) but glutamine is also elevated. Alanine (Ala) is another amino acid that accumulates as a result
of hyperammonemia due to a urea cycle defect. These two amino acid elevations (glutamine, alanine) may
precede hyperammonemia and the onset of clinical symptoms and can serve as useful biochemical markers of
decompensation in a patient with a urea cycle defect.
Clinical Presentation of an Acute Hyperammonemic Episode
Poor feeding
Lethargy

Tachypnea
Hypothermia
Irritability
Vomiting
Ataxia
Seizures
Hepatomegaly
Coma
Hyperammonemic crises in neonates or infants with OTC deficiency are frequently precipitated by sepsis.
Consequently, any neonate or infant with OTC deficiency who has clinical signs of a severe illness should be
evaluated for a hyperammonemic crisis precipitated by infection or any other stressor.
Clinical Assessment
Assess cardiac, respiratory, neurologic, and hydration status.
Identify potential precipitant(s) of metabolic decompensation such as infection (presence of fever) or any other
physical stressor (e.g. injury, surgery).
Initial laboratory tests to order:
Plasma ammonia (1.5 ml blood in sodium-heparin tube sent STAT to lab on ice, run immediately)
Plasma amino acid profile
Urine orotic acid
Liver function tests (AST,ALT, alakaline phospatase, bilirubin)
Arterial or venous blood gas
Serum electrolytes, bicarbonate, BUN, creatinine
Blood glucose
Blood, urine, and/or CSF cultures (as clinically indicated)
Plasma ammonia level is a direct index of CNS toxicity and is important to follow in acute management.
Plasma amino acids should be drawn at presentation and should be monitored frequently thereafter. Glutamine, as
an ammonia buffer, reflects the direction of control of the hyperammonemia and, therefore, it is a useful marker
for monitoring of ammonia status.
Other amino acids, including glutamate, glycine, asparagine, aspartate and lysine, may be elevated when there is
an excess in waste nitrogen burden.
Please note that prolonged therapy with phenylacetate (phenylbutyrate) and/or benzoate may lead to a
disproportionate decrease or only a modest elevation of glutamine and/or glycine.
Laboratory Findings in OTC Deficiency
High plasma ammonia level
High plasma glutamine and alanine levels, low plasma citrulline level; low plasma arginine level
Very high urine orotic acid level

Respiratory alkalosis (initially)


Treatment
Immediate treatment of hyperammonemia is crucial to prevent neurologic damage and avoid associated morbidity
and mortality. Cognitive outcome is inversely related to the number of days of neonatal coma in the urea cycle
disorders. Rapid control of the hyperammonemia is crucial in preventing or lessening the degree of mental
retardation.
Key steps of immediate treatment:
Stop all protein intake (but do not withhold protein for longer than 36-48 hours as that can promote breakdown of
endogenous proteins and hamper metabolic control).
Provide intravenous fluids with dextrose and intralipids: 10%-25% IV dextrose at 1.5 times maintenance rate and
intralipid at 1-3 g/kg/day to provide 120-130 kcal/kg/day. This will require a central line.
Provide ammonia scavenger medications as detailed below.
Prepare for probable hemodialysis by contacting the relevant renal and surgical specialists in anticipation of
imminent need.
The treatment of a hyperammonemic crisis in a patient with a urea cycle disorder rests on the principles which are
detailed in the following sections:
A. Promote waste nitrogen excretion
B. Reverse catabolism/optimize caloric intake
C. Treat the underlying precipitant
A. Promote Waste Nitrogen Excretion
There are two main ways to promote ammonia detoxification: hemodialysis and medications that facilitate
ammonia excretion.
Hemodialysis is the most effective way of rapidly disposing of excess ammonia and is far superior to other methods
of dialysis (hemofiltration, peritoneal dialysis). Hemodialysis has the added benefit of removing amino acids such
as glutamine and, in that way, disposing of additional waste nitrogen from the body. A newborn or young infant
with a plasma ammonia greater than 300 mol/L, should have hemodialysis ASAP, and administer IV Ammonul until
hemodialysis is instituted. Central venous catheters should be placed in a critically ill patient in hyperammonemic
crisis in anticipation of the potential for hemodialysis and the appropriate nephrology and surgical specialists
should be alerted in advance for this potential need. The decision to hemodialyze is critical in preventing or
minimizing irreversible CNS damage; when in doubt in the face of a markedly elevated ammonia level, the decision
should be to hemodialyze as soon as possible.
Ammonia scavenger medications include IV Ammonul (sodium benzoate and sodium phenylacetate). Sodium
benzoate conjugates with glycine to form hippuric acid and sodium phenylacetate conjugates with glutamine to
form phenylacetylglutamine; both compounds are excreted in the urine, thereby removing the nitrogen (N) in
glycine and glutamine which contribute to the hyperammonemia.

Sodium should not be provided in supplemental IV fluids when IV Ammonul is given since this solution contains
sufficient amounts of sodium. Otherwise, hypernatremia may result.
Side effects of IV Ammonul may occur in children, including nausea and vomiting. This may be controlled with
antiemetic medications such as ondansetron, either prior to or during the infusion. Overdoses (3-5x the
recommended dose) of IV Ammonul can lead to agitation, confusion and hyperventilation. Caution should be
exercised to avoid overdosing.
IV Ammonul dosing:
0-20 kg: 2.5 mL/kg (prior to mixture with dextrose solution) IV bolus over 90 min followed by the same dose as a
24-hour infusion.
>20 kg: 55 mL/m2 (prior to mixture with dextrose solution) IV bolus over 90 min followed by the same dose as a
24-hour infusion.
IV Arginine (600 mg/kg/day) used to provide supplemental arginine which may be deficient due to early
enzymatic blocks and to stimulate the urea cycle. After the diagnoses of citrullinemia and argininosuccinic aciduria
have been ruled out, the dose may be reduced to 250 mg/kg/day.
Citrulline used in OTC deficiency when the child is able to have enteral feeding. Citrulline (by mouth, NG or gtube) may help pull aspartate into the urea cycle and, thus, increase nitrogen clearance.
B. Reverse Catabolism/Optimize Caloric Intake
*Diet should be planned in conjunction with a metabolic dietitian*
The caloric intake for neonates and infants with OTC deficiency in hyperammonemic crisis should be kept at least
at 120-130 kcal/kg/day to reverse catabolism. Strict intake and output records should be monitored.
The caloric intake on day 1 is provided by intravenous dextrose and supplemented with intralipid to provide 120130 kcal/kg/day. Protein intake should ideally commence within 36-48 hours to avoid breakdown of endogenous
proteins but may be delayed depending on clinical status. Start protein at 0.6 grams/kg/day, administered as
essential amino acids. After 48 hours of this regimen and if patient tolerates this well, the protein concentration
can be increased to 1.2 grams/kg/day, half in the form of essential amino acids and the other half in the form of a
natural protein source (regular infant formula or breast milk) but, avoid elemental formulas as they are high in
nitrogen content. Supplemental calories are added from a non-nitrogenous formula with vitamins and minerals
(Ross formula Pro-Phree or equivalent). Thereafter, the protein intake can be gradually increased by 0.25 0.5
gram/kg/day increments to a maximum of 2 grams /kg/day.
Enteral feeds (oral or NG/NJ) should be started as soon as practical and that can even occur concomitant with IV
nutrition and fluids if necessary.
C. Treat the Underlying Precipitant
A metabolic decompensation in a patient with a UCD is often precipitated by an underlying illness such as an

infection or dehydration which results in a state of catabolism. Diagnostic investigation and treatment aimed at
this underlying precipitant is extremely important to optimize metabolic control of the decompensation and should
be undertaken at the time of initial presentation and continued throughout the management phase of
hyperammonemia.
*Caution: Do not perform a lumbar puncture (LP) before evaluating for the presence of cerebral edema, which may
contraindicate an LP.
Monitoring
Plasma ammonia levels do not always directly correlate with the presence or severity of clinical signs and
symptoms and, thus, monitoring of clinical status and changes in that is crucial. Clinical decisions on appropriate
treatments should be based on the combination of clinical assessment and plasma ammonia levels.
Monitor ammonia levels every 4 hours, and electrolytes and arterial/venous blood gas as clinically indicated.
Plasma amino acids should be monitored frequently depending on levels. If another IV is required, that solution
should not contain sodium if given simultaneously with IV Ammonul
There may be a rebound hyperammonemia initially as stored glutamine is metabolized to glutamate and
ammonia, and with the efflux of intracellular ammonia into the relatively ammonia-depleted blood while on
treatment and, thus, it is important to continue closely monitoring plasma ammonia levels until they remain stable
in the normal range. On rare occasions it may be necessary to assess the magnitude of glutamine excess in brain
tissue by performing brain magnetic resonance spectroscopy (MRS).
Plasma glucose levels should be kept below 150 mg%. If hyperglycemia occurs while IV 10%-20% dextrose is supplied
for added calories, an IV insulin drip at 0.01 units/kg/hour should be started to maintain plasma glucose between
100 and 150 mg%. Insulin may be increased by 0.01-0.03 units/kg/hour until desired effect is obtained. Please note
that as the patients condition improves and anabolic homeostasis is restored, it may be necessary to rapidly
eliminate or reduce the rate of the insulin infusion as hypoglycemia may develop. High IV dextrose solutions should
not be decreased or stopped in the face of hyperglycemia. The goal is to keep the level from rising above 150 mg/
dl. Wide swings in glucose levels are not ideal and so plasma glucose should be kept within the above as best as
possible.
Cerebral edema: Oncotic agents such as albumin will increase the overall nitrogen load but may in selected cases
be considered. Mannitol has been used but may not be as effective as hypertonic saline in alleviating cerebral
edema due to hyperammonemia. Steroids should never be used in a patient with hyperammonemia.
Hyperventilation is recommended, but only under close supervision.
Neurologic status should be closely monitored for signs of CNS toxicity and cerebral edema while the patient is
under treatment and in the recovery period.
Caloric intake should be kept at a high level at all times to prevent catabolism. Close monitoring of the intake of
calories is an essential part of treatment and monitoring of a patient with a urea cycle defect in crisis.
Avoid certain medications, such as valproic acid, as it interferes with urea cycle function and accentuates
hyperammonemia.
Recovery
Once the patient is stabilized and improving, oral diet has been established, and the plasma ammonia level is
stable in an acceptable range, oral scavenger medications (sodium benzoate, sodium phenylbutyrate) and oral
arginine can be provided in place of their IV forms.

The dose of sodium benzoate and sodium phenylbutyrate is determined based on either body weight or body
surface area. The dose should be decided in conjunction with a metabolic physician if the patient does not have an
up to date regimen.

http://newenglandconsortium.org/for-professionals/acute-illnessprotocols/urea-cycle-disorders/ornithine-transcarbamylase-deficiency-otc/
http://www.namrata.co/case-study-urea-cycle-disorders/

28) A child was found to have hypoglycemia, hepatomegaly and accumulation of


highly branched glycogen called limit dextrins. He is likely to be suffering from
a) Cori's Disease
b) Von Gierke's D.
c) Andersons D.
d) McArdles D.
A
Glycogen storage disease type III
GSD type III is also known as Forbes-Cori disease or limit dextrinosis. In contrast to
GSD type I, liver and skeletal muscles are involved in GSD type III. Glycogen deposited
in these organs has an abnormal structure. Differentiating patients with GSD type III
from those with GSD type I solely on the basis of physical findings is not easy.
http://emedicine.medscape.com/article/1116574-overview

29) A 48 yr old lady presented with On examination ofcrumpled tissue paper


appearance is seen. Which is the product likely to have accumulated
a) Glucocerebroside
b) Sphingomylin
c) Sulfatide
d) Ganglioside
A
Clinical Features of Gaucher Disease
The hallmark of Gaucher disease is the presence of lipid-engorged cells of the
monocyte/macrophage lineage with a characteristic appearance in a variety of tissues. These
distinctive cells contain one or more nuclei and their cytoplasm contains a striated tubular
pattern described as "wrinkled tissue paper". These cells are called Gaucher cells.
Clinically, Gaucher disease is classified into three major types. These types are
determined by the absence or presence and severity of neurological involvement. Type 1 is
the most commonly occurring form of Gaucher disease and is called the nonneuronopathic
type (historically called the adult form). Both type 2 and type 3 Gaucher disease have

neuronopathic involvement. Type 2 disease is the acute neuronopathic form, exhibits early
onset of severe central nervous system dysfunction and is usually fatal within the first 2 years
of life. Type 3 Gaucher disease (subacute neuronopathic) patients have later onset
neurological symptoms with a more chronic course than type 2 patients.
Enlargement of the liver is characteristic in all Gaucher disease patients. In severe cases
the liver can fill the entire abdomen. Splenomegaly is present in all but the most mildly
affected individuals and even in asymptomatic individuals spleen enlargement can be found.
In addition to hepatosplenomegaly, bleeding is a common presenting symptom in Gaucher
disease. The most common cause of the bleeding is thrombocytopenia (deficient production
of platelets).
Type 1 (adult, nonneuronopathic): Type 1 Gaucher disease has a broad spectrum of
severity from early onset of massive hepatosplenomegaly and extensive skeletal
abnormalities to patients lacking symptoms until the eighth or ninth decade of life. The
median age of appearance of symptoms in type 1 Gaucher disease is 30 years. Almost all
mildly affected type 1 patients harbor the N370S mutation. The symptoms of type 1 Gaucher
disease result from engorged macrophages that result in hepatosplenomegaly with resultant
dysfunction of the liver and spleen.
Type 2 (infantile, neuronopathic): Type 2 Gaucher disease is characterized by onset at
an early age (hence being called the infantile form) and severe neurologic involvement.
Extensive visceral involvement with heptosplenomegaly is characteristic of type 2 Gacuher
disease. Abnormalities in oculomotor function is often the first manifesting symptom in type 2
disease. Patients often thrust their heads in an attempt to compensate when following a
moving object. An ichthyosis-like (dry scaly skin) disorder has been suggested as a symptom
to allow for the differentiation of type 2 Gaucher disease from types 1 and 3.
Type 3 (juvenile, neuronopathic): Type 3 Gaucher disease is divided into three subclasses. Type 3a presents with progressive neurologic involvement dominated by dementia
and myoclonus (involuntary muscleTWITCHING ). Type 3b presents with aggressive
skeletal and visceral symptoms. The neurological symptoms are limited to horizontal
supranuclear gaze palsy. Type 3c presents with neurological involvement limited to horizontal
supranuclear gaze palsy, cardiac valve calcification and corneal opacities but with little
visceral involvement.

http://themedicalbiochemistrypage.org/gaucherdisease.php

30) A child comes with coarse facial features, big tongue, hepatosplenomegaly,
mucous discharge from nose. Diagnosis is
a) Hurlers syndrome
b) Beckwith Weidemann S.
c) Hypothyroidism
d) Proteus syndrome
ANS=A

Mucopolysaccharidoses
The National Organization for Rare Disorders (NORD) web site, its databases, and the contents
thereof are copyrighted by NORD. No part of the NORD web site, databases, or the contents
may be copied in any way, including but not limited to the following: electronically
downloading, storing in a retrieval system, or redistributing for any commercial purposes without

the express written permission of NORD. Permission is hereby granted to print one hard copy of
the information on an individual disease for your personal use, provided that such content is in
no way modified, and the credit for the source (NORD) and NORDs copyright notice are
included on the printed copy. Any other electronic reproduction or other printed versions is
strictly prohibited.
The information in NORDs Rare Disease Database is for educational purposes only. It should
never be used for diagnostic or treatment purposes. If you have questions regarding a medical
condition, always seek the advice of your physician or other qualified health professional.
NORDs reports provide a brief overview of rare diseases. For more specific information, we
encourage you to contact your personal physician or the agencies listed as Resources on this
report.
Copyright 1989, 1997, 2003, 2004, 2011
NORD is very grateful to Deborah Marsden, MD, Metabolism Program, Children's Hospital
Boston, for assistance in the preparation of this report.

Synonyms of Mucopolysaccharidoses
MPS
MPS Disorder

Disorder Subdivisions
MPS 1 H/S (Hurler/Scheie Syndrome)
MPS I H (Hurler Disease)
MPS II-(Hunter Syndrome)
MPS III A, B, C, and D (Sanfillipo Syndrome)
MPS I S (Scheie Syndrome)
MPS IV A and B (Morquio Syndrome)
MPS IX (Hyaluronidase Deficiency)
MPS VII (Sly Syndrome)
MPS VI (Maroteaux-Lamy Syndrome)

General Discussion
The mucopolysaccharidoses (MPS) are a group of inherited lysosomal storage disorders.
Lysosomes function as the primary digestive units within cells. Enzymes within lysosomes break
Page 2
down or digest particular nutrients, such as certain carbohydrates and fats. In individuals with
MPS disorders, deficiency or malfunction of specific lysosomal enzymes leads to an abnormal
accumulation of certain complex carbohydrates (mucopolysaccharides or glycosaminoglycans)
in the arteries, skeleton, eyes, joints, ears, skin, and/or teeth. These accumulations may also be
found in the respiratory system, liver, spleen, central nervous system, blood, and bone marrow.

This accumulation eventually causes progressive damage to cells, tissues, and various organ
systems of the body. There are several different types and subtypes of mucopolysaccharidosis.
These disorders, with one exception, are inherited as autosomal recessive traits.

Symptoms
Individuals with MPS disorders share many similar symptoms such as multiple organ
involvement, distinctive "coarse" facial features, and abnormalities of the skeleton especially
joint problems. Additional findings include short stature, heart abnormalities, breathing
irregularities, liver and spleen enlargement (hepatosplenomegaly), and/or neurological
abnormalities. The severity of the different MPS disorders varies greatly among affected
individuals, even among those with the same type of MPS and even among individuals of the
same family.
In most cases of MPS, affected infants appear normal at birth and symptoms become apparent
around the age of one or two. Initial symptoms may include frequent colds, runny nose,
infections, growth delays, or mild developmental delays. Mild forms of these disorders may not
become apparent until childhood or adolescence. In most cases, the mucopolysaccharidoses are
chronic, progressive disorders and, depending upon the type of MPS and severity, affected
individuals may experience a decline in physical and mental function, sometimes resulting in
life-threatening complications.
There are different types of mucopolysaccharides that are not broken down due to enzyme
malfunction or deficiency. Specifically, the mucopolysaccharides known as dermatan sulfate,
heparan sulfate, or keratan sulfate may be involved alone or in some combination.
Disorder Subdivisions:
Hurler syndrome (mucopolysaccharidosis type 1-H; MPS 1-H) is the most severe form of
mucopolysaccharidosis. It is characterized by a deficiency of the enzyme alpha-L-iduronidase,
which results in an accumulation of dermatan and heparan sulfates. Symptoms of the disorder
first become evident at six months to two years of age. Affected infants may experience
developmental delays, recurrent urinary and upper respiratory tract infections, noisy breathing
and persistent nasal discharge. Additional physical problems may include clouding of the cornea
of the eye, an unusually large tongue, severe deformity of the spine, and joint stiffness. Mental
development begins to regress at about the age of two.
Scheie syndrome (mucopolysaccharidosis type I-S; MPS 1-S) is the mildest form of
mucopolysaccharidosis. As in Hurler syndrome, individuals with Scheie syndrome have a
deficiency of the enzyme alpha-L-iduronidase. However, in Scheie syndrome the deficiency is
specific for dermatan sulfate. Individuals with Scheie syndrome have normal intelligence, height,
and life expectancy. Symptoms include stiff joints, carpal tunnel syndrome, backward flow of
Page 3
blood into the heart (aortic regurgitation), and clouding of the cornea that may result in the loss
of visual acuity. The onset of symptoms in individuals with Scheie syndrome usually occurs
around the age of five.
Hurler-Scheie syndrome (mucopolysaccharidosis type I-H/S; MPS-IH/S) is an extremely rare
disorder that refers to individuals who have a less severe form of Hurler syndrome, but a more
severe form than Scheie syndrome. Like Scheie syndrome, affected individuals have a deficiency
of the alpha-L-iduronidase specific to dermatan sulfate. Hurler-Scheie syndrome is not as severe
as Hurler syndrome, but more severe than Scheie syndrome. Affected individuals may develop
coarse facial features, joint stiffness, short stature, clouding of the corneas, abnormally enlarged

liver and/spleen (hepatosplenomegaly), and skeletal and cardiac abnormalities. Intelligence may
be normal or mild to moderate intellectual disability may develop. Symptoms usually become
apparent between three and six years of age.
Hunter syndrome (mucopolysaccharidosis type II; MPS II) is the only type of MPS disorder
inherited as an X-linked trait. Initial symptoms and findings associated with Hunter syndrome
usually become apparent between ages two to four years. Such abnormalities may include
progressive growth delays, resulting in short stature; joint stiffness, with associated restriction of
movements; and coarsening of facial features, including thickening of the lips, tongue, and
nostrils. Affected children may also have an abnormally large head (macrocephaly), a short neck
and broad chest, delayed tooth eruption, progressive hearing loss, and enlargement of the liver
and spleen (hepatosplenomegaly). Accumulation of heparin sulfate may occur. Two relatively
distinct clinical forms of Hunter syndrome have been recognized. In the mild form of the disease
(MPS IIB), intelligence may be normal or only slightly impaired. However, in the more severe
form (MPS IIA), profound intellectual disability may become apparent by late childhood. In
addition, slower disease progression tends to occur in those with the mild form of the disorder.
Sanfilippo syndrome (mucopolysaccharidosis type III; MPS III) has four subtypes (A, B, C, and
D) that are distinguished by four different enzyme deficiencies. Initial symptoms of the four
types of Sanfilippo syndrome include hyperactivity,SLEEP DISORDERS , and delays in
attaining
developmental milestones (e.g., crawling and walking). All forms of Sanfilippo syndrome are
characterized by varying degrees of intellectual disability, progressive loss of previously
acquired skills (e.g., language), and hearing loss. Affected individuals may experience seizures,
unsteady gait, and aggressive behavior. Affected individuals may eventually lose the ability to
walk. Accumulation of heparan sulfate may occur.
Morquio syndrome (mucopolysaccharidosis type IV; MPS IV) exists in two forms (Morquio
syndromes A and B) and occurs because of a deficiency of the enzyme N-acetyl-galactosamine6-sulfatase and beta-galactosidase, respectively, resulting in accumulation of keratan and
chondroitin sulfate in type A and keratan sulfate in type B. A deficiency of either enzyme leads
to the accumulation of mucopolysaccharides in the body, abnormal skeletal development, and
additional symptoms. In most cases, individuals with Morquio syndrome have normal
intelligence. The clinical features of MPS IV-B are usually fewer and milder than those
associated with MPS IV-A. Symptoms may include growth retardation, a prominent lower face,
an abnormally short neck, knees that are abnormally close together (knock knees or genu
valgum), flat feet, abnormal sideways and front-to-back or side-to-side curvature of the spine
Page 4
(kyphoscoliosis), abnormal development of the growing ends of the long bones (epiphyses),
and/or a prominent breast bone (pectus carinatum). In some cases, hearing loss, weakness of the
legs, and/or additional abnormalities also occur.
Mucopolysaccharidosis type V is the former designation for Scheie syndrome. However when it
was discovered that both Hurler and Scheie syndromes occur due to a deficiency of the same
enzyme, Scheie syndrome was reclassified as a subtype of mucopolysaccharidosis type I.
Maroteaux-Lamy syndrome (mucopolysaccharidosis type VI; MPS VI) is characterized by a
deficiency of the enzyme N-acetylgalactosamine-4-sulfatase, resulting in accumulation of
dermatan sulfate. This form of MPS varies greatly among affected individuals. Some affected
individuals only experience a few mild symptoms, other develop a more severe form of the

disorder. Possible symptoms of Maroteaux-Lamy syndrome include coarse facial features,


umbilical hernia, a prominent breast bone (pectus carinatum), joint contractures, clouding of the
corneas, an abnormal enlargement of the liver and/or spleen (heptasplenomegaly). Skeletal
malformations and heart disease may occur in individuals with this form of MPS. In most cases,
intelligence is normal.
Sly syndrome (mucopolysaccharidosis type VII; MPS VII) is characterized by a deficiency of the
enzyme beta-glucuronidase, resulting in the accumulation of three glycosaminoglycans:
dermatan sulfate, heparan sulfate and chondroitin sulfate. The symptoms may vary greatly from
case to case. Individuals may have normal intelligence or mild to moderate intellectual disability.
Some skeletal abnormalities are often present. Hernias, clouding of the corneas, excessive
accumulation of cerebrospinal fluid in the skull (hydrocephalus), short stature, heart disease, and
coarse facial features have also been reported. In rare cases, some newborn infants with Sly
syndrome may experience abnormal accumulation of fluid in various tissues of the body
(hydrops fetalis).
DiFerrante syndrome (mucopolysaccharidosis VIII; MPS VIII) is an obsolete term for a form of
MPS described in a single individual with clinical and biochemical features of Morquio and
Sanfilippo syndromes. The disorder had been reported to be due to a deficiency of glucosamine6-sulfate sulfatase. Subsequently, this disorder was called MPS VIII (DiFerrante syndrome). Dr.
DiFerrante later found that the enzyme was normal in his patient, and the disorder had been
misdiagnosed. Therefore, DiFerrante syndrome is not a valid medical disorder.
Hyaluronidase deficiency (mucopolysaccharidosis IX; MPS IX) is an extremely rare form of
MPS characterized by a deficiency of the enzyme hyaluronidase, which is needed to breakdown
the mucopolysaccharides known as hyaluronan (hyaluronic acid). This form of MPS was first
described in 1996. Symptoms may include mild short stature, cysts, frequent ear infections, cleft
palate, and the development of soft-tissue masses. However, more cases of this form of MPS
must be identified before a clear clinical picture can be established.

Causes
All of the MPS disorders result from deficiency or malfunction of a specific lysosomal enzyme
necessary in the breaking down of dermatan sulfate, heparan sulfate, or keratan sulfate, either
Page 5
alone or together. Failure to breakdown these mucopolysaccharides results in their accumulation
in cells, tissues and organs throughout the body. All of these disorders are inherited as autosomal
recessive traits except for Hunter syndrome, which is an X-linked recessive trait.
Genetic diseases are determined by two genes, one received from the father and one from the
mother. Recessive genetic disorders occur when an individual inherits the same abnormal gene
for the same trait from each parent. If an individual receives one normal gene and one gene for
the disease, the person will be a carrier for the disease, but usually will not show symptoms. The
risk for two carrier parents to both pass the defective gene and, therefore, have an affected child
is 25% with each pregnancy. The risk to have a child who is a carrier like the parents is 50%
with each pregnancy. The chance for a child to receive normal genes from both parents and be
genetically normal for that particular trait is 25%.
X-linked recessive genetic disorders are conditions caused by an abnormal gene on the X
chromosome. Females have two X chromosomes but one of the X chromosomes is "turned off"
and all of the genes on that chromosome are inactivated. Females who have a disease gene
present on one of their X chromosomes are carriers for that disorder. Carrier females usually do

not display symptoms of the disorder because it is usually the X chromosome with the abnormal
gene that is turned off. Males have one X chromosome and if they inherit an X chromosome that
contains a disease gene, they will develop the disease. Males with X-linked disorders pass the
disease gene to all of their daughters, who will be carriers. Males cannot pass an X-linked gene
to their sons because males always pass their Y chromosome instead of their X chromosome to
male offspring. Female carriers of an X-linked disorder have a 25% chance with each pregnancy
to have a carrier daughter like themselves, a 25% chance to have a non-carrier daughter, a 25%
to have a son affected with the disease, and a 25% chance to have an unaffected son.

Affected Populations
The prevalence of all forms of mucopolysaccharidosis is estimated to be one in 25,000 births.
However, because mucopolysaccharidoses, especially the milder forms of the diseases, often go
unrecognized, these disorders are under-diagnosed or misdiagnosed, making it difficult to
determine their true frequency in the general population.
Estimates for the specific types of mucopolysaccharidosis range from: one in 100,000 for Hurler
syndrome; one in 500,000 for Scheie syndrome; one in 115,000 for Hurler-Scheie syndrome; one
in 70,000 for Sanfilippo syndrome; one in 200,000 for Morquio syndrome; and fewer than one in
250,000 in Sly syndrome. Hunter syndrome occurs predominantly in males. In extremely rare
cases, affected females have been reported. The incidence of Hunter syndrome is estimated at
one in 100,000-150,000 male births.
More than 40 distinct lysosomal storage diseases have been identified.

Related Disorders
Symptoms of the following disorders can be similar to those of mucopolysaccharidosis.
Comparisons may be useful for a differential diagnosis:
Page 6
The mucolipidoses are another subgroup of lysosomal storage disorders and produce symptoms
similar to those of the mucopolysaccharidoses. These disorders include I-cell disease, pseudohurler polydystrophy, and mucolipidosis (ML) type IV. I-cell disease (mucolipidosis type II)
resembles Hurler syndrome and the two disorders are difficult to distinguish. I-cell disease has
similar physical and mental deterioration as MPS I, but usually occurs earlier and is more severe.
I-cell disease is characterized by diffused deficiency of lysosomal enzymes within the cell and is
not associated with excretion of mucopolysaccharides in the urine. Pseudo-Hurler polydystrophy
(mucolipidosis type III) is characterized by a deficiency of multiple lysosomal enzymes needed
to break down mucopolysaccharides. ML III affects males more often than females, and can be
identified by such symptoms as claw-like hands, somewhat coarse facial features, short stature
and pain in the hands. Intelligence tends to be normal in most individuals, but mild intellectual
disability is possible. ML type IV is thought to be caused by a deficiency of transport channel
receptor protein. This deficiency may lead to the accumulation of certain fatty substances
(mucolipids) and certain complex carbohydrates (mucopolysaccharides) within the cells of many
tissues of the body. ML IV is characterized by intellectual disability; severe impairment in the
acquisition of skills requiring the coordination of muscular and mental activities (psychomotor
retardation); diminished muscle tone (hypotonia); clouding (opacity) of the clear portion of the
eyes through which light passes (cornea); and/or degeneration of the nerve-rich membrane lining
the eyes (retinal degeneration). (For more information on these disorders, choose
"mucolipidosis" as your search term in the Rare Disease Database.)
Lipid storage disorders are a group of disorders that include Fabry disease and Gaucher disease.

Fabry disease is characterized by a deficiency of the enzyme alpha-galactosidase A. Low levels


or inactivity of this enzyme leads to the abnormal accumulation of a substance consisting of fatty
material and carbohydrates (i.e., glycolipids such as glycosphingolipid) in various organs of the
body, particularly blood vessels and the eyes. Symptoms of Fabry disease may include the
appearance of clusters of wart-like discolorations on the skin (angiokeratomas), abdominal pain,
and/or visual impairment. Later in the course of the disease, kidney failure, heart irregularities,
and/or progressive neurological abnormalities may cause serious complications. Gaucher disease
is rare disorder in which deficiency of the enzyme glucocerebrosidase results in the accumulation
of harmful quantities of certain fats (lipids), specifically the glycolipid glucocerebroside,
throughout the body especially within the bone marrow, spleen and liver. The symptoms and
physical findings associated with Gaucher disease vary greatly from case to case. Some
individuals will develop few or no symptoms (asymptomatic); others may have serious
complications. Common symptoms associated with Gaucher disease include an abnormally
enlarged liver and/or spleen (hepatosplenomegaly), low levels of circulating red blood cells
(anemia), low levels of platelets (thrombocytopenia), and skeletal abnormalities. (For more
information on these disorders, choose the specific disorder name as your search term in the Rare
Disease Database.)
Multiple sulfatase deficiency is a rare hereditary metabolic disorder characterized by impairment
of all known sulfatase enzymes. Major symptoms include coarse facial features, deafness, and an
enlarged liver and spleen (hepatosplenomegaly). Abnormalities of the skeleton may occur such
as abnormal curvature of the spine (lumbar kyphosis). The skin is usually dry and scaly
(ichthyosis). Before symptoms are noticeable, children with this disorder usually develop more
Page 7
slowly than normal. They may not learn to walk or speak as quickly as other children
(developmental delays). Multiple sulfatase deficiency is inherited as an autosomal recessive trait.
(For more information on this disorder, choose "multiple sulfatase deficiency" as your search
term in the Rare Disease Database.)
Additional groups of disorders that have similar symptoms to the mucopolysaccharidoses include
glycoprotein disorders (oligosaccharidoses), which include fucosidosis and mannosidosis;
leukodystrophies, which include Krabbe's disease and metachromatic leukodystrophy; and
gangliosidoses such as Tay-Sachs disease. (For more information on these disorders, choose the
specific disorder name as your search term in the Rare Disease Database.)
http://www.med.upenn.edu/orphandisease/docs/Mucopolysaccharidoses.pdf

31) Thiamine deficiency can cause lactic acidosis due to which enzyme dysfunction
a) PEPcarboxykinase
b) PDH
c) Pyruvate carboxylase
B

Lactic acidosis due to thiamine deficiency

Thiamine participates as a cofactor in oxidative phosphorylation, and its


absence is sorely missed.
Historically, this is a cause of lactic acidosis in the
malnourished patient in who there is no other apparent cause of
lactic acidosis.

Among its many uses, thiamine acts as a coenzyme


with pyruvate dehydrogenase to form acetyl-CoA. So
of course, if you completely abolish thiamine, there
will be no entry for pyruvate into Krebs cycle. And
even if it could enter, alpha-ketoglutarate
dehydrogenase would also need thiamine as a
coenzyme to convert alpha-ketoglutarate into succynyl
coenzyme A. Thus, a deficiency of thiamine produces

an excess of lactate by preventing pyruvate from


becoming a substrate for oxidative metabolism.
It seems pretty difficult to become so thiamine
deficient that it would produce a clinically relevant
lactic acidosis. The case reports in the literature all seem
to involve people on TPN in whom multivitamins are
accidentally not administered.

Who gets thiamine deficient these days?


One can easily envision a developing world
environment where this sort of nutritional deficiency is
commonplace. However, it seems this problem can
also exist in the nutritionally competent West.
Specifically, people on long-term frusemide (a huge
proportion of the ICU population) seem to have a thiamine
deficiency due to a constant urinary loss. And yes, it is a
deficiency which is relevant to their outcome- the
patients in that study all had an improvement in their
systolic function once thiamine was replaced at 100g
IV per day, for 7 days.
In this way, one might argue that the hopelessly
shocked cardiac patient could benefit from thiamine
supplementation, and is unlikely to be harmed by it,
which is a strong argument for supplementing
everybody.

But of course these are occult thiamine deficits.


Nobody expected those frusemide-intoxicated patients
to develop a thiamine deficiency. The chronic
alcoholic, on the other hand, is at risk of this every
day; the term wet beriberi is used to describe the
cardiovascular collapse which ensues, which is typically
associated with a lactic acidosis.

Red cell transketolase is an enzyme of the pentose


pathway which is affected by the presence of thiamine;
its activity is decreased in the absence of thiamine
(which it uses as a cofactor). Its increased activity with
the addition of thiamine in the laboratory (measured by
red cell NADH consumption) is used as an indirect evidence
of thiamine deficiency.
http://www.derangedphysiology.com/php/Acid-BaseDisturbance/lactic-acidosis-due-to-thiamine-deficiency.php

32) Enzyme used for both glycogenesis and glycogenolysis is


a) Glycogen synthase
b) Phosphoglucomutase
c) Glycogen transferase
B
33) In fasting states RBC use
a) Glucose
b) Alanine

c) Ketone body
d) Fatty acid
A
FASTING STATE:

RBC can ONLY use glucose for energy

exclusively anaerobic glycolysis produces lactate which gets sent to the liver for

glucoeogenesis
http://my.ico.edu/document.doc?id=2729
34) RNAi can cause the following in a gene
a) Knock in
b) Knock out
c) Knock down
d) Knock up

C
ANS=C
Recent advances in technologies for genome editing-the use of TALEN or CRISPR to make
targeted,permanent changes to genes-have revolutionized molecular genetics. They have also
presented userswith a choice between these relatively new technologies and that of the more
established method ofRNA interference (RNAi)-mediated knockdown using short hairpin RNA
(shRNA) or short interfering RNA(siRNA). In this Technical Note, we explore the differences
between the two methods for ablating genefunction, and situations where one technology is more
appropriate than the other. RNAi-mediated gene silencing In higher eukaryotes, RNAi-mediated
knockdown is the most common strategy for depleting cells of agene product of interest. However,
RNAi usually does not completely shut off the gene. Essentially,short (approximately 20-25
nucleotides) double stranded RNA molecules are either generated fromhairpin-forming precursors
(shRNAs) or introduced exogenously (siRNAs). After processing by Dicer, asingle stranded RNA
base-pairs with a target mRNA (Ketting, 2013). Depending on the organism, RNAi-mediated gene
silencing is carried out by Argonaute proteins via either mRNA degradation or translationinhibition
(Figure 1). The end result is post transcriptional down-regulation of gene expression,
withoutchanging the genetic code (Mittal, 2004). Some functional RNA or protein remains and is
translated atlower levels. So, the RNAi strategy for reducing gene function is termed a knockdown.
Gene functionis reduced, but not eliminated
Genome editing for gene knockout By contrast, genome editing changes the genetic code, typically
causing a knockout, or completeelimination of gene function. The process begins with creation of a
double-strand break (DSB) in thechromosome (Bogdanove & Voytas, 2011). Recently, two tools
have been developed for generatingDSBs with high efficiency: Transcription Activator-Like Effector
Nucleases (TALENs), and Clustered,Regularly Interspaced Palindromic Repeat Associated
(CRISPR-Cas) proteins (Bogdanove & Voytas, 2011;Jinek, et al., 2012; Shalem, et al., 2014; Wang,
et al., 2014). Both these tools are adapted from bacterialsystems that either cause plant

pathogenesis (TALEN), or defend the genome from insertionalmutagenesis (CRISPR-Cas). TALENs


are chimeric proteins consisting of site-specific DNA binding proteinsfused to the restriction
endonuclease FokI. CRISPR-Cas uses a site-specific, 20 nucleotide single guideRNA (sgRNA) to
bring the Cas9 nuclease to its target locus. For both TALEN and CRISPR-Cas, the nucleasecuts
both DNA strands of the target. This break must be repaired or the cell will die, so eukaryotic
cellsrespond by two major mechanisms (Figure 2). The first, non-homologous end joining (NHEJ),
re-ligatesthe two free chromosome ends. However, NHEJ is error-prone, often resulting in small
insertions ordeletions that can disrupt, or knock out, the gene. Alternatively, cells can repair DSBs
throughhomologous recombination (HR), which provides researchers more options for gene
knockout. Defineddeletions can be introduced, insertional mutations can be created, or single bases
can be changed, toname a few

http://www.genesil.com/docs/Knockouts%20&%20RNAi.pdf
http://www.genecopoeia.com/wpcontent/uploads/2014/02/Technotes_Knockdown_vs_knockout.pdf

35) Specific site to which the CRE recombinase binds


a) RE site
b) LoxP site
ANS=B
Cre-lox

Recombination

In the early 1990's a new method was developed to delete a specific portion of DNA.
The procedure took advantage of the basic research performed on the bacteriophage
called P1. In this virus, there is an enzyme called cre and particular DNA sequences
called lox P sites. The lox P sites work in pairs and they flank a segment of DNA called
a
target
When the cre enzyme binds to the lox P sites, it cuts the lox sites in half and then
splices together the two halves after the target DNA has been removed
Molecular biologists recognized the specificity and utility of this viral recombination
system and put it to good use. Now if you want to excise a piece of DNA at a
particular time, all you need to do is to flank the target DNA with a pair of lox P sites
and introduce the cre enzyme when you want the target excised. Mike Snyder's group
used this to added epitope tags onto yeast proteins (Proteomics Chapter).
An additional twist is to express a Cre transgene under control of an inducible
promoter so you can delete the target DNA inside selected cells of a transgenic
organism
when
you
want
it
deleted.
http://www.bio.davidson.edu/genomics/method/CreLoxP.html

36) 10yrs old boy with muscle weakness and fatigue with increased lead in blood.
Which of following enzyme production in the liver is increased
a) ALA synthase
b) Ferrochetalase
c) PBG deaminase
d) porphobilinogen ????
ANS=A
Alterations in the activity of enzymes of haem biosynthesis in lead poisoning and
acute hepatic prophyria.
Campbell BC, Brodie MJ, Thompson GG, Meredith PA, Moore MR, Goldberg A.
Abstract
1. The activities of six of the enzymes of haem biosynthesis have been assayed in
peripheral blood from patients with lead poisoning, acute intermittent porphyria or
hereditary coproprophyria. 2. Compared with normal subjects the lead-poisoned
subjects had highly significant depression of delta-aminolaevulinate dehydratase,
coproporphyrinogen oxidase and ferrochelatase. 3. Lead-poisoned subjects had highly
significant elevation of delta-aminolaevulinate synthase activity. 4. deltaAminolaevulinate synthase activity was inversely related to the haemoglobin
concentration. 5. Increased delta-aminolaevulinate synthase and decreased deltaaminolaevulinate dehydratase activity are also found in acute intermittent porphyria.
6. Increased delta-aminolaevulinate synthase, normal prophobilinogen deaminase and
uroporphyrinogen decarboxylase and decreased coproporphyrinogen oxidase are found
in both lead poisoning and hereditary coproporphyria. 7. These enzyme changes
explain the recognized patterns of porphyrins and prophyrin precurosrs in blood and
urine in these conditions.
http://www.ncbi.nlm.nih.gov/pubmed/913057
37) In Granular dystrophy of cornea which stain is used
a) Masson trichome
b) Congo Red
A &B
Granular Corneal Dystrophy Type 1
[edit source]
Also know as Corneal dystrophy Groenouw type I.
Genetics & Inheritance[edit source]
Autosomal dominant inheritance of the TGFBI gene on the 5q31 locus.
Slit Lamp Examination[edit source]
Discrete crumb-like opacities are seen in the central anterior stroma. These deposits may appear in

early childhood, and at first may appear as more subtle fine dots or lines before they develop into the
more characteristic granules associated with this dystrophy. Early in life these granules are
separated by clear cornea. The lesions do not extend typically to the limbus, and become more
numerous over time. Late in the disease the lesions may coalesce and extend deep into posterior
stroma.
Granular Dystrophy Feldman Patient.jpeg
Homozygotes for the dystrophy have a more severe course and presentation.
Granular Dystrophy Photo--Feldman.jpg
Pathology[edit source]
Hyaline deposits that stain bright red with Masson Trichome and are weakly PAS positive.
On EM, the deposits appearance trapezoidal or rod-shaped.
Symptoms[edit source]
While some visual symptoms may occur early in life, with problems of glare and light sensitivity,
visual acuity is not usually affected until the 4th decade for most patients. Recurrent erosions may
occur. There is wide variety in patient symptoms.
Granular Corneal Dystrophy Type 2
[edit source]
Also known as Granular-Lattice Dystrophy, because it displays findings of both diseases, and
Avellino Dystrophy, because it was first described in families from Avellino, Italy.
Genetics & Inheritance[edit source]
Autosomal dominant inheritance of the TGFBI gene on the 5q31 locus.
Slit Lamp Examination[edit source]
Deposits begin to appear in early childhood (in homozygotes) or adolescence as tiny whitish dots in
the anterior stromal. Over time, these lesions progress into larger stellate, ring, or snowflake-like
opacities. Lattice lines are seen later than the granular lesions, and are usually found in deeper
stroma. Some patients go on to develop near confluent anterior granular lesions, but not as
frequently as in type 1 granular.
Pathology[edit source]
Mixed deposits of amyloid (as seen in Lattice dystrophy) and Hyaline (typical of Granular). The
amyloid stains red with Congo red stain and the hyaline stains red with Masson Trichome.
Symptoms[edit source]
Vision loss may occur earlier than in Type 1 Granular, with many patients noting vision loss in
adolescence. However, few patients drop below the 20/70 level, even late in the disease.
Recurrent erosions and photophobia are also more common in Type 2 Granular than Type 1.

38) OIL red O stain is used for

a) Frozen specimen
b) Glutaraldehyde fixed specimen
c) Alcohol fixed specimen
d) Formalin fixed specimen
A
Staining Methods: Connective Tissue, Muscle Fibers and Lipids
Results
OIL Red O
Feature
Visualization
Fat
Red
Nuclei
Blue

Sudan Black B
Feature
Visualization
Fat
Blue-black
Nuclei
Red

Technical Considerations

For demonstration of simple lipids, frozen sections must be used.

Frozen sections may be unfixed or post-fixed in 10% neutral buffered formalin or


formol-calcium.

Tissue must not be exposed to fat solvents such as alcohols, acetone, xylene or
paraffin.

Aqueous mounting media must be used when coverslipping finished slides.


References
1. Sheehan DC, Hrapchak BB. Theory and practice of histotechnology. 2nd ed. Columbus, OH:
Batelle Press; 1980. p. 180213.
2. Carson FL. Histotechnology: a self-instructional text. 2nd ed. Chicago: American Society of
Clinical Pathologists Press; 1996. p. 13156.
3. Luna LG. Histopathologic methods and color atlas of special stains and tissue artifacts.
Downers Grove, IL: Johnson Printers; 1992. p. 399479.
4. Connective tissue and muscle morphology and staining. Bowie, MD: National Society for
Histotechnology; 1990.
5. Bancroft J, Gamble M. Theory and practice of histological techniques. 5th ed. New York:
Churchill Livingstone

http://www.lab.anhb.uwa.edu.au/hb313/main_pages/timetable/Tutorials/2008/DAKO.guide_to_s
pecial_stains.pdf

39) In the following karyotype what is the abnormality seen. [PICTURE] Shows 2 Xchromosomes and 1 Y-chromosome.
a) Gynecomastia with long thin limbs
b) Short stature with polydactyly
c) ?
d) ?

A
40) [PICTURE] ?Immunofluorescence study. This photo was taken by attaching the
camera to the microscope. What is the requirement for such a microscope
a) Dark field condenser
b) Phase shifter
c) Dichroic mirror
d) ???
ANS=C
The basic principle of immunofluorescence
To use a fluorescent compound (usually fluorescein) to detect the binding of antigen and antibody
The Ab is labelled with the fluorescent compound
Under a fluorescence microscope, fluorescein appears bright green wherever the binding occurs

Green fluorescence of FITC

Using the fluorescence microscope


Select the correct filter block for the fluorescent compound
Fluorescence fades quickly under UV light; try to limit the time of exposure to UV as much as possible
Use high speed films for photography

Direct Immunofluorescence
The aim is to identify the presence and location of an antigen by the use of a fluorescent labelled specific
antibody

One step
Direct Immunofluorescence

Two step
Direct Immunofluorescence

Medical applications of direct IF

Renal diseases for evidence of immune deposition


Skin diseases for evidence of immune deposition
Detection of specific antigens, especially those of infective organisms

Application in renal diseases


IgG

A section of kidney is placed on a slide; a fluorescein-labeled antiglobulin (specific for IgG, in this case) is added,
then rinsed away
The presence of fluorescence in the glomeruli indicates that IgG was deposited prior to the biopsy
IgG is deposited in granular clumps along the capillary walls, enabling a diagnosis of membranous
glomerulonephritis in this case

A section of kidney is placed on a slide; a fluorescein-labeled antiglobulin (specific for IgG, in this case) is added,
then rinsed away
The presence of fluorescence in the glomeruli indicates that IgG was deposited prior to the biopsy
IgG is deposited in granular clumps along the capillary walls, enabling a diagnosis of membranous
glomerulonephritis in this case

Direct Fluorescent Antibody Test for the Presence of Immunoglobulin Deposits in Skin
IgG

A section of skin is placed on a slide; a fluorescein-labeled antiglobulin (specific for IgG, in this case) is added,
then rinsed away
The presence of fluorescence in the upper layers of the epithelium indicates that IgG was deposited in this skin
(prior to the biopsy)
The presence of immunoglobulins deposited around keratinocytes is consistent with a diagnosis of pemphigus

A section of skin is placed on a slide; a fluorescein-labeled antiglobulin (specific for IgG, in this case) is added,
then rinsed away
The presence of fluorescence in the upper layers of the epithelium indicates that IgG was deposited in this skin
(prior to the biopsy)
The presence of immunoglobulins deposited around keratinocytes is consistent with a diagnosis of pemphigus

Double labelling
Lymphoid tissue:
the two Ig light chains are separately labelled.

Indirect Immunofluorescence

Indirect Immunofluorescence
The aim is to identify the presence of antigen specific antibodies in serum. The method is also be used to compare
concentration of the antibodies in sera.
Indirect Immunofluorescence
A known antigen is placed on a slide; the patient's serum is added, then rinsed away.
A fluorescein-labeled antiglobulin is added, then rinsed away.
The presence of fluorescence over the antigen indicates the presence of antibodies to this antigen in the patient.

Diagnosis of Bacterial Diseases


Clostridial diseases (direct)
Brucella canis (indirect)
Afipia catei, cat scratch disease (indirect)
Borrelia burgdorferi (indirect)
Coxiella burnetii, Q Fever (indirect)
Rickettsia rickettsiae, Rocky Mountain Spotted Fever (indirect)

Diagnosis of Viral Diseases


rabies virus (direct)
bovine immunodeficiency-like virus (indirect)
canine coronavirus (indirect)
canine distemper (indirect)
feline infectious peritonitis (corona-) virus (direct)
porcine respiratory and reproductive syndrome (indirect)

Diagnosis of Protozoal Diseases

Babesia species (indirect)


Ehrlichia species (indirect)
Toxoplasma gondii (indirect)
Trypanosoma cruzi (indirect)
Cryptosporidia/Giardia (direct)
Encephalitozoon cuniculi (indirect)
Neosporum caninum (direct, indirect)

Some examples
Indirect Immunofluorescence

Indirect Fluorescent Antibody Test for Antibodies to Toxoplasma gondii

Indirect Fluorescent Antibody Test for Antibodies to Toxoplasma gondii


Toxoplasma organisms are killed and placed on the slide; the patients serum is added, then washed away.
A fluorescein-labeled antiglobulin is added, then washed away.
The presence of the green fluorescence outlining the T. gondii organisms indicates the presence of antibodies in
the patient's serum.

Indirect Fluorescent Antibody Test for Antibodies to Toxoplasma gondii

Immune-Mediated Disorders
antinuclear antibody (ANA) test (for diagnosis of systemic lupus erythematosus)
Direct fluorescent antibody test for deposition of Abs in tissues, e.g. kidney, skin

Indirect Fluorescent Antibody Test for Antinuclear Antibodies

Indirect Fluorescent Antibody Test for Antinuclear Antibodies


Cells from a cultured cell line are placed on a slide; the patient's serum is added, then rinsed away.
A fluorescein-labeled antiglobulin is added, then rinsed away.

The presence of fluorescence in the nucleus of these cells indicates the presence of antibodies to nuclear antigens
in the patient.

Indirect Fluorescent Antibody Test for Antinuclear Antibodies

Advantage over Immunoperoxidase


Technically easier (fewer steps)
More sensitive results

Drawbacks
Microscope is more costly
Frozen sections preferred
Preparations need refrigeration
Preparations cannot be kept for too long
Quick fading of fluorescence under illumination (bleaching effect)

Part II: Confocal microscopy

Learning Objectives
To understand the working principles of confocal scanning microscope
To know the current use of confocal scanning microscopy in medical science

Principles of confocal microscopy


a focused laser beam serves as a high intensity point source
light reflected or fluorescence emitted by the specimen is allowed to pass through a pinhole that filters light
coming from outside (above and below) of the focal plane

Principles of confocal microscopy


a sensitive detector (photomultipler) behind a pinhole to measure the intensity of light
the laser beam, the pinhole and detector scan through the specimen to build up an image on a monitor

The confocal
concept

Modes of scanning
Mechanical scanning stage
Beam scanning (by means of mirror)
Combined stage and beam scanning
Slit may be used instead of a pinhole
Shortens time for scanning an area
Direct vision of real color image made possible
At the expense of a lower resolution

Use of confocal microscope


Performs optical sectioning of thick samples
Three dimensional image reconstruction
Detects very weak fluorescent signals
Selective photobleaching
Cell ablation

Image modalities
Autofluorescence
Single, double or treble fluorescent labeling:
immunofluorescence, in-situ hybridization
Image formed by reflectance intensified with metallic coating e.g. AgNOR, immunogold labeling

Application in biomedical science


Growth of small organisms, cells, embryos
Movement of intracellular structures
Change in membrane permeability
3 dimensional reconstruction
Image analysis

BIO-RAD MRC-1000
beam scanning LSM
Zeiss "Axioskop" upright microscope for ordinary bright field transmitted light and epi-fluorescence

argon/krypton laser, 15 mW, wavelengths at 488 nm (e.g. FITC), 568 and 647 nm
reflectance mode
fluorescence mode:
simultaneous recording of 2 channels, ie. For double labels

BIO-RAD MRC-1000
upgradable to 3 simultaneous channels in fluorescence mode
computer-controlled stage motor for vertical motion at 0.5m per step
performs frame scan, line scan, vertical scan
maximum image size 1024 x 1024 pixels
scanning speed at maximum of 16 frames/s at 768 x 30 pixels, and 1 frame/s at 512 lines

BIO-RAD MRC-1000
beam park function for photobleaching, ablation
computer programs for morphometric analysis, 3-D visualization
IgG
IgG
Confocal
Confocal microscope
gives a clearer image and
cleaner background over
conventional fluorescent
microscope
Microphotography
for weak signals
is much easier
with a confocal
microscope
Microphotography
for weak signals

is much easier
with a confocal
microscope
Excellent for picking up weak signals
Immunofluorescence
and
Confocal Microscopy
Dr. KW Chan

Learning Objectives
To understand the working principles of immunofluorescence microscope
To understand the difference between direct and indirect immunofluorescence
To know the current use of immunofluorescence studies in medicine

Learning Objectives
To understand the working principles of confocal scanning microscope
To know the current use of confocal scanning microscopy in medical science

http://www.patho.hku.hk/docs/mmedsc/mms-ifcf.ppt

41) Platelet adhesion to vessel wall is due to


a) Factor IX
b) Fibrinogen
c) vWF
C
42) A10 yr old boy with mass in the abdomen. On imaging the paraaortic LN is
enlarged. On biopsy starry sky appearance is seen. What is the underlying
abnormality
a) p53 gene mutation
b) RB gene mutation
c) Translocation involving BCR-ABL genes
d) Translocation involving MYC gene
D

43) A child with edema and decreased urine output. On evaluation, serum albumin is
2.5 g/dL, S.Creatinine is 0.5 mg/dL, urine protein is 3+ with no RBC casts.
Pathological change expected is
a) Minimal change disease
b) Interstitial nephritis
c) IgA nephropathy
A
44) Most common nephropathy associated with malignancy
a) Membranous
b) MCD
c) IgA
d) FSGS
A
Membranous nephropathy: A frequent type of primary glomerular disease -- a disease affecting the
glomerulus, the tiny ball-shaped structure in the kidney composed of capillary blood vessels that is
actively involved in the filtration of the blood to form urine. The main sign of the disease is marked
proteinuria (protein in the urine).
The majority (about 2/3) of cases are primary (or idiopathic), meaning the cause is not known. The
remaining cases are secondary to conditions such as cancer, infection, and drug side effects.
Idiopathic MN is the most common cause of primary nephrotic syndrome in older (>60 years)
Caucasian adults and is rare in children. It usually accounts for less than 5% of pediatric patient
undergoing biopsy for nephrotic syndrome. About 30% of all biopsy specimens for primary nephrotic
syndrome reveal membranous nephropathy in adults and about 50% in older Caucasian adults. A
large number of agents appear to be capable of initiating membranous nephropathy in genetically
susceptible individuals. More common agents include infections such as hepatitis B (less
commonlyhepatitis C and syphilis), immune diseases such as lupus and diabetesmellitus (less
commonly associated with rheumatoid arthritis and other connective tissue diseases),
medications such asGOLD , penicillamine, non-steroidal anti-inflammatory agents,
and captopril, and some tumors (the colon, kidney, and lung are the most common primary sites).
Membranous nephropathy has been associated with the chronic immune response to renal
transplants as well. Without intervention, about 40% of people with membranous nephropathy
progress to end-stage renal failure after 10 years. Conversely, up to 30% of people with have a
spontaneous remission and approximately 30% of people will have stable renal function. Female sex
and lower grade (non-nephrotic) proteinuria at presentation are the only two features associated with
a higher likelihood of spontaneous remission. Although the majority of membranous nephropathy
patients do reasonably well long term, membranous nephropathy is still the second or third leading
cause of end-stage renal failure among subjects with primary glomerulonephritis. Factors associated
with worse renal survival in membranous nephropathy include older age at presentation, male sex,
hypertension, decreased renal function (elevated creatinine and/or decreased GFR) at time of

presentation, higher level of proteinuria, and evidence of chronic features (scarring) on biopsy.
Relapses after complete remission have occurred in approximately 25% to 40% of patients.
Relapses have been reported up to 20 years after the primary remission. the great majority of
patients will relapse only with low-range proteinuria and will maintain stable long-term kidney
function with conservative management alone. In contrast, the relapse rate is as high as 50% in
those achieving only a partial remission. Achievement of either a complete or partial remission,
however, significantly slows progression and increases renal survival.
Membranous nephropathy is a glomerular immune-complex disease. Immune deposits form in the
glomerular basement membrane that cause a membrane-like thickening. The constituent immune
complexes consist of IgG. Cells called podocytes and their membrane-associated proteins play a
pivotal role in the development of the disease by providing antigenic targets for circulating antibodies
to form the glomerular immune deposits. Membranous nephropathy is also called membranous
glomerulonephritis.
The disease can occur before birth due to transmission of antibodies from the mother to the baby.
The antibodies are directed against an antigen in podocytes called neutral endopeptidase (NEP).
Mutations that truncate (shorten) the metallomembrane endopeptidase (MME) gene, which encodes
NEP, have been found to cause the neonatal form of this disease

http://www.medicinenet.com/script/main/art.asp?articlekey=39513

45) The phenomenon where subsequent generations are at risk of earlier and more
severe disease is known as
a) Anticipation
b) ??
A
46) In Langerhans Cell Histiocytosis, the characteristic abnormality seen is
a) Birbecks granules
A
47) A 45 yr old patient presents with fever, night sweats, weight loss. On X ray a mass
in apical lobe of lung is seen. On histopathology found to have caseous necrosis. What
is the underlying process
a) Enzymatic degeneration
b) Hypersensitivity reaction with modified macrophages, lymphocytes and giant cells
c) Acute decrease in blood supply ?
d) ??
???
48) Forward scatter in flow cytometry is used to assess

a) Cell death
b) Cell size
c) Cell granules
d) florecence
ANS=B
Light scattering occurs when a particle deflects incident laser light. The extent towhich this occurs
depends on the physical properties of a particle, namely its size andinternal complexity. Factors that
affect light scattering are the cell's membrane,nucleus, and any granular material inside the cell. Cell
shape and surface topographyalso contribute to the total light scatter.Forward-scattered light (FSC)
is proportional to cell-surface area or size. FSC is ameasurement of mostly diffracted light and is
detected just off the axis of the incidentlaser beam in the forward direction by a photodiode (Figure
3-1 on page 14). FSCprovides a suitable method of detecting particles greater than a given size
independentof their fluorescence and is therefore often used in immunophenotyping to triggersignal
processing
Side-scattered light (SSC) is proportional to cell granularity or internal complexity.SSC is a
measurement of mostly refracted and reflected light that occurs at anyinterface within the cell where
there is a change in refractive index (Figure 3-1). SSC iscollected at approximately 90 degrees to the
laser beam by a collection lens and thenredirected by a beam splitter to the appropriate detector

49) Collagen typical of basement membrane


a) Type I
b) Type V
c) Type IV
d) Type III
C
50) Best hypertensive drug in pulmonary hypertension
a) Bosentan
b) Amlodipine
A
51) Antihypertensive not used in pregnancy
a) Enalapril
b) Nifidipine
A
52) DoC for hypertension in pregnancy

a) Enalpril
b) Verapamil
c) Alpha Methyldopa
C
53) Hyperchromia iridis is caused by
a) Latanoprost
b) Prednisolone
c) Timolol
d) Olapatadine
A
Contributor: Gordon K. Klintworth
Heterochromia iridis occurs in several conditions (including Waardenburg syndrome, Fuchs
heterochromic iridocyclitis, melanosis oculi, iris melanoma [melanoma - iris] and Horner syndrome).
The iris may be excessively pigmented (as in melanosis oculi,, a primary iris melanoma or an iris
that has been invaded by a melanoma from the ciliary body [melanoma - ciliary body]. In some of
conditions, such as Waardenburg syndrome, the heterochromia can be segmental and only involve
part of the iris. In Horner syndrome the iris is ligher than normal on the affected side. Because the
iris originally has a normal color Horner syndrome illustrates that an intact sympathetic innervation of
the iris is important in the maintaining iris pigmentation. Certain drugs, such as latanoprost
[latanoprost complications] can also make the iris darker than normal.

https://eyepathologist.com/disease.asp?IDNUM=327970

54) Adrenergic beta receptor having lipolysis property in fat cells


a) Beta 1
b) Alpha 2
c) Beta 3
d) Alpha 1
C

Autonomic Receptors

Autonomic receptors are broadly divided into those for ACh (cholinergic receptors) and
those for catecholamines such as NE or EPI (adrenergic receptors or adrenoceptors).
As we shall see later, subdivisions of each of these families of receptors are based on
sensitivity to various agonist and antagonist drugs.

Cholinergic receptors were originally divided and named on the basis of sensitivity to
alkaloids that mimicked some, but not all, of the actions of ACh. These alkaloids are
present in the fly agaric mushroom, Amanita muscaria, and in the tobacco plant,
Nicotiana tabacum. The alkaloids are muscarine and nicotine, respectively. Thus,
cholinergic receptors are broadly classified as muscarinic (mAChR) or nicotinic
(nAChR). The 3-dimensional structures of muscarine and nicotine molecules are fairly
rigid and differ significantly. By contrast, ACh can assume 3-dimensional structures that
are similar to muscarine on the one hand and nicotine on the other hand. Thus, one
neurotransmitter molecule can interact with quite different receptor proteins, based on
its molecular flexibility.

Nicotinic receptors are located postsynaptically in all autonomic ganglia and at the NMJ.
At these junctions nicotinic receptors function as the excitatory receptor for the
postsynaptic cell. Release of a sufficient quantity of ACh from the adjoining presynaptic
cell causes an excitatory response in autonomic ganglion cells and in somatic muscle
fibers.

Muscarinic receptors are located postsynaptically at the parasympathetic neuroeffector


junction. At these junctions muscarinic receptors function either to increase or decrease
the activity of the effector cells. Muscarinic receptors are also located postsynaptically at
the neuroeffector junction of sympathetic fibers in sweat glands. At this junction
muscarinic receptors function to increase sweating. Drugs that block muscarinic
receptors can thus interfere with sweating. Another important site of muscarinic
receptors is on endothelial cells of blood vessels. Although these muscarinic receptors
are not innervated by cholinergic nerve fibers, they are sensitive to circulating
molecules.
In 1948, adrenergic receptors were subdivided into alpha and beta by Ahlquist (a
graduate of the University of Washington). The distinction was based on sensitivities of
different organs to catecholamines of closely related structure. Regulation of the
functions of different organs depends to a greater or lesser extent on alpha or beta
receptors. Pharmacological differentiation of alpha and beta receptors, and application
of this technology to the treatment of disease, is an outstanding biomedical
achievement.

Alpha receptors are located postsynaptically at sympathetic neuroeffector junctions of


many organs. In general, alpha receptors mediate excitation or increased activity of the
effector cells. Vascular smooth muscle is an important site of alpha receptors. SNS

activity maintains vascular tone, and thus blood pressure, by maintaining a tone of
neurotransmitter on vascular alpha receptors.
Beta receptors are also located postsynaptically at sympathetic neuroeffector junctions
of many organs. In general, beta receptors mediate relaxation or decreased activity of
the effector cells. Thus, blood vessels dilate and uterine smooth muscle relaxes in
response to activation of beta receptors. Heart muscle is an important exception to this
rule. Activation of beta adrenoceptors in heart increases the automaticity and
contractility of all parts of the heart.

Cholinergic receptors

Cholinergic, receptors mediate the actions of acetylcholine (ACh). Other choline esters
such as methacholine (Mecholyl) or carbamylcholine (Carbachol) mimic the actions
of ACh at the same receptors. Cholinergic receptors are subdivided into:
muscarinic
nicotinic

Ganglionic and other neural muscarinic receptors (M1) are apparently involved in CNS
transmission. They may modulate classical ganglionic transmission process. They have
been implicated in promoting the late EPSP of ganglion cells.. These receptors are
classified as muscarinic because they are blocked by atropine. Oxotremorine and McNA-343 activate these receptors somewhat selectively. Pirenzepine is a relatively
selective antagonist of M1 receptors.
Classical muscarinic receptors (M2) (and M3, not shown) subserve effects such as
salivation, urination, defecation, pupillary constriction, vasodilation, cardiac slowing,
depressed AV nodal conduction, and bronchoconstriction. The classical site of
muscarinic receptors is at the postganglionic parasympathetic neuroeffector junction in
smooth muscle, heart, and exocrine glands. However, muscarinic receptors also exist
on effectors cells, even in the absence of cholinergic innervation. Prototype agonists
include muscarine and ACh. The prototype non-selective antagonist is atropine. AF-DX
116 is a selective antagonist of M2 receptors and hexahydrosilafenidol is a selective
antagonist of M3 receptors.
The main contribution of such selective experimental agents is that they provided
evidence for the existence of more than one type of muscarinic receptor, as well as
raising the possibility of pharmacologically selective agonism or antagonism. Tiotropium,
a selective antagonist of M3 receptors has recently been approved for treatment of
chronic obstructive pulonary disease (COPD). One of the advantages tiotropium (over
atropine, for example) is that it generally blocks only postsynaptic M3 receptors (that
promote bronchoconstriction) but does not block presynaptic M2 receptors (that inhibit
the release of ACh). Blockade of presynaptic M2 receptors tends to increase the release
of acetylcholine and worsen COPD symptoms). Thus, tiotropium is a prime example of

the therapeutic significance of being able to pharmacologically distinguish between


receptor subtypes.
Modern molecular biology techniques have identified at least 5 different muscarinic
receptors. At present, the pharmacologic and/or therapeutic significance of M4 and M5
is not yet definitive

Nicotinic receptors are cholinergic receptors which, when activated, mediate most of the
actions of nicotine and some of those ACh. Examples of nicotinically-mediated effects
include the release of catecholamines from the adrenal medulla, ganglionic
transmission, and transmission of the somatic neuromuscular junction (NMJ).
Autonomic and somatic nicotinic receptors are distinct pharmacologically.
Ganglionic nicotinic receptors, (NN) are receptors for which classical agonists include
ACh and dimethylphenylpiperazinium (DMPP) and whose antagonists are typified
bymecamylamine (Inversine), trimethaphan (Arfonad) or hexamethonium.
Somatic muscle nicotinic receptors, (NM) are receptors for which prototype agonists
include nicotine, and ACh. Phenyltrimethylammonium (PTMA) is a selective agonist.
The classical antagonist of the NM receptor is the non-depolarizing neuromuscular
junction blocker, d-tubocurarine (Tubarine). An ingredient of snake venom, alphabungarotoxin, binds almost irreversibly to and blocks the NM receptor.
Drugs which act initially as depolarizing neuromuscular junction blockers (NM agonists),
and then produce neuromuscular junction blockade even when the motor end plate has
repolarized, include decamethonium and succinylcholine (Anectine). It may be that
these agents produce initial block by depolarizing the motor-end plate and then,
because they are not rapidly broken down by acetylcholinesterase, promote receptor
desensitization.

This diagram summarizes the cholinergic receptor system and some of its agonists and
antagonists. The arrows are meant to show the range of selectivity of the various drugs
for most practical purposes. Apparent selectivity of agonists or antagonists may be lost
at relatively high concentrations. Relative affinities of a given drug for different receptors
may be implied, but the affinities of different drugs are not shown in this qualitative
guide. For such information, you will need to consult data on the apparent Kd, IC50 or
other comparable measurements.

Adrenergic receptors

Adrenergic receptors, also called adrenoceptors, mediate the actions of epinephrine


(Adrenalin) and related compounds. Adrenergic receptors can be acted upon by a
variety of agonists and antagonists, most of which are related to beta-phenylethylamine.
The classical site of adrenergic receptors is the sympathetic neuroeffector junction. As
in the case of cholinergic receptors, however, adrenergic receptors may exist on the
cells of effector organs, even in the absence of sympathetic innervation. In other words,
not necessarily all adrenergic receptors are innervated. Adrenergic drugs mimic the
actions of epinephrine and are also called sympathomimetics. Direct-acting
sympathomimetics act by direct stimulation of adrenergic receptors.

Adrenergic receptors may be divided into two major types according to drug (especially
antagonist) potency on the receptors. Alpha receptors, when activated, generally
produce excitatory responses of smooth muscle in which they are located. Beta
receptors, when activated, generally produce inhibitory responses of smooth muscle in
which they are located.
Alpha receptors may be subdivided into:
Alpha1 receptors are sensitive to blockade by prazosin (Minipress). Alpha1
receptors are mainly postsynaptic.
Alpha2 receptors are sensitive to blockade by yohimbine. Alpha2 receptors are
found mainly presynaptically on sympathetic postganglionic nerve terminals.
Activation of the presynaptic alpha2 receptors inhibits the release of NE and the
term autoreceptor came into fashion. Some of the pharmacological effects of
alphamethyl-norephinephrine (functioning as a false transmitter produced
from alphamethyl DOPA) may be due to activation of alpha2 receptors and
selective presynaptic inhibition of the release of NE.
Recently it has been shown that some alpha2 receptors are present postsynaptically so
the distinction between alpha1 and alpha2 receptors should be considered functional,
rather than anatomic. Molecular biology has subdivided the alpha1receptors into
alpha1A, alpha1B, and alpha1D. The alpha2 receptors subdivide into alpha2A (probably
is the presynaptic autoreceptor), alpha2B and alpha2C. The major antihypertensive
action of selective alpha agonists such as clonidine (Catapres), probably occurs as a
result of agonism of alpha2A receptors in the CNS.

Beta receptors when activated, generally produce "inhibitory" responses of smooth


muscle in which they are located. Bronchodilation and vasodilation in some vascular
beds can be produced by activation of beta-receptors. The classical agonist of betareceptors is isoproterenol (Isuprel). The classical antagonist is propranolol (Inderal).

Some beta-receptors subserve "excitatory" responses. For example, in heart, betareceptors are excitatory. Sympathetic positive chrono- and inotropism are mediated
through excitation of beta-receptors. Therapeutic differentiation of beta-receptors is well
established. Thus, we may consider:
Beta1 receptors, when activated, produce cardiac positive chrono- and inotropic
responses and lipolysis. The beta receptors of the heart are somewhat unique. For
example, they produce cardiac excitation and, although NE does not act on betareceptors in vascular smooth muscle, it is equipotent with epinephrine in stimulating the
heart. Isoproterenol acts as an agonist of beta1 receptors. Selective antagonists of
beta1 receptors include metoprolol(Lopressor) and CGP 20712A.
Beta2 receptors, when activated, produce bronchodilation, vasodilation, and uterine
relaxation. Selective beta2 agonists, which are useful in the treatment of asthma,
includeterbutaline (Brethine). Propranolol, which blocks both beta1 and beta2
receptors is generally contraindicated in bronchial asthma. ICI 118551 is a selective
antagonist of beta2 receptors. labetalol (Normodyne) is a clinically useful drug which is
a selective alpha1 antagonist, and a non-selective antagonist of both beta1 and beta2
receptors.
Beta3 receptors, when activated, produce lipolysis in adipose tissue. The significance
and therapeutic application of this knowledge is still in its infancy. BRL 37344 is a
relatively selective agonist of beta3 receptors. Antagonists include ICI 118551 and CGP
20712A, each of which is selective for beta1 and beta2 receptors, respectively.

This diagram summarizes the adrenergic receptor system and some of its agonists and
antagonists. The arrows are meant to show the range of selectivity of the various drugs
for most practical purposes. Apparent selectivity of agonists or antagonists may be lost
at relatively high concentrations. Relative affinities of a given drug for different receptors
may be implied, but the affinities of different drugs are not shown in this qualitative
guide. For such information, you will need to consult data on the apparent Kd, IC50 or
other comparable measurements.

https://courses.washington.edu/chat543/cvans/sfp/ansrec.html

55) Methacholine has main agonist action at


a) M2
b) M1
c) M4
d) M3
D
Airway smooth muscle constriction induced by cholinergic agonists such as
methacholine (MCh), which is typically increased in asthmatic patients, is regulated
mainly by muscle muscarinic M3 receptors and negatively by vagal muscarinic M2
receptors.
http://www.pubfacts.com/detail/23691511/Role-of-m2-muscarinic-receptorin-the-airway-response-to-methacholine-of-mice-selected-for-minimal-o

56) Diuretic which can be given in mild to moderate hypertension

a) Loop diuretic
b) Thiazide
c) Osmotic diuretic
d) Potassium sparing diuretic
B
57) All of these drugs can cause Hearing loss except
a) Metronidazole
b) Vancomycin
c) Kanamycin
d) Quinine
A
58) Which of the following is a protease inhibitor
a) Saquinavir
b) Nevirapine
c) Abacavir
d) Enfuvirtide
A
59) What is the type of inhibition of Acetylcholinesterase caused by
Organophosphates
a) Competitive
b) Non competitive
c) Uncompetitve
d) irreversible inhibitor
D
60) Which does not act by increasing insulin secretion for its action
a) Exenatide
b) Sitagliptin
c) Rosiglitozone
d) Repaglinide
C
ANS=Rosiglitozone
The thiazolidinediones rosiglitazone and pioglitazone are insulin sensitizers.
Troglitazone, another thiazolidinedione, was removed from the MARKET in 2000
because of hepatotoxicity. These drugs bind to peroxisome proliferator-activated

receptors (PPARs) in cells, and this drug-PPAR complex (with one or more
coactivators) acts on response elements in promoter regions to affect the
transcription of as many as 100 genes. They may act to stimulate production of
proteins that increase insulin sensitivity, such as adiponectin.14,15 They may also act
by blocking transcription of other proteins responsible for insulin resistance or
inflammation.14,16 PPARs exist in several different forms: PPAR, PPAR, and PPAR.
PPAR is the major target of thiazolidinediones
http://clinical.diabetesjournals.org/content/25/4/131.full

61) Nitroglycerine is effective as sublingual medication because


a) Non Ionic, highly lipid soluble
b) Ionic, lipid soluble
c) Ionic, less lipid soluble
d) Non ionic, less lip soluble
A
62) In a child admitted with H. Influenza meningitis, Cefotaxime was started instead
of ampicillin. Which of these is the likely reason for this
a) H.influenzae stains known to produce Beta lactamase
b) H.influenzae stains known to have altered pencillin binding protein
c) More easier to give
d) ??
A
Antibiotics and supportive care
These are the mainstays of treatment.
Initially, invasive and serious H influenzae type b (Hib) infections are best treated
with an intravenous third-generation cephalosporin until antibiotic sensitivities
become available. In Malawi, Africa, intramuscular ceftriaxone was compared with
intravenous ceftriaxone and was not found to increase the mortality rate. This may be
important in developing countries where the intravenous route may not be possible.
[12]
The site of infection and the clinical response determine the length of antibiotic
treatment.
Meningitis
Administer parenteral antibiotics (eg, ceftriaxone, ceftazidime, cefotaxime,
ampicillin-sulbactam, fluoroquinolones, azithromycin) to patients with uncomplicated
meningitis for 7-14 days. Cefotaxime and ceftriaxone are the initial drugs of choice
for suspected Hib meningitis.
Once the susceptibilities are known, adjust antibiotics accordingly.
Do not use ampicillin empirically, since as many as 50% of the isolates are resistant,

usually because of plasmid-mediated beta-lactamase production.


63) Reverse transcriptase is a RNA dependent DNA polymerase. Which of these use it
a) Hepatitis A virus
b) Hepatitis B virus
c) Hepatitis E virus
d) Hepatitis C virus
B
64) A vesicle on shin microscopy on Tzank smear showed giant cells. Causative
agents is
a) Vaccinia virus
b) Varicella zoster
c) Tuberculous
d) Molluscum contagiasum
B
65) Vibrio cholerae acts by disrupting which of the following
a) Hemidesmosome
b) Gap junctions
c) Anchoring junctions
d) Tight junctions
D
Vibrio cholerae produces a second enterotoxin, which affects intestinal tight
junctions.
Attenuated Vibrio cholerae vaccine strains specifically mutated in genes encoding
cholera toxin (CT) are still capable of causing mild to moderate diarrhea. Culture
supernatants of V. cholerae strains, both CT-positive and CT-negative, were examined
in Ussing chambers, and a toxin was found that increases the permeability of the
small intestinal mucosa by affecting the structure of the intercellular tight junction,
or zonula occludens. The activity of this toxin is reversible, heat-labile, sensitive to
protease digestion, and found in culture supernatant fractions containing molecules
between 10 and 30 kDa in size. Production of this factor (named ZOT for zonula
occludens toxin) correlates with diarrheagenicity of V. cholerae strains in volunteers
and may represent another virulence factor of infectious diarrhea that must be
eliminated to achieve a safe and effective live oral vaccine against cholera.
http://www.ncbi.nlm.nih.gov/pubmed/2052603

66) Filarial stage of adult worms responsible for diseases in all of the following
except:
a) Onchocerca volvulus
b) Brugia malayi
c) Wuchereria bancrofti
d) Mansonella ozzardi
ANS=D
Mansonella ozzardi
Mansonella ozzardi nematodes are confined to the westerm hemisphere. These are
non-pathogenic filarial nematodes. The parasites cause nodules in the skin of the
vertebrate hosts.
Morphology
The adult worms are located in the mesenteric tissues and their size is comparable to
the pathogenic species already discussed (0.6m long). The microfilariae are found in
the peripheral blood and range between 173-240m in length. The nuclei do not
extend to the tip of the tail which has a pointed end. The male adult worm is almost
unknown.
Clinical Disease
Infections caused by M. ozzardi are generally symptomless, however
lymphadenopathy, arthralgia, fever and eosinophilia have been reported.
http://www.phsource.us/PH/PARA/Chapter_10.htm

67) IgE receptor present on


a) Mast cell
b) NK cell
c) B cell
d) ??
A
68) Two bodies are found outside the car following an accident. The doctor
conducting the autopsy was able to decide who was driver and who was passenger
based on all of these features except
a) Sparrow foot mark
b) Seat belt abrasion over Left shoulder
c) Steering wheel imprint
d) Whiplash injury?

B
ans=b
Front seat passenger/ Whiplash injury: It is due to a violent acceleration or deceleration
force applied to the passenger, usually front seat occupant. X. He may not get the
momentary warning of the impending collision. XI. There may be peculiar facial lacerations
due to contact with the shattered windscreen known as Sparrow Foot marks. XII.
Passenger of the rare seat often escape such injuries, they may be injured against
internalFITTINGS ,

like

Doors,

Handles

or

ejected

through

burst-open

doors.

http://www.slideshare.net/neharikasingh9678/road-traffic-injury
http://www.slideshare.net/neharikasingh9678/road-traffic-injurY
Whether the victim was the driver or the passenger? Some times it is necessary to know who was driving the
vehicle for insurance purpose. Following can assist the autopsy surgeon in determining if a particular occupant was
the driver. Steering wheel impact abrasions may be seen on the chest. Dicing injuries on the right side of the
body. Pattern seat belt abrasions is seen on the right side of the shoulder going diagonally across the chest of the
left. Imprint marks of the break and clutch pedals on the sole of shoe is pressed at the time of impact.
45. In different jurisdictions autopsy, surgeons may rule the manner of death in Hit and run pedestrian fatalities
as HomicideA or accidentA or undeterminedA depending on the existing protocol. Alcohol, Drugs And
Trauma: Alcohol and substance abuse are major associated factors in all major trauma. About 10% of drivers
with blood alcohol level higher then the legal limit account of nearly 1/3rd of non fetal and half of fatal driver
deaths.

69) Lower two parts of the sternum fuses with its body by age
a) 12 years
b) 14 years
c) 8 years
d) 10 years
B
In cartilaginous sternum, five double bony centres appear from above downwards during fifth, sixth,
seventh, eighth and ninth fetal months.
The upper centre forms the manubrium.
Remaining four form body of sternum and fuse with one another from below to upwards.
Lower two centres fuse at age of 14 years and upper two centres fuse between 14 to 25 years.
The centre of xiphoid process appears during the third years or later and fuses with the body at
about 40 years

70) Mr.X fired his gun at Mr.Y who moved and escaped with the bullet only grazing his
thigh. There was only a little bleeding and no other significant injury. Mr. X is liable
for arrest under which section of IPC?
a) 302
b) 304
c) 324
d) 326

C
Section 324 in The Indian Penal Code
324. Voluntarily causing hurt by dangerous weapons or means.Whoever, except in the case provided for by
section 334, voluntarily causes hurt by means of any instrument for shooting, stabbing or cutting, or any
instrument which, used as weapon of offence, is likely to cause death, or by means of fire or any heated
substance, or by means of any poison or any corrosive substance, or by means of any explosive substance or by
means of any substance which it is deleterious to the human body to inhale, to swallow, or to receive into the
blood, or by means of any animal, shall be punished with imprisonment of either description for a term which may
extend to three years, or with fine, or with both.

http://indiankanoon.org/doc/724142/

71) High court has the power to stay the execution of a pregnant woman according to
section
a) 416 Cr. P. C.
b) 417 Cr. P. C.
c) 418 Cr. P. C.
d) 419 Cr. P. C.
A
72) A person was advised by his Orthopedician to get regular dressing of his wound
done. But the patient did not give much care. The patient visited after some time at
which time the Orthopedician again advised regular dressing but dint do the dressing
himself as he said he was busy. Finally the wound enlarged and the underlying bone
developed osteomyelitis. Which statement is true
a) Doctor not guilty under contributory negligence
b) Doctor guilty under Last clear chance doctrine
c)
d)
ANS=B
LAST CLEAR CHANCE: HELPLESS PLAINTIFF
A plaintiff who has negligently subjected himself to a risk of harm from the defendant's subsequent
negligence may recover for harm caused thereby if, immediately preceding the harm,
(a) the plaintiff is unable to avoid it by the exercise of reasonable vigilance and care, and
(b) the defendant is negligent in failing to utilize with reasonable care and competence his then
existing opportunity to avoid the harm, when he
(i) knows of the plaintiff's situation and realizes or has reason to realize the peril involved in it or

(ii) would discover the situation and thus have reason to realize the peril, if he were to exercise the
vigilance which it is then his duty to the plaintiff to exercise.

https://en.wikipedia.org/wiki/Last_clear_chance
Contributory Negligence
Sometimes the unexpected results may not be only due to negligence of the doctor but also due to
negligence of patients or relatives. This is known as contributory negligence. Examples include: (a)
Not coming for follow-up as per the advice of doctor; (b) Failure to follow the instructions given by
the treating doctor; (c) Investigations advised by the doctor are not done by the patient; (d) Patient
fails to take advice of a specialist (for example, in case of acute abdomen or head injury, the
Pediatrician has referred to a surgeon but the patient fails to take such a consultation); and (e)
patient leaves the hospital against medical advice. The liability for the damage in such cases is
suitably divided between the doctor, patients and relatives. The burden of proof of contributory
negligence on the part of patient is on doctors.

http://www.indianpediatrics.net/may2001/may-488-495.htm

73) Rat hole tear is associated with which injury


a) Postmortem artifact
b) Firearm
c) Razor blade
d) ??
B
74) Segmented of blood in blood vessel after death is called
a) Kevorkian sign
b) Rokitansky sign
c) Tache noir
d) ??
A
75) Acrid (pear like) smell is seen in
a) Ether
b) Paraldehyde
c) Phenol
d) ??
B
Welcome to the Sniff a Poison Challenge!
How this works

Below is a list of odo(u)rs. For each different odour try to identify the poison(s) or
toxin(s) that can give rise to it then click on the odour (in blue) to show/hide the
answer. If there are multiple answers, the number of answers listed is shown in
brackets.
Start sniffing!

Ammonia
ammonia (its good to get off to an easy start)

Bitter almonds
cyanide (40% of people are genetically incapable of smelling this)

Burnt rope (2)


marijuana
opium

Disinfectant (2)
phenol
creosote

Fruity (12!)
nitriles
isopropyl alcohol
ketoacidosis
lacquer
ethanol
isopropanol
chloroform
trichloroethane
paraldehyde
chloral hydrate
methylbromide
nitrites (amyl, butyl)

Garlic (7)
phosphorus
tellurium
inorganic arsenicals and arsine gas

organophosphates
selenium
thallium
dimethyl sulfoxide (DMSO)

Fish or raw liver (musty) (3)


zinc phosphide
aluminum phosphide
nickel carbonyl

Hay
phosgene

Mint (2)
methylsalicylate OIL of Wintergreen)
menthol

Mothballs (3)
napthalene
camphor
p-dichlorobenzene

Pear
chloral hydrate

Pepper
o-chlorobenzylidene malonitrile (CS or tear gas)

Pine
pineOIL

Rotten eggs (5)


Hydrogen sulfide
(olfactory fatigue occurs after about 15 minutes at the potentially toxic level of 50
ppm. This happens even faster at higher concentrations so the victim is at least
spared the stench of rotten eggs)
carbon disulfide
mercaptans
disulfiram
N-acetylcysteine (antidote)

Shoe polish
nitrobenzene

Tobacco
nicotine!

Vinegar (2)
acetic acid
hydrofluoric acid

Violets

turpentine (urinary metabolites)


http://lifeinthefastlane.com/toxicology-conundrum-027/

76) A man fell from 35 feet height. Eyewitnesses say that he landed on his feet. Which
of the following injuries is possible
a) Ring fracture of Foramen magnum + Lumbar spine injury
b) Pond fracture skull + Cervical spine injury
c) Gutter fracture skull + Cervical spine injury
d) Depressed fracture skull + Cervical spine injury
A
LUMBOSACCRAL SPINE IS MORE PRONE TO FRACTURES AND COMPRESSION INJURIES
Fracture around foramen magnum (Ring Fracture) : This is a type of fissured fracture which
encircles the base of skull around the foremen magnum running 3 5 cm outside foramen magnum
at the back and sides of the skull. Such fractures are seen in following cases : (A) Fall from height
where a person falls on feet or buttock and impact passes upward through spinal column. (B) Fall
from height where head strikes the ground first. (C) Fall of heavy load on head. (D) Violent twisting
of head.

77) The knot in judicial hanging is placed at


a) The back of the neck
b) The angle of jaw
c) Below the chin
d) Choice of hangman
B
Judicial hanging: The knot is under the angle of jaw in Indian legal death sentence.
------------------------------------------------------------------------------------------------------------------

Hanging is one of the oldest methods for capital punishment in most countries with
India being no exception. The procedure of execution has largely remained unchanged
over the centuries. Some modifications to its process were introduced in the late 19th
century to make it more humane, quick and painless, and without decapitation. But
still death in most cases were due to strangulation causing delayed deaths and not the
intended fracture of cervical column which would cause instantaneous death. After
autopsy examination of many cases of judicial hanging, researchers proposed that for
instant death, the position of the knot should be submental and the drop length
should be based upon the weight of the condemned person. The other concern is the
involvement of doctors in the process of execution which is considered against the
Hippocratic Oath. Over the years, doctors have helped the state to devise newer
methods of execution. They are actively involved in preparing the condemned person,
checking on their health with the purpose of supervising their death, prescription of
the lethal cocktails, putting in of intravenous lines, pronunciation of death, and some
are also involved in removal of their organs. The Medical Council of India should
strongly oppose to doctors taking part in execution process. Ethical practice
guidelines should be framed in lines similar to American Medical Association. Actions
which are at the most acceptable are providing sedative to calm anxiety at the
prisoners request and certification of death after another person had pronounced it
http://www.jaypeedigital.com/Chapter/ChapterDetail/67547

78) A bomb blast took place in Delhi following which 2 persons died. All of the
following are true about their injuries except
a) Injuries due to ... burns or air blast
b) Force of explosion is directional
c) Force of explosion decreases rapidly.
d) Bruise, laceration ,fractures are triad of main injuries seen
ans=??
A CONTROVERSIAL QUESTION
Explosion is a sudden release of previously confined energy characterized by release of
large volume of gas with high pressure, heat and noise. Explosion may be from two
sources, improvised explosion devices or fuel-air explosion. Four workers were working
at a chemical reactor in a pharmaceutical factory. On 5th January, 2013 explosion of
chemical reactor took place in pharmaceutical factory at Nakkapalli, two succumbed to
blast injuries on spot and two other succumbed to burns in hospital. Explosive forces
are directional and decline rapidly.

http://www.jebmh.com/latest-articles.php?at_id=884

Emergency Preparedness 18:


Explosion and Blast Injuries Portal
In the past, explosive and blast injuries have occurred during times of organized military conflict or have
resulted from non-intentional events, such as industrial accidents. More recently, explosive devices have
become a preferred weapon of domestic and foreign terrorists since they are inexpensive to produce,
capable of causing large numbers of casualties, and effective at instilling fear in large populations.1
Consequently, blast injuries are being observed in non-combat locations during peace time as evidenced
in the Alfred P. Murrah Federal Building in Oklahoma City in 1995, the United States Embassy in Lagos,
Nigeria, Africa in 1998, and the 9/11 attacks on the WorldTRADE Center and Pentagon in 2001.
Whenever medical personnel are called upon to treat patients with explosive or blast injuries, conditions
will obviously be difficult or austere. Thus it is important for the health provider team to approach these
situations in an organized fashion, cognizant of the teams resources as well as the potential for injuries
unique to blast situations.
General Treatment Considerations in Explosion and Blast Injuries
In non-war or non-combat situations, for an explosion or blast to occur other than when unanticipated or
by surprise is rare. The health care team needs a general plan outlining an approach to these
disasters. (Vol IIIEMP1 Community-Wide Collaboration)While the organized implementation of
this disaster plan is of great value in any large emergency, some considerations are unique to
implementing the plan during explosion or blast scenarios. These include:
1. What available resources are at your disposal?
o What available resourcesincluding the number of trained medical personnel, support
staff, and equipmentcan be mobilized in your health care institution?
o What community resources can be mobilized to assist with the care of the disaster
victims? How will these be obtained?
o What resources can be obtained from outside the community? How will these facilities be
notified of your needs?
o What state or national agenciessuch as the State Department of Health, Federal
Emergency Management Agency (FEMA), and American Red Crossneed to be
contacted? Who will contact these agencies?

2. How will you triage the patients? Large numbers of victims in a disaster situation can overwhelm
almost any medical system. The use of an effective triage system helps to assure that limited
resources are utilized to do the greatest good for the greatest number of people. One triage
method to consider is a modified START Triage System,5 which is based on classifying patients
into one of four categories. (See Vol III EMP9, Triage.) These categories include patients
who:
o Need immediate care to survivePriority 1 (P-1).
o Need care but can have the care delayedPriority 2 (P-2).
o Do not appear to have any serious injury but need to be held for later evaluationPriority
3 (P-3).
o Will most likely die no matter how you assist themPriority 0 (P-0).
3. What special problems may be associated with the blast/explosion? These may include
additional, non-visible, risks such as:
o Biological agent contamination
o Exposure to inhalation of toxic agents
o Nuclear contamination

Repeat terrorist assault

4. What injuries unique to blast situations may your patient have? Explosions can produce unique
patterns of injury seldom seen outside of combat. These injuries can involve multiple organ
systems, especially the lungs, ears, bowel (gastrointestinal [GI] tract), and central nervous system
(CNS).
5. How do you manage the wide range of routine traumatic injuries inflicted on your patients in
addition to all of the injuries unique to blasts?
6. How do you manage the post-traumatic stress disorder (PTSD) in the victims of the blast, their
friends and relatives, and treating medical staff?
Classification of Explosives2
One method of classifying non-nuclear explosives is to divide them as either high-order explosives (HE)
or low-order explosives (LE). Examples of HE include TNT, ammonium nitrate fuelOIL , dynamite, and
nitroglycerine. Examples of LE include gunpowder, pipe bombs, and pure petroleum-based bombs like
Molotov cocktail or aircraft used as guided missiles. HE and LE create different injury patterns, primarily
because HE produces a supersonic over-pressurization shock wave that causes unique injuries.
The HE shock wave is created at detonation by the rapid chemical decomposition of the explosive
material into gas.1 This gas is created under high pressure and temperature and is transmitted as the
blast (or shock) wave. This pressure-pulse is only a few millimeters thick and travels outward from the
point of detonation at supersonic speeds. The blast (or shock) wave may have focal over-pressure up to
eight times atmospheric pressure. This sudden change in environmental pressure (blast-wave) impacting
with the body causes the primary blast injuries unique to HE. The blast injuries are especially evident in
the gas-filled structures on the body, including the lung, GI tract, and middle ear.
As the blast front travels from the explosion, it decreases in pressure until the blast-wave is
reduced into an acoustic wave.1This creates a blast-wind, which can propel objects and people
considerable distances, resulting in additional damage to life and objects.
Basic Categories of Blast Injuries2
1. Primary blast injuries2result from the impact of the blast-wave with the body. The effects are
most evident in gas-filled structures on the body where the over-pressurized blast-wave causes
rapid compression and expansion of the gas-filled structures resulting in tissue damage,
hemorrhage, or organ rupture. (See below for further information about specific organ damage.)
2. Secondary blast injuries2are due to objects (including bomb fragments and flying debris)
becoming energized by the blast, resulting in their acting as projectiles. These flying objectprojectiles can affect any part of the body with either penetrating or blunt trauma.
3. Tertiary blast injuries2result from individuals being thrown by the blast-wind or injuries caused
by the collapse of structures due to an explosion. Any part of the body can be affected. Common
injuries include fractures, amputations, and/or brain injuries.
4. Quaternary (miscellaneous) blast injuries2This group includes the other explosion-related
injuries, illnesses, or diseases not due to one of the three mechanisms just listed. These include
burns and the exacerbation or complications of existing conditions such as asthma; chronic
obstructive pulmonary disease (COPD); breathing problems from dust, smoke, or toxic fumes;
and the patients development of angina, hyperglycemia, etc.
Specific Blast Injuries
1. Primary Blast Injuries to the Lung (Blast-Lung)2Blast-lung is a direct consequence of an HE
over-pressurized shock wave impacting the body. The signs and symptoms of blast-lung are
usually present at the time of initial presentation to the health care facility but may not become

evident for up to 48 hours after the victim is exposed to the explosion. The characteristic clinical
triad of severe blast-lung is apnea, bradycardia, and hypotension; but all patients who complain of
dyspnea, cough, hemoptysis, or chest pain after an explosion exposure should be evaluated for
blast-lung. Lung pathology ranges from simple scattered pulmonary petechiae to confluent lung
hemorrhages resulting in severe hypoxia and respiratory failure. Primary blast-lung is the most
common cause of death in patients who initially survive an explosion. Management of patients
suspected of having blast-lung includes obtaining a routine chest x-ray. The characteristic x-ray
finding of significant blast-lung is a butterfly pattern due to pulmonary hemorrhage, but this
pattern may not be apparent at the time of initial evaluation. Patients with severe blast-lung may
require intubation and respiratory support. Prophylactic chest tubes are recommended for
patients suffering from an explosion-generated blast-lung who need general anesthesia or air
transport to prevent the patient from developing a post-blast tension pneumothorax during
surgery or flight.
2. Ear Injury from Blast2 Ear injuries are a common result of explosion exposure, especially in the
ear oriented toward the blast. While a perforated TM is the most common manifestation of a
blast-ear injury, not infrequently there are no significant early complaints or physical findings at
the time of initial evaluation. Ear signs and/or symptoms such as hearing loss, tinnitus, otalgia,
vertigo, bleeding from the ear canal, or otorrhea should raise the providers suspicion of ear
injury. All patients exposed to a blast should have an otological evaluation and audiogram after
the more serious injuries have been treated.
3. Abdominal Injuries from Blast (Blast-Abdomen)2 Gas-containing structures of the GI tract are
especially vulnerable to the primary blast effects of a blast-wave. Injuries include:
o Immediate or late bowel perforation
o Hemorrhageranging from small petechiae to large hematomas
o Mesenteric shear injuries
o Solid-organ lacerations
o Testicular rupture
Symptoms that may suggest a blast injury to the abdomen in a blast victim include:
o Abdominal pain
o Nausea and vomiting
o Hemetemesis
o Rectal pain
o Tenesmus
o Testicular pain
o Unexplained hypovolemia
o Evidence for an acute abdomen
Clinical findings of a blast-abdomen may be delayed from hours to days.
4. Central Nervous System Injuries from a Blast2 Brain injury from a blast can occur without a
direct blow to the head. The primary blast (shock) wave can cause varying central nervous
system (CNS) damage from a mild concussion (often referred to as mild traumatic brain injury
[MTBI]see below) to severe brain injuries. Blast CNS injuries may include:
o Concussion
o Closed or open brain injury
o Stroke syndrome
o Spinal cord injury
o Air-embolism induced CNS injury

MTBI3 refers to a syndrome caused by a seemingly mild brain injury or concussion that usually presents
with minimal or non-observable symptoms but which may have significant long-term sequelae. Long-term
signs and symptoms of patients experiencing MTBI may include3:
Cognitive changes: manifested as attention difficulties, memory problems, difficulty with
concentration and orientation.
Physical complaints: headaches, dizziness, insomnia, fatigue, nausea, blurred vision, or ataxia.
Behavior changes: including irritability, depression, anxiety, sleep disturbance, emotional lability,
loss of initiative, and relationship problems.
A diagnosis of MTBI may be difficult to make since the symptoms are non- specific and are similar to
many other medical conditions including PTSD. Consider MTBI in patients with the above symptoms and
a history of head trauma or blast exposure, especially if they have amnesia of the event, had loss of
consciousness for < 30 minutes, and had a GCS score > 13.
5. Eye Injuries from Blast2 As many as 10% of all blast survivors have a significant eye injury. The
most common eye injuries are globe perforations from high-velocity projectiles, foreign bodies on
the cornea or in the globe, air emboli, or orbital rim fractures. For a patient to have minimal initial
discomfort and not present for care of the eye injury for days, weeks, or even months after the
event is not unusual.
Symptoms that may suggest eye injury include:
Eye pain
Eye irritation
Foreign body sensation
Altered vision
Findings that suggest an eye injury include:
Periorbital swelling or contusions
Decreased visual acuity
Hyphema
Globe perforation
Subconjunctival hemorrhage
Eye foreign body
Lid laceration
6. Musculoskeletal and Other Soft Tissue Injuries from Blasts1 Musculoskeletal injuries from
blasts result from the combination of forces generated by a blast. The abrupt rise in atmospheric
pressure created by the passing blast-wave causes many fractures and contributes to limb
avulsion. The mechanism of explosive blast amputation is frequently the combination of the
coaxial forces of the blast-wave inducing the fracture followed by limb avulsion through the
fracture site by the dynamic forces of the blast-wind acting on the whole limb. Blast-induced lower
limb amputations are most common at the level of the tibial tuberosity, while the upper limb
amputations tend to be more distal in the limb. Other soft tissue injury is also common. The debris
and casing fragments energized by the blast are propelled with a tumbling and shimmy effect that
increases the amount of tissue damage upon impact with a victim. In addition, the debris is very
contaminated with environmental bacteria resulting in extensive wound contamination.

7. Other Special Considerations During Evaluation of Blast Victims1


o
o
o

Fragments, energized by a blast, do not travel in straight lines after they enter the body.
Small entry wounds can be associated with extensive internal injury.
Entry wounds in the buttocks, thighs, or perineum can be associated with intra-abdominal
injuries.

Health care providers need to maintain a high degree of suspicion that a compartment
syndrome may develop in injured extremities.
o An entry wound in the groin or the development of a hematoma elsewhere may mean
that a major vascular injury has occurred.
Special Considerations in the Care of Patients Manifesting Blast Injuries
The initial approach to patients who have experienced injuries from an explosion or blast follows the
CALS Universal Approach for Critical Patient Care with coordination of the teams efforts to be as efficient
and effective as possible. Some special treatment considerations include:
The potential risk that the victim may have been contaminated with a biological, chemical, or
nuclear agent.
The unique syndromes associated with blast exposure including the development of blast-lung or
blast-abdomen.
When the number of victims is large compared to the number of providers, how to triage patients
to maximize the good performed by the treatment team.
Blast wounds are contaminated with bacteria and debris and are associated with a high risk of
infection.1 Meticulous wound care is essential, including the removal of non-metallic foreign
material, excision of non-viable tissue, and irrigation of wounds with copious amounts of solution
(isotonic if possible) to help remove bacteria and foreign material.
Prophylactic antibiotics should be considered.1
1. Gas gangrene, by anaerobic Clostridium species, is a major threat with blast injuries.
Penicillin IV helps reduce the risk. Alternative antibiotics include erythromycin,
chloramphenicol, or a cephalosporin.
2. In severely contaminated blast injuries, Pseudomonas aeruginosa may be a problem.
Consider giving an aminoglycoside IV or a different drug effective against Pseudomonas.
o

3. Open fractures need coverage against gram-positive organisms. A penicillinase-resistant


penicillin or a cephalosporin IV is appropriate.
4. Tetanus is a high risk. Give the patient tetanus prophylaxis and consider the need for
anti-tetanus immunoglobulin.
Post-Traumatic Stress Disorder4 The development of PTSD is a risk following any traumatic
event associated with a sense of horror or helplessness, the development of serious injury, or the
threat of serious injury or death. It may manifest not only in survivors who have directly
experienced the traumatic event but also in the relatives and/or close friends of the victims as well
as in rescue workers. Frequently, symptoms associated with the PTSD do not become apparent
for weeks or months following a traumatic event. The symptoms characteristic of PTSD are
similar to those observed with patients suffering with the late sequelae of MTBI and include:
1. Cognitive effects such as poor concentration, memory loss, shortened attention span,
or indecisiveness.
2. Emotional manifestations including depression, numbness, feeling of being
overwhelmed, or volatile emotions.
3. Physical complaints of nausea, dizziness, rapid heart rate, headache, tremors, or
undefined pain.
4. Behavior changes including irritability, excessive silence, suspicion, argumentum, or
withdrawn.
The care of patients with injuries as a result of a blast or explosion exemplifies the critical need for the
provider team to have a preplanned approach to major disasters. This includes having a well developed
disaster plan utilizing all of the available resources as efficiently as possible; a triage system designed to
fit your facility; the proper basic equipment to handle critical, life-threatening, emergencies; and a system

built to maximize the cooperation of the health care team. With proper planning, major disasters can be
managed to the benefit of the great majority of the victims with a minimum of risk to the provider team.
https://calsprogram.org/manual/volume3/Section13/19-EMP18Expl&BlastInjuries13.html

Introduction:
An explosion is a process where any
substance or device capable of creating a
sudden gas expansion, releasing potential
energy and thus creating a pressure wave.
Compression of the air in front of the
pressure wave, which heats and accelerates air
molecules, leads to sudden increase in
atmospheric pressure and temperature
transmitted to the surrounding environment as a
radially propagating shock wave, known as the
Blast wave. [1]
Injuries directly inflicted by this sudden
increase in air pressure after an explosion are
referred to as primary blast injuries, and mainly
affect primary gas containing structures (lungs,
middle ears and gastrointestinal tract). [1-3]
A minimum pressure of about 700 kPa
(100 lbs/sq inch) is necessary for severe tissue
damage in humans. [4
Secondary blast injuries result from
blast-energised bomb fragments and other
displaced objects causing penetrating trauma.
Tertiary blast injuries occur when the body is
accelerated away from the blast wave at first
and is then abruptly decelerated on rigid objects
resulting in blunt force trauma. [1]
Quaternary injuries (Miscellaneous blast
related injuries) encompass injuries caused by
collisions, falling masonry, buildings, beams, etc.
[5] Explosive force is highly directional. The parts
of the body directly exposed to the explosive
force only are injured. An explosion at ground
level mainly injures legs and feet.
When a person is in front of a bomb
when it blows up, the face, chest, forearms,
hands, inner thighs and legs below knees are
injured, but the back of the body, the lower legs
and face escape. If a person is bending down
over the bomb, the face, chest, legs and hands
are severely damaged. [4]
Orthopaedic trauma resulting from an
explosion is manifested as a primary, secondary,
tertiary or quaternary (miscellaneous) blast injury
in isolation or in combination.
Although uncommon in survivors, the
direct effects of changes in atmospheric
pressure caused by the blast wave (primary
blast injury) can fracture bones and it is probably
responsible for limb avulsions in victims exposed
to stress waves of sufficiently high intensity

http://medind.nic.in/jal/t15/i1/jalt15i1p85.pdf

79) After an incised wound, new collagen fibrils are seen along with growing
epithelium. The age of the wound is
a) 4-5 days
b) About 1 week
c) 12-24 hours
d) 24 -72 hours
A

WOUND HEALING
Healing of skin wounds provides a classical example of
combination of regeneration and repair described above.
This can be accomplished in one of the following two ways:
Healing by first intention (primary union): and
Healing by second intention (Secondary union)
Healing by First Intention (Primary Union)
This is defined as healing of a wound which has the following
characteristics:
i) Clean and uninfected;
ii) Surgically incised;
iii) Without much loss of cells and tissue; and
iv) Edges of wound are approximated by surgical sutures
Page 2
Page 3
WOUND HEALING
The sequence of events in primary union are described below:
1. Initial haemorrhage: Immediately after injury, the space
between the approximated surfaces of incised wound is filled
with blood which then clots and seals the wound against
dehydration and infection.
2. Acute inflammatory response: This occurs within 24 hours
with appearance of polymorphs from the margins of incision.
By 3rd day, polymorphs are replaced by macrophages
3. Epithelial changes: The basal cells of epidermis from both
the cut margins start proliferating and migrating towards

incisional space in the form of epithelial spurs. A wellapproximated wound is covered by a layer of epithelium in 48
hours. The migrated epidermal cells separate the underlying
viable dermis from the overlying necrotic material and clot,
forming scab which is cast off. The basal cells from the
margins continue to divide. By 5th day, a multilayered new
epidermis is formed which is differentiated into superficial and
deeper layers.
Page 4
WOUND HEALING
4. Organization: By 3rd day, fibroblasts also invade the wound
area. By 5th day, new collagen fibrils start forming which
dominate till healing is completed. In 4 weeks, the scar tissue
with scanty cellular and vascular elements, a few inflammatory
cells and epithelialised surface is formed.
5. Suture tracks: Each suture track is a separate wound and
incites the same phenomena as in healing of the primary wound
i.e. filling the space with haemorrhage, some inflammatory cell
reaction, epithelial cell proliferation along the suture track from
both margins, fibroblastic proliferation and formation of young
collagen. When sutures are removed around 7th day, much of
epithelialised suture track is avulsed and the remaining epithelial
tissue in the track is absorbed. However, sometimes the suture
track gets infected (stitch abscess) or the epithelial cells may
persist in the track (implantation or epidermal cysts).
Thus, the scar formed in a sutured wound is neat due to close
apposition of the margins of wound; the use of adhesive tapes
avoids removal of stitches and its complications.
http://faculty.ksu.edu.sa/8210/Documents/Inflammation.Lecture9.pdf
80) Sweating is absent in
a) Heat stroke
b) Heat syncope
c) Heat exhaustion
d) Miners cramps
A
81) According to the 2013 amendment the age for sexual consent is
a) 16 years

b) 18 years
c) 20 years
d) 15 years
B
ans-B
375. A man is said to commit "rape" if he Rape- (a) penetrates his penis, to any
extent, into the vagina, mouth, urethra oranus of a woman or makes her to do so with
him or any other person; or (b) inserts, to any extent, any object or a part of the
body, not being the penis, into the vagina, the urethra or anus of a woman or makes
her to do so withhim or any other person; or (c) manipulates any part of the body of a
woman so as to cause penetrationinto the vagina, urethra, anus or any part of body of
such woman or makes her todo so with him or any other person; or (d) applies his
mouth to the vagina, anus, urethra of a woman or makes her to do so with him or any
other person, under the circumstances falling under any of the following seven
descriptions: First. Against her will. SecondlyWithout her consent. ThirdIyWith
her consent, when her consent has been obtained byputting her or any person in
whom she is interested, in fear of death or of hurt. F ourthlyWith her consent, when
the man knows that he is not herhusband and that her consent is given because she
believes that he is anotherman to whom she is or believes herself to be lawfully
married. Fihly.With her consent when, at the time of giving such consent, by
reason of unsoundness of mind or intoxication or the administration by him personally
or through another of any stupefying or unwholesome substance,she is unable to
understand the nature and consequences of that to which shegives consent
*****SixthlyWith or without her consent, when she is under eighteen years of
age.****** Seventhly.--When she is unable to communicate consent. Explanation I.For the purposes of this section, "vagina" shall also includelabia majora. Explanation
2.Consent means an unequivocal voluntary agreement whenthe woman by words,
gestures or any form of verbal or non-verbal communication,communicates
willingness to participate in the specic sexual act
http://indiacode.nic.in/acts-in-pdf/132013.pdf
82) HPV can cause all of these cancers except
a) Base of tongue carcinoma
b) Tonsillar carcinoma
c) Nasopharyngeal carcinoma
d) Ca Cervix
C
What head and neck cancers can be caused by HPV?
Ads by RainbowPricesAd Options
HPV can cause cancers in the back of the throat, most commonly in the base of the

tongue and tonsils, in an area known as the oropharynx. These cancers are called
oropharyngeal cancers.
How does HPV cause cancer?
HPV can cause normal cells in infected skin to turn abnormal. Most of the time, you
cannot see or feel these cell changes. In most cases, the body fights off the HPV
infection naturally and infected cells then go back to normal. But in cases when the
body does not fight off this virus, HPV can cause visible changes and certain types of
HPV can cause an oropharyngeal cancer. Cancer caused by HPV often takes years to
develop after initially getting an HPV infection. It is unclear if having HPV alone is
sufficient to cause oropharyngeal cancers, or if other factors (such as smoking or
chewing tobacco) interact with HPV to cause these cancers. More research is needed
to understand all the factors leading to oropharyngeal cancers.
http://www.cancer.med.umich.edu/news/nasopharyngeal-cancer09.shtml
What head and neck cancers can be caused by HPV?
Ads by RainbowPricesAd Options
HPV can cause cancers in the back of the throat, most commonly in the base of the
tongue and tonsils, in an area known as the oropharynx. These cancers are called
oropharyngeal cancers.
How does HPV cause cancer?
HPV can cause normal cells in infected skin to turn abnormal. Most of the time, you
cannot see or feel these cell changes. In most cases, the body fights off the HPV
infection naturally and infected cells then go back to normal. But in cases when the
body does not fight off this virus, HPV can cause visible changes and certain types of
HPV can cause an oropharyngeal cancer. Cancer caused by HPV often takes years to
develop after initially getting an HPV infection. It is unclear if having HPV alone is
sufficient to cause oropharyngeal cancers, or if other factors (such as smoking or
chewing tobacco) interact with HPV to cause these cancers. More research is needed
to understand all the factors leading to oropharyngeal cancers.
http://www.cdc.gov/std/hpv/stdfact-hpvandoralcancer.htm

83) Kashima operation done for


a) Vocal cord
b) Recurrent Cholesteatoma
c) Atrophic rhinitis
d) ??
A
This video clipping shows coablation technology being used to perform kashimas operation. This
surgery is performed to manage patients with bilateral abductor nerve paralysis of vocal folds.

https://youtu.be/XK_D7oLd28g

84) The main vessel involved in bleeding from JNA


a) Internal maxillary A.
b) Ascending pharyngeal A.
c) ??
A
85) Eustachian tube function is best assessed by
a) Politzer test
b) VEMP
c) Rhinomanometry
d) Tympanometry
A
ans=d>a?
politzers Test Politzers test is performed by compressing one naris into whichthe end
of a rubber tube attached to an air bag has been insert-ed while compressing the
opposite naris with finger pressure.The subject is asked to repeat the letter K or is
asked to swallowto close the velopharyngeal port (Figure 813). When the testresult is
positive, the overpressure that develops in thenasopharynx is transmitted to the
middle ear, thus creating pos-itive middle-ear pressure. Assessment of the middle-ear
pressureand the significance of the test results are the same as withValsalvas test in
that a normal result indicates only tubal paten-cy. However, both Valsalvas and
Politzers methods can be ofbenefit as a treatment when effusion or high negative
pressure ispresent within the middle ear if the patient can successfullyinflate the
middle ear. Valsalvas and Politzers maneuvers maybe more beneficial as
management options in selected patientsthan they are as methods to assess tubal
function, althoughthere is controversy about the efficacy of these procedures
fortreatment of middle-ear effusion (see Chapter 9, Role inManagement of Otitis
Media).
----------------------------------------Tests of Pressure Regulation Function When theTympanic Membrane Is Intact
Eustachian tube function in individuals with intact tympanicmembranes may be
determined by manometry, tympanometry or sonotubometry. A pressure chamber may
or
may
not
be
nec-essary
for
testing.
Tympanometry Determination of middle-ear pressure and acoustic immittanceusing
electroacoustic impedance equipment were introduced byMetz about 50 years ago.51
These same techniques have beenused to perform tympanometry, which is the
measurement ofthe acoustic driving-point immittance as a function of the stat-ic
pressure in the canal. If low-frequency tones are used for themeasurement, the static
pressure that produces the maximalacoustic immittance is approximately equal to the

gas pressurein the middle ear.Tympanometry in a Pressure Chamber Thomsen adapted


theacoustic impedance method for use in a pressure chamber.43 Hevaried the
chamber pressure and measured the percentage ofabsorption of a tone presented into
the ear canal. He found thatthere was a fall in absorption as the pressure difference
betweenthe middle ear and the chamber was increased. The absorptionreached a
peak when the two pressures were identical.Unfortunately, Thomsens technique
failed to account for thechange in middle-ear pressure caused by the measurement
pro-cedure. As the pressure in the chamber is varied (in search ofmaximal loudness or
absorption), the tympanic membranemoves from its original position to a new
position, thus chang-ing the volume of the middle-ear cavity. However, according
toBoyles law, as the volume of the cavity changes, the pressuremust also change.
Thus, by knowing the volume displacementand measuring the final pressure, the
original pressure can bededuced.Bylander used tympanometry with a pressure
chamber toevaluate Eustachian tube function in normal children.23 In thismethod,
the resting middle-ear pressure is obtained from theinitial tympanogram. Then the
chamber pressure is lowered to100 mm H2O relative to ambient pressure, and a
second tym-panogram is obtained, verifying the relative overpressure in themiddle
ear. After this deglutition of the subject, a tympanogramis recorded to determine
middle-ear pressure. The same proce-dure is repeated with 100 mm H2O relative
overpressure in thechamber to assess the subjects ability to actively equilibrate relative underpressure in the middle ear. With use of this method,the inflation-deflation
test was conducted on 50 children, andthe results were compared with the results of
tests that measuredtubal function in adults. In this way, the first database for
tubalfunction in otologically normal children was established.Shupak and colleagues
also used tympanometry inside apressure chamber to assess the ability of naval scuba
divers toequilibrate negative middle-ear pressure.5

86) Topical trt for recurrent respiratory papillomatosis includes


a) Acyclovir
b) Cidofovir
c) Ranitidine
d) Zinc
B
Recurrent respiratory papillomatosis (RRP) is a rare but potentially severe disease caused by papillomavirus, most
often types 6 and 11. The disease, which occurs in both juvenile and adult forms, is characterized by benign
epithelial tumors of the airway that most frequently affect the larynx but can also spread along the entire
aerodigestive tract. Recurrent respiratory papillomatosis is the most common benign neoplasm of the larynx in
children and the second most frequent cause of childhood hoarseness. Standard treatment, which is palliative only,
consists of surgical excision of papillomata to maintain airway patency and improve voice quality. Recurrence
despite repeated surgical procedures is the rule. To date, incorporation of adjuvant treatments has not been
reliably beneficial in altering the disease course. Several case series have described promising results with
cidofovir, a cytosine nucleoside analog with antiviral activity. To evaluate the data available on the safety and

efficacy of cidofovir for the treatment of RRP, we conducted a MEDLINE search for all case reports or series from
January 1966-August 2004 describing cidofovir therapy in either adults or children with RRP. The bibliographies of
qualifying articles were also searched for relevant references. In both adults and children with mild-to-severe RRP,
intralesional administration of cidofovir directly into the site of papillomata was associated with partial-tocomplete regression of papillomata, improvement in voice quality and airway status, and decreased need for
surgery. Wide variation in intralesional cidofovir dose (2-57 mg), frequency (every 2-8 wks), and duration (4 mo-4
yrs) was found. Successful outcomes have also been reported with intravenous cidofovir, but data are limited to
three case reports. Rash, headache, and precordialgia were the only adverse effects reported with intralesional
cidofovir. Nephrotoxicity and neutropenia secondary to either intralesional or intravenous cidofovir were not
observed. Long-term risks associated with intralesional administration remain to be seen. Further studies are
necessary to determine the most appropriate dose, frequency, and duration of therapy, and to fully characterize
the safety profile of cidofovir when given intralesionally.
Recurrent respiratory papillomatosis (RRP) is a rare disease caused by human papillomavirus (HPV), most
commonly types 6 and 11.[1, 2] Papillomavirus is a small, nonenveloped virus consisting of an icosahedral capsid
enclosing a double-stranded, circular DNA genome. At least 70 HPV types have been identified, most of which are
associated with epithelial tumors of the skin and mucous membranes, such as plantar warts, condylomata
acuminata (anogenital warts), and epidermodysplasia verruciformis.[3, 4] Recurrent respiratory papillomatosis is
manifested as exophytic lesions that most frequently affect the larynx but can also spread along the entire
aerodigestive tract. The presence of these benign neoplasms can cause symptoms ranging from dysphonia to lifethreatening respiratory distress, and can profoundly affect the quality of life of patients with RRP.[5, 6]
Traditionally, RRP has been classified based on patient age at diagnosis, such that two forms of the disease have
been described: one with onset in childhood, which is arbitrarily defined as younger than 12 years (juvenile-onset
RRP), the other in adulthood (adult-onset RRP). Juvenile-onset RRP is observed more often and is typically more
aggressive than its adult counterpart. Peak age for the juvenile form is around 4 years, compared with the third
decade of life for the adult form.[7, 8] Among children, RRP is the most common benign neoplasm of the larynx
and the second most frequent cause of hoarseness.[9]

http://www.medscape.com/viewarticle/507662

87) The MC mode of spread of retinoblastoma


a) Lymphatics
b) Optic nerve
c) Direct invasion
d) Vascular
B
88) Evisceration of eye is not done in
a) Malignancy
b) Panophthalmitis
c) Severe globe trauma
d) Expulsive Hemorrhage

A
89) Multifocal ERG is very useful because
a) Can assess rods
b) Can assess macular cones
c) Can assess function of ganglion layer
d) ??
B
Electro Diagnostics
1. Full field Electroretinography (ERG): Test which measures the electrical responses
of various cell types in the retina, including the photoreceptors (rods and cones),
inner retinal cells (bipolar and amacrine cells). Used to diagnose various retinal
degenerations.
Retinitis pigmentosa and their variants
X-linked juvenile retinoschisis
Heredo-macular degenerations
Retinal Vascular occlusions
Intraocular Foreign Body
***2. Multifocal Electroretinography (mfERG): Multifocal electroretinography (mfERG)
is a valuable technique in assessing macular function in retinal disease objectively. It
is used to record separate responses for different retinal locations.**** It is also used
in the detection of
Macular dystrophies
Macular hole
X-linked retinoschisis
Drug toxicity
Multifocal choroditis
White-dot syndrome
3. Electrooculogram (EOG): This is used to assess the function of the outer retina and
Retinal Pigment Epithelium (RPE). EOG is used to confirm
Best disease
Suspected drug toxicities
4. Pattern Electroretinogram (PERG): This test provides information about central
macular and retinal ganglion cell layer. It is also used to differentiate vision loss due
to retinal or optic nerve diseases. PERG is used in evaluating
Glaucoma and ocular hypertension,
Optic neuritis other optic neuropathies,
Maculopathies
5. Visually Evoked Responses (VEP): The VEP is a test to detect problems with the
optic nerve and lesions in the anterior part of our visual pathway. VEP tests are used
to evaluate

Optic neuritis,
Compressive Optic neuropathy,
Toxic amblyopia
Cortical Blindness
Demyelenating diseases such as multiple sclerosis.
Unexplained visual loss
6. Multifocal Visually Evoked Responses (mfVEP): This test allows for topographical
assessment of visual field. In this test multiple individual VEP responses are recorded
simultaneously from 60 or so regions of the central 20 to 25 radius of the visual field.
This is also known as objective visual field perimetry. Indications are
Diagnosing and Following of Optic Neuritis/Multiple Sclerosis
Unexplained visual loss
Detecting and following of Glaucomatous damage
Confirming unreliable or questionable fields
http://www.sankaranethralaya.org/patient-care-electra-diagnostics.html
90) When compared to blood, Vitreous humor has high concentration of
a) Glucose
b) Sodium
c) Potassium
d) Ascorbate
D
Vitreous humor contains ascorbic acid at a higher concentration than blood.[1] Different species of animals have
different concentrations of ascorbic acid in vitreous humor.[2] During development of the eye. the ascorbic acid
content of the vitreous in fetuses was found to increase gradually until the adult level was reached prior to birth.

[3] We report the same to be true in human fetal eye


http://www.ijo.in/article.asp?issn=03014738;year=1983;volume=31;issue=2;spage=73;epage=74;aulast=Sen

91) Sensory supply of cornea is by


a) Infratrochlear N.
b) Infraorbital N.
c) Naso lacrimal N
d) Supraorbital N.
A
ans=A
Nerve supply of Cornea:
Cornea is one of the highly sensitive tissue of human body. Density of the nerve ending in cornea is about 300 times
of that of skin. An area of 0.01 mm2 cornea may contain as many as 100 nerve endings. Cornea is primarily
innervated through the ophthalmic branch of the trigeminal nerve. The ophthalmic division of the trigeminal nerve

has three parts: the frontal nerve, the lacrimal nerve, and the nasociliary nerve. The nasociliary nerve provides
sensory innervation to the cornea.
infratrochlear nerve infratrochlear nerve (n'fr-trk'l-r)
n.
A branch of the nasociliary nerve, supplying the skin of the eyelids and the root of the nose.
The American Heritage Stedman's Medical Dictionary
Copyright 2002, 2001, 1995 by Houghton Mifflin Company. Published by Houghton Mifflin Company.

http://www.eophtha.com/eophtha/anatomy/anatomyofcornea3.html
92) Universal marker of limbal stem cells
A. Elastin
B. Keratin
C. Collagen
D.ABCG2
D
Putative positive and negative LESC markers
The literature reflects many attempts to prospectively identify LESC using a specific marker. As yet no single,
reliable marker has been found. However, the expression of a combination of several features seems to allow for
greater specificity.
Putative markers can either be positive (present) or negative (absent). Limbal basal cells lack differentiation
markers such as the 64 kDa cytokeratin 3 (CK3) that is present in all other layers of the corneal epithelium and the
suprabasal layers of the limbal epithelium (Schermer et al., 1986). The corneal specific 55 kD protein, cytokeratin
12 (CK12) is also expressed in a similar pattern (Chaloin-Dufau et al., 1990). Furthermore, connexin 43 (Shortt et
al., 2007a; Matic et al., 1997) and involucrin (Chen et al., 2004), both markers of cells destined for differentiation,
are also absent.
The transcription factor p63 is required for formation of epidermis and has been proposed as a putative positive
LESC marker (Pellegrini et al., 2001). In vitro,p63 was found to be expressed in limbal epithelial cell derived
holoclones with little or no expression in meroclones and paraclones. In vivo, p63 was located in the limbal basal
epithelium. However, since these initial observations a number of reports have suggested that p63 is not
sufficiently specific to act as an LESC marker as it has also been localised to basal cells of the peripheral and
central cornea in humans (Chen et al., 2004; Dua et al., 2003) and in rats (Chee et al., 2006). However, limbal
epithelial cells expressing high levels of p63 with a high nuclear to cytoplasmic ratio appear to be more stem like
(Arpitha et al., 2005). Further work has since indicated that the Np63 isoform may more specifically label LESC
(Di Iorio et al., 2005).
Many types of organ-specific stem cells, including LESC have been recently shown to exhibit a side population (SP)
phenotype. The SP cells are able to efflux Hoechst 33342 dye through the ATP-binding cassette transporter
Bcrp1/ABCG2. ABCG2 has therefore been proposed to be a universal marker for stem cells (Zhou et al., 2001;
Watanabe et al., 2004). In putative LESCs, this protein has been immunolocalised to the cell membrane and

cytoplasm of a population of limbal basal cells and a few suprabasal cells (Chen et al., 2004). Furthermore, ABCG2
positive cells produce higher colony forming efficiency values in vitro than their negative counterparts (de Paiva et
al., 2005). Our laboratory has localised ABCG2 to the outer edge of holoclones where it is thought that the stem
cells reside.

http://www.stembook.org/node/588
93) High molecular wt proteins in cataractous lens seen only in humans
A. HM 1&2
B. HM 2&4
C. HM 3&4
D. HM 2&3
ANS=C
REF=The Eye Part IB
edited by Hugh Davson,p310
Tanak and Benedek measured the protein diffusivity in intact human and bovine lens and could find
no evidence of protein longer than alfa-crystallin in normal lens(calf and human aged 43 years),but
found large aggregates in human cataracts .Of aggregates found in vitro,only fractions HM3 and
HM4 are found exclusively in human cataracts
Crystallin is found in epithelial and fiber cells of the ocular lens
Crystallin role structural
alpha crystallin role molecular chaperones, inhibits crystallin aggregation which form senile cataracts
The difference between normal and cataractous lens is the amount of water soluble (same in normal) and insoluble
(more in cataractous)
HM3 forms cortical cataracts
HM4 forms nuclear cataracts
tryptophan associated with yellowing of lens due to UV light oxidation
Rhodopsin initiates visual transduction, is an intrinsic membrane protein
opsin protein portion of rhodopsin
nonprotein portion of rhodopsin vitamin A
Mucous glycoprotein role biological lubrication, immunological recognition, structural orientation
Mucins mucus glycoproteins found in the tear layer, maintain stability of tear layer

mucins secreted by conjuctival goblet cells


Which types of collagen are in cornea I (70%), V, VI
Vitrosin Type II collagen found in the vitreous humor
Type IV collagen found in which structures of the eye bowmen's membrane, lens capsule, blood vessels
Descemet's membrance contains which type of collagen VIII, (IV also is associated on nonhelical regions)

94) Right trochlear nerve palsy can lead to all except


a) Diplopia on upward gaze and adduction
b) Right head tilt
c) Exotropia
d) Hyperopia
AB

A 4th nerve lesion causes atrophy of the superior oblique muscle. When looking down
and in (medially) with the bad eye there will be DIPLOPIA. The false image will lie
below the true image (vertical diplopia) and will be somewhat oblique (torsional
diplopia). The weakness of downward movement of the affected eye, most markedly
when the eye is turned inward, results in the patient complaining of difficulty in
especially reading or going downstairs.
The weakness of the superior oblique in the primary position (looking straight ahead)
results in the bad eye being slightly extorted and elevated due to the unopposed
action of the inferior oblique. This will result in torsional and vertical diplopia. For
instance, if the LEFT superior oblique is paralyzed, the LEFT eye is extorted and
elevated. In order to get rid of the torsional part of the double vision, the patient will
tilt their head to the side OPPOSITE the paralyzed muscle, that is to the RIGHT. This
causes reflex (from the otoliths) intorsion of the normal RIGHT eye (on side of head
tilt) so that the vertical axis of the two eyes become parallel (the eye associated with
the paralyzed superior oblique is already extorted by the unopposed inferior oblique).
To alleviate the vertical diplopia, the patient will also FLEX his/her chin when tilted
to the RIGHT. In this position the patient will have to elevate the normal RIGHT eye
in order to look straight ahead. The bad (LEFT) eye is already elevated and when
the two eyes are located at the same vertical (up-down) position in the socket, the
vertical diplopia is ameliorated.
REMEMBER, LESION OF TROCHLEAR NERVE (after it has crossed the midline)
= HEAD TILTED AWAY FROM PARALYZED MUSCLE; HEAD ALSO FLEXED

IN THIS POSITION. HOWEVER, IF LESION IS IN THE TROCHLEAR


NUCLEUS, HEAD TILT = TOWARDS THE LESION
http://www.neuroanatomy.wisc.edu/virtualbrain/BrainStem/19CNIV.html
95) Painful U/L dimness of vision
a) Optic neuritis
b) Aortic arch syndrome
c) Ischemic
d) ???
A
I. Differential Diagnosis: Acute Vision Loss based on pain
Minimal or variable pain with Vision Loss
Optic Neuritis ASSOCIATED with Multiple Sclerosis)
Retinal Detachment
Ocular tumor
Vitreous Hemorrhage
Central Retinal Artery Occlusion (Amaurosis Fugax)
Pain present
Acute Angle-Closure Glaucoma
Iritis
Corneal Ulcer
Temporal Arteritis (Giant Cell Arteritis)
advertisement
II. Differential Diagnosis: Acute unilateral Vision Loss
Transient
Central Retinal Vein Occlusion
Retinal Detachment (early)
Thromboembolism
Uveitis
Persistent
Acute Angle-Closure Glaucoma
Central Retinal Artery Occlusion
Central Retinal Vein Occlusion
Retinal Detachment (later)
Optic Neuritis (Multiple Sclerosis)
Ischemic Optic Neuropathy
Nonarteritic anterior optic Neuropathy (see Medications with Adverse Ocular Effects)
Eye Trauma
Tumor
Vitreous Hemorrhage
Occipital cortex infarction (vertebrobasilar thromboembolic event)
Endophthalmitis
Keratopathy

Acute Maculopathy
Psychogenic visual loss
III. Differential Diagnosis: Acute bilateral Vision Loss
Transient
Migraine Headache aura
Congestive Heart Failure
Hypertensive Emergency
Severe bilateral Carotid Artery Stenosis
Transient Ischemic Attack involving visual cortex
Persistent
Bilateral Occipital Lobe ischemia
Temporal Arteritis (Giant Cell Arteritis)
Lymphoma
Posterior ischemic Neuropathy
IV. Risk Factors: Acute Vision Loss predisposing factors
Diabetes Mellitus
Hypertension
Hyperlipidemia
Hypercoagulable States
Cardiac arrhythmias
Carotid Insufficiency
Glaucoma
Migraine Headaches

http://www.fpnotebook.com/Eye/Sx/ActVsnLs.htm
96) Cherry red spot after trauma is due to
a) CRVO
b) CRAO
c) Berlins edema
d) ??
C
Retina - CHERRY-RED SPOT IN MACULA
1. Cardiac myxomas
2. Cryoglobulinemia
3. Dapsone poisoning
4. Hallervorden-Spatz disease (pigmentary degeneration of globus pallidus)
5. Hollenhorst syndrome (chorioretinal infarction syndrome)
6. Hurler syndrome (MPS I-H)
*7. Hypertension (severe)
8. Intralesional chalazion corticosteroid injection
9. Leber congenital amaurosis
10. Macular hemorrhage

* 11. Macular hole with surrounding retinal detachment


12. ML I (lipomucopolysaccharidosis)
13. Myotonic dystrophy syndrome (Curschmann-Steinert syndrome)
14. Multiple sulfatase deficiency
*15. Occlusion of central retinal artery (see p. 457)
16. Quinine toxicity
17. Sphingolipidoses
A. Cherry-red spot myoclonus
B. Farber syndrome (Farber lipogranulomatosis)
C. Gangliosidosis GM1-type (juvenile gangliosidosis)
D. Gaucher disease (glucocerebroside storage disease)
E. Goldberg syndrome
F. Infantile metachromatic leukodystrophy (van Bogaert-Nyssen disease)
G. Niemann-Pick disease type A
H. Niemann-Pick disease type B
I. Sandhoff disease (gangliosidosis GM2-type 2)
*J. Tay-Sachs disease (gangliosidosis GM-type I)
18. Steroid injection intranasally
*19. Temporal arteritis (giant cell arteritis)
20. Traumatic retinal edema (commotio retinae; Berlin edema)
21. Vogt-Spielmeyer cerebral degeneration (Batten-Mayou syndrome)

97) Cherry red spot and Hollenhurst plaque are seen in


a) CRAO
b) CRVO
c) Branch RAO
d) Branch RVO
A
Emboli (clots) can be seen within the retinal arterial system in about 20% of eyes with
CRAO. The most common variant is the refractile yellow cholesterol embolus
(Hollenhorst plaque), which is thought to most commonly originate from
atherosclerotic disease in the carotid arteries in the neck. However, the central
retinal artery itself can have atherosclerotic plaque, and it too may produce emboli.
These cholesterol emboli are often small and may not totally obstruct retinal arteries.
They may even occur asymptomatically. Calcium emboli are less commonly seen, but
tend to be larger and cause more severe obstructions. They usually arise from
diseased cardiac valves.
The vision in eyes with central retinal artery obstruction ranges between counting
fingers and light perception in 90% of eyes at the time of initial examination. The
presence of an embolus is usually associated with poorer acuity. A few patients are
unable to even see light.

Approximately 25% of eyes with a CRAO also have sparing of part of the macular
(central) retina. This is due to the presence of a cilioretinal artery. These arteries are
usually derived from a different (choroidal) circulation so they are not affected by the
retinal artery occlusion. When a cilioretinal artery spares the central retina, the
vision may be initially poor, but over a period of weeks improves to 20/50 or better in
over 80% of eyes.
Although the incidences of rubeosis iridis and neovascular glaucoma following CRAO
have previously been thought to range from 1% to 5%, more recent data suggest that
the occurrence of iris neovascularization approaches 15% to 20%. The hypothesis has
been advocated that acute central retinal artery obstruction causes rapid inner-layer
retinal death, thus preventing the elaboration of an angiogenic factor. However,
fluorescein angiographic evidence suggests that the more severe, chronic CRAOs are
those that predispose to the development of rubeosis iridis.
http://www.williamsoneyeinstitute.com/retina-center/retinal-arteryocclusions
98) Drugs causing amorphous whorl like corneal opacity are a/e
a) Amiodarone
b) Chloroquine
c) Indomethacin
d) Chlorpromazine
ANS=D?
Epithelium
Vortex keratopathy, or corneal verticillata, is a common side effect of a number of
systemic medicationse.g., amiodarone, aminoquinolones, indomethacin, tamoxifen,
atovaquone and tiloroneand results from the intralysosomal accumulation of lipids.
This condition presents with golden-brown deposits in a whorl-like pattern normally
seen in the inferior corneal epithelium. This configuration is a result of the corneal
epitheliums growth and repair process.2 Patients who present with this finding are
typically asymptomatic, though they may report photophobia or halos around lights.
Vortex keratopathy as a side effect is indistinguishable from what we see in Fabrys
disease, an inherited lysosomal storage disease.
Amiodarone, an anti-arrhythmic agent, is strongly linked to the development of
corneal verticillata. In fact, it is present in 69% to 100% of patients taking 200mg to
2,400mg daily, and is detectable within four to six months. It is concentrated in the
tears and appears to be more severe in contact lens wearers.3 Discontinuation of the
drug typically allows for resolution of the keratopathy within three to 20 months.4-6
John Dovie, OD, and Andrew Gurwood, OD, reported on a case of amiodarone-induced
keratopathy with acute onset of bilateral corneal edema and subepithelial cysts that
caused decreased acuity, glare and halos, which persisted for two months after
discontinuation of the medication.7 Mesut Erdurmus, MD, and colleagues reported a
rare case of amiodarone-associated endothelial deposition, seen with a confocal laser

scanning microscope, in a patient taking 200mg daily for six years.8 Amiodarone has
also been linked to lenticular opacities and optic neuropathy.9,10
Chloroquine (Aralen, Sanofi Aventis) and hydroxychloroquine (Plaquenil, Sanofi
Aventis) are antimalarial agents used in the management of rheumatoid arthritis and
lupus. Corneal manifestations include vortex keratopathy and decreased corneal
sensation. The corneal findings are benign, but the retinal toxicity is concerning, as it
is irreversible (even with discontinuation of the drug) and can lead to permanent
central and peripheral vision loss.11
While corneal findings had been thought to have no correlation with the development
of retinal toxicity, Aljoscha Neubauer, MD, and colleagues conducted a screening of 93
patients with marked corneal deposits who were taking either chloroquine or
hydroxychloroquine, and using electro-oculogram and computerized color vision
testing, found a 50% sensitivity and 90% specificity for retinopathy.12,13
Tamoxifen, an estrogen antagonist used in the long-term treatment of breast cancer,
creates bilateral, white or multi-colored, whorl-like, central subepithelial opacities
that can cause reduced visual acuity.14 Tamoxifen retinopathy is rare, but has been
detected at even low levels of treatment. After tamoxifen cessation, almost all of the
ocular abnormalities are reversibleexcept for the retinal opacities, which can
include bilateral macular edema, and yellow-white dots in the paramacular and
foveal areas.
http://www.reviewofcontactlenses.com/con/d/disease/c/40061/
-------------------------------------------------------------------------------------------Tiny, yellow-white, axially-placedanterior subcapsular lenticular opacitieshave been
reported in patients takingamiodarone6. It is unlikely that theseopacities would cause
a loss of vision andsimilar deposits have been attributed tochloroquine7.^^^^^ They
should be readilydistinguishable from axial pigmentationof the lens - an hourglasslikeconglomeration of tiny pigment depositsseen in many patients in the older
agegroups, and the larger stellate opacitiesfrom the use of chlorpromazine^^^^^^^^
http://www.optometry.co.uk//5ccb4a0b3629b7e7cc53cfbb916c5f
99) True statements regarding Direct Opthalmoscopy are a/e
a) 2 disc diameter field of vision
b) Image is virtual and erect
c) Magnification is 5 times
d) Self illuminated device
C
Direct Ophthalmoscopy. Examination of the optic nerve can be performed with a handheld direct
ophthalmoscope. Direct ophthalmoscopy provides an upright, magnified (approximately 15x's),
monoscopic view of the optic nerve. The structure of the exam is similar to the one described below
with the 7 Steps. Depending on the specific patient exam, certain features of the ophthalmoscope
should be adjusted. For example, the size of the pupil influences the optimal aperture selected.
Ideally, the aperture should approximate the size of the pupil as this will allow sufficient light to enter
eye to provide enough illumination of the fundus. The selection of an aperture larger than the pupil
introduces more light than can enter the eye and creates secondary problems such as glare and

constriction of the pupil. Additonally, the use of the red-free light can help identify hemorrhages such
as optic nerve head (Drance) hemorrhages.
Slit-lamp Exam. Slit-lamp examination of the optic nerve is ideal and uses hand-held lenses to
magnify and control the view of the exam. This method provides a stereoscopic, inverted view of the
optic nerve.
Optic Disc Stereophotography. Monitoring progression of optic nerve changes can best be
performed with optic disc stereophotography. This method of comparing prior with current optic
nerve images allows the examiner to evalute for both gross and fine changes. For example,
examination using this method allows for changes seen with optic nerve head hemorrhages that may
lead to notching, or monitoring progressiving neuroretinal rim loss by following barring of vessels.

Ancillary Testing[edit source]


GDx. The GDx involves scanning laser polarimetry that uses polarized light and bifirengence to
monitor retinal nerve fiber layer.
OCT. The OCT (Optical Coherence Tomography) can be used to evaluate the optic nerve based on
low coherence interferometry.
HRT. The HRT (Heidelberg Retina Tomograph) is a confocal scanning laser that uses multiple cross
section images to create and analyze three dimensional structures such as the optic nerve.

http://eyewiki.aao.org/Examination_of_the_optic_nerve_at_the_slitlamp_biomicroscope_with_a_handheld_lens

100) In a group of 100 people, the average GFR is 85 ml/min with a


standard deviation of 25. What is the range for 90% confidence interval
a) 80-90
b) 81-89
c) 75-95
d) 70 -100
B

Confidence Interval Estimates for Smaller Samples

With smaller samples (n< 30) the Central Limit Theorem does not apply, and another distribution
called the t distribution must be used. The t distribution is similar to the standard normal
distribution but takes a slightly different shape depending on the sample size. In a sense, one
could think of the t distribution as a family of distributions for smaller samples. Instead of "Z"
values, there are "t" values for confidence intervals which are larger for smaller samples,
producing larger margins of error, because small samples are less precise. t values are listed by
degrees of freedom (df). Just as with large samples, the t distribution assumes that the outcome
of interest is approximately normally distributed.

A table of t values is shown in the frame below. Note that the table can also be accessed from
the "Other Resources" on the right side of the page.

Confidence Intervals for One Sample: Continuous


Outcome
Suppose we wish to estimate the mean systolic blood pressure, body mass index, total
cholesterol level or white blood cell count in a single target population. We select a sample and
compute descriptive statistics including the sample size (n), the sample mean, and the sample
standard deviation (s). The formulas for confidence intervals for the population mean depend on
the sample size and are given below.
Confidence Intervals
for
For n > 30
For n < 30

Use the Z table for the


standard normal
distribution.

Use the t table with


df=n-1

Example
Descriptive statistics on variables measured in a sample of a n=3,539 participants attending the
7th examination of the offspring in the Framingham Heart Study are shown below.
Characteristic

Sample
Mean

Standard Deviation
(s)

Systolic Blood Pressure 3,534

127.3

19.0

Diastolic Blood
Pressure

3,532

74.0

9.9

Total Serum Cholesterol 3,310

200.3

36.8

Weight

3,506

174.4

38.7

Height

3,326

65.957

3.749

Body Mass Index

3,326

28.15

5.32

Because the sample is large, we can generate a 95% confidence interval for systolic blood
pressure using the following formula:

The Z value for 95% confidence is Z=1.96. [Note: Both the table of Z-scores and the table of tscores can also be accessed from the "Other Resources" on the right side of the page.]
Substituting the sample statistics and the Z value for 95% confidence, we have

Therefore, the point estimate for the true mean systolic blood pressure in the population is
127.3, and we are 95% confident that the true mean is between 126.7 and 127.9. The margin of
error is very small (the confidence interval is narrow), because the sample size is large.
The 90% confidence interval for mean systolic blood pressure would be:

What is the 90% confidence interval for BMI? (Note that Z=1.645 to reflect the
90% confidence level.)

Answer
The table below shows data on a subsample of n=10 participants in the 7th examination of the
Framingham offspring Study.
Characteristic

n Sample Mean

Standard Deviation (s)

Systolic Blood Pressure

10

121.2

11.1

Diastolic Blood Pressure

10

71.3

7.2

Total Serum Cholesterol

10

202.3

37.7

Weight

10

176.0

33.0

Height

10

67.175

4.205

Body Mass Index

10

27.26

3.10

Suppose we compute a 95% confidence interval for the true systolic blood pressure using data
in the subsample. Because the sample size is small, we must now use the confidence interval
formula that involves t rather than Z.

The sample size is n=10, the degrees of freedom (df) = n-1 = 9. The t value for 95% confidence
with df = 9 is t = 2.262.

Substituting the sample statistics and the t value for 95% confidence, we have

Interpretation: Based on this sample of size n=10, our best estimate of the true mean
systolic blood pressure in the population is 121.2. Based on this sample, we are 95% confident
that the true systolic blood pressure in the population is between 113.3 and 129.1. Note that the
margin of error is larger here primarily due to the small sample size.

http://sphweb.bumc.bu.edu/otlt/MPHModules/BS/BS704_Confidence_Intervals/BS704_Confidence_Intervals_print.html
101) Nikshay is a newly launched central government software. It is used for
tracking
a) TB cases
b) High risk pregnancies
c) High risk new borns
d) Cases with malaria
A
NIKSHAY- A web based solution for monitoring of TB patients To monitor Revised
National Tuberculosis Programme (RNTCP) effectively, a web enabled and case based
monitoring application called NIKSHAY has been developed by National Informatics
Centre (NIC). This is used by health functionaries at various levels across the country in
association with Central TB Division (CTD), Ministry of Health & Family Welfare.
NI KSHAY covers various aspects of controlling TB using technological innovations. Apart

from web based technology, SMS services have been used effectively for communication
with patients and monitoring the programme on day to day basis.

http://nikshay.gov.in/AboutNikshay.htm

Nikshay (Case Based


online software)
Background: RNTCP since implementation followed international
guidelines for recording and reporting for Tuberculosis Control
Programme with minor modifications. Epi-info based EPI-CENTRE
software was being used for the purpose of electronic data
transmission from district level upwards. Initially DOS version was in
use and the programme shifted to windows version in 2007. However,
the data available at district, state or national level was in aggregated
form, with a lead time of >4 months, excluding private sector and
neither could help much for TB burden estimation or individual case
management or monitoring. To address this Central TB Division (CTD)
in collaboration with National Informatics Centre (NIC) undertook
the initiative to develop a Case Based Web online (cloud) application
named Nikshay. This software was launched in May 2012 and has
following functional components.
yy Master management
yy User details
yy TB Patient registration & details of diagnosis, DOT Provider, HIV
status, Follow-up, contact tracing, Outcomes
yy Details of solid and liquid culture & DST, LPA, CBNAAT details
yy DR-TB patient registration with details
yy Referral and transfer of patients
yy Private health facility registration and TB Notification
yy Mobile application for TB notification
yy SMS alerts to patients on registration
yy SMS alerts to programme officers
yy Automated periodic Reports
. Case Finding
. Sputum conversion
. Treatment outcome

Data security / confidentiality


Security audit of Nikshay application is done as per guidelines of Department of IT. Password
protection is

applicable for each level of user. Password reset facility is available at higher users in hierarchy.
Access to
relevant information for each user, based on defined functions.

Data quality
Since the software do not itself generate information and almost all information is digitized from
the source
which exists in the programme; the inherent quality of data of the programme is transferred.
Transcription
errors if any are being evaluated by the programme in implementation research mode. However,
Nikshay
already has internal validations for most of the variables based on the logic flow and
conditionalitys. But a
judgement of choice of stricter validations against the availability of complete and accurate
information; is
also an opportunity to improve processes in the programme. It started with certain mandatory
fields which
were defined and these now ensure completeness of information regarding those variables e.g.
DOT provider
details. Unwanted characters avoided at entry. Regular feedback from administrator to check bugs
if any, has
been established. Most importantly data point formats of Metadata and Data Standards (MDDS)
have been
followed in the development of this could application. In future, this will be the basis for system
integration and
interoperability to set an example of EMR/EHR.
Till 10th March 2014, status of implementation and is as below:
TB Patients Registered under RNTCP 23,69,515
Peripheral Health Institutes (PHI) registered 41,277
Tuberculosis Officials details 2703
District TB Officers details 667
State TB Officers details 35
Contractual Employees details 6901
Non-RNTCP Health Establishments registered 64,073
Non-RNTCP Patients registered 56,087
Culture & Drug Resistant Labs Patients registered 20995
Drug Resistant Tuberculosis Patients registered 1979

Implementation challenges
Many of the PHCs in the country do not have adequate ICT infrastructure like computer, internet
connectivity

and Data Entry Operator. Also intermittent electricity supply hampers the data entry and use of
Nikshay. Also
patient treatment cards need to be brought to TU/Block level or even at district level in certain
areas for data
entry. Slow internet / web connectivity in some places and incomplete treatment cards at many
places also
slows down the process.
However, support from NRHM in terms of ICT infrastructure and data entry operators has
significantly contributed
to use of Nikshay software across the country.

National e-Governance award


NIKSHAY was honored with National
e-Governance Award (Gold) 2013-14
during the National Conference on
e-Governance held at Kochi on 30-31
January, 2014 under category Sectoral
Award Healthcare. These awards are
organized by Ministry of Administrative
Reforms, Grievence Redressal and
Pensions in collaboration with Department
of IT, Ministry of Communication and
Information Technology, Government of
India.
The award was given by Sh Nikhil Kumar,
Honble Governor of Kerala, Sh V. Narayanasamy, Honble Minister for State of Personnel, Public
Grievances and
Pensions. The gold award was received by officials from Central TB Division and NIC.

102) Sensitivity measures


a) True positive
b) True negative
c) False positive
d) False negative
A
103) In a survey the highest sample of 58 was entered by mistake as 85. This will lead
to
a) Increased mean, increased median

b) Increased mean, no change in median


c) No change in mean, increased median
d) Increased mean, decreased median
B
104) Immunisation statistical data of the community can be found by
a) Sample registration system
b) District health mission
c) national health survey
d) ??
C
The National Health Interview Survey (NHIS) provides information on the health status of the U.S. civilian
noninstitutionalized population through confidential interviews conducted in households by Census Bureau
interviewers. NHIS is the Nation's largest in-person household health survey, providing data on health status, access
to and use of health services, health insurance coverage, immunizations, risk factors, and health-related
behaviors.

http://www.cdc.gov/nchs/surveys.htm
National Family Health Survey (NFHS): The 2005-06, National Family Health Survey
(NFHS-3) was the third in a series of national surveys preceded by earlier NFHS
surveys carried out in 1992-93 (NFHS-1) and 1998-99 (NFHS-2) with the objective to
provide essential data on health and family welfare needed by the Ministry of Health
and Family Welfare and other agencies for policy and programme purposes, and to
provide information on important emerging health and family welfare issues.
Annual Health Survey (AHS): The Ministry of Health & Family Welfare, in
collaboration with the Registrar General of India (RGI), had launched an Annual
Health Survey (AHS) in the erstwhile Empowered Action Group States (Bihar,
Jharkhand, Madhya Pradesh, Chhattishgarh, Uttarakhand, Uttar Pradesh, Orissa and
Rajasthan) and Assam. AHS will provide District-wise data on Total Fertility Rate
(TFR), Infant Mortality Rate (IMR) and the Maternal Mortality Ratio (MMR) at the
regional level. Other RCH indicators like Ante-natal care, Institutional delivery,
immunisation, use of contraceptives will also be available. The aim of the survey
was to provide feedback on the impact of the schemes under NRHM in reduction of
Total Fertility Rate (TFR), Infant Mortality Rate (IMR) at the district level and the
Maternal Mortality Ratio (MMR) at the regional level by estimating these rates onanhttp://mohfw.nic.in/

WriteReadData/l892s/3503492088FW%20Statistics%202011%20Revised
%2031%2010%2011.pdf
annual basis for around 284 districts in these States.

105) Tuberculin test is a cheap and logistically simple test. But it leads to confusion
because of

a) High percentage of immunized people


b) HIV cases are less?
c) Infection is more?
d) ??
A
106) Food safety and standards authority of India comes under which ministry
a) Rural statistics
b) Ministry of Health and Family and welfare
c) District level
d) Ministry of Agriculture
B
107) Haddon matrix is related to
a) Injury prevention
a
Preventing Injuries: the Haddon Matrix
1. Core Knowledge:
William Haddon made a useful contribution to thinking about how injuries occur and how to prevent
them. The "Haddon Matrix" is a table showing the host, agent and environmental factors involved,
set against the time sequence of an incident.
The cells of the matrix illustrate the range of risk or protective factors involved; Haddon emphasized
the multidisciplinary nature of potential interventions.
Nice to Know:
Note how the matrix links neatly to the stages of prevention. In this context, primary prevention
refers to interventions before the event, to prevent it completely; secondary prevention involves
lessening the extent of injury given that an event occurs (e.g., wearing a seatbelt will not prevent the
collision but may lessen its effects). Tertiary prevention limits the subsequent difficulties a person
encounters given his injuries (rehabilitation, etc.). You couls also add primordial prevention (not
shown above), which would refer to underlying social change that would alter the circumstances in
which the accident occurred. For example, encouraging public transportation would reduce crowding
on the roads, likely preventing some accidents.
Haddon also proposed a generic sequence of ten countermeasures to reduce the risk of injuries.

http://www.med.uottawa.ca/sim/data/Injury_Prevention_Haddon_e.htm

108) Increasing number of non communicable disease when compared to previous


years is a trend called as
a) Cyclical

b) Periodical
c) Secular
d) Seasonal
c
Secular trend Implies long-term changes in the frequency of diseases occurring over many years or
decades. ... Among the non-communicable diseases, diabetes and coronary heart disease are
depicting an upward secular trend
REF=Foundations of Community Medicine, 2/e
Dhaar - 2008

109) In a study two groups of newborns are checked for their weights based on
whether their mothers received food supplements or not. The appropriate test which
is can be used for comparing the data is
a) Fischer exact test
b) Student T test
c) Paired T test
d) Chi square test
ANS=B
Student's' t Test (For Paired Samples)
Use this test to compare two small sets of quantitative data when data in each sample set are
related in a special way.
Criteria
The number of points in each data set must be the same, and they must be organized in pairs, in
which there is a definite relationship between each pair of data points
If the data were taken as random samples, you must use the independent test even if the number of
data points in each set is the same
Even if data are related in pairs, sometimes the paired t is still inappropriate
Here's a simple rule to determine if the paired t must not be used - if a given data point in group one
could be paired with any data point in group two, you cannot use a paired t test
Examples
The paired t test is generally used when measurements are taken from the same subject before and
after some manipulation such as injection of a drug. For example, you can use a paired t test to
determine the significance of a difference in blood pressure before and after administration of an
experimental pressor substance. You can also use a paired t test to compare samples that are
subjected to different conditions, provided the samples in each pair are identical otherwise. For
example, you might test the effectiveness of a water additive in reducing bacterial numbers by
sampling water from different sources and comparing bacterial counts in the treated versus
untreated water sample. Each different water source would give a different pair of data points.

The value of the paired t test is best demonstrated in an example. Suppose patient 1 responds to a
drug with a 5 mm Hg rise in mean blood pressure from 100 to 105. Patient 2 has a 30 mm Hg rise,
from 90 to 120. Likewise for several other subjects. The response to the drug varied widely, but all
patients had one thing in common - there was always a rise in blood pressure. Some of that
experimental error is avoided by the paired t test, which likely will pick up a significant difference.
The independent test, which would be improperly applied in this case, would not be able to reject the
null hypothesis.
Be certain that use of the paired t test is valid before applying it to real data. An applied statistics
course or supervision of a qualified mentor may provide the experience you need.
Some spreadsheet programs include the paired t test as a built-in option. Even without a built-in
option, is is so easy to set up a spreadsheet to do a paired t test that it may not be worth the
expense and effort to buy and learn a dedicated statistics software program, unless more
complicated statistics are needed.

110) A researcher wants to do a study of ?Plasma lipids among people who jog and
those who dont. But he is now concerned that the joggers might differ from the
others in their diet, etc as well. This concern is known as
a) Recall bias
b) Interviewer bias
c) selection bias
d) confounding bias?
ANS=C?
111) Human Developmental Index is a composite measure which uses
a) Life expectancy at age one, standard of living
b) Freedom, spice and all things nice icon_wink.gif
c) Life expectancy at birth, knowledge, and income
d) Life expectancy at age one, knowledge and standard of living
C
112) Window period is defined as the time taken from
a) Entry of pathogen to appearance of first clinical symptoms
b) Exposure to laboratorial detection of disease
c) ??
d) ??
C
113) Screening is not useful in which carcinoma
a) Ca Prostate

b) Ca Colon
c) Ca Breast
d) Ca Testis
d
114) A 10 yr old boy with dog bite comes to you. Appropriate action is
a) Give CCDV
b) Withhold vaccine and observe dog for 10 days
c) Kill dog and send brain for biopsy
d) ??
A
115) Tropical spastic paraparesis is caused by
a) HTLV
b) Hep B virus
c) HIV
d) ??
A
116) A person is HBsAg positive, but anti- HBc ab is negative. What should be the next
step
a) Repeat test after 6 months
b) Check HBeAg, if positive start trt
c) Check HBV DNA load
d) ??
C
Initial Assessment of Positive Hepatitis B Surface Antigen
Initial laboratory screening to diagnose hepatitis B virus (HBV) infection in an
asymptomatic individual generally consists of testing for hepatitis B surface antigen
(HBsAg), hepatitis B surface antibody (anti-HBs), and hepatitis B core antibody (antiHBc)[1,2]. For those patients who have a positive HBsAg, follow-up laboratory testing
is indicated to define the stage of their hepatitis B infection. The follow-up laboratory
evaluation of a patient with a positive HBsAg should include:
Hepatitis B core IgM antibody (IgM anti-HBc)
Hepatitis B e antigen (HBeAg) and hepatitis B e antibody (anti-HBe)
HBV DNA level
Hepatic panel, including measurement of alanine aminotransferase (ALT or SGPT) and
aspartate aminotransferase (AST or SGOT), levels, albumin, and prothrombin time
The interpretation of the results of these tests (Figure 1) can determine whether the

patient has acute HBV infection, chronic immune tolerant infection, chronic immune
active infection with wild-type virus, chronic immune active infection with precore or
core promoter mutant virus, or chronic inactive infection (carrier). The phases and
classification of chronic HBV infection are discussed in detail in the case Natural
History of Chronic Hepatitis B Infection and as noted in the discussion of the natural
history, patients with chronic hepatitis B virus infection can have a dynamic transition
between the different phases of chronic infection.
Acute Infection with HBV
In the setting of acute HBV infection, HBsAg typically becomes detectable 4 to 8
weeks after infection. Shortly thereafter, IgM anti-HBc appears in the blood. Thus, the
diagnosis of acute hepatitis B is generally made by the simultaneous detection of
HBsAg and IgM anti-HBc[3]. The serologic and virologic response to acute HBV
infection is discussed in detail in the case Serologic and Virologic Markers of Hepatitis
B Virus Infection. Rarely, acute hepatitis B may be diagnosed during the period when
HBsAg titers have declined below detectable levels and anti-HBs have not yet
appeared. In this scenario, known as the "window period", the diagnosis of acute HBV
infection is made based on the presence of positive IgM anti-HBc titers (Figure 2).
These patients will thus have isolated anti-HBc as the only marker of acute hepatitis B
infection; other scenarios exist that can cause isolated anti-HBc as discussed in detail
in the case Interpretation of Isolated Hepatitis B Core Antibody (anti-HBc). The HBV
DNA titers are extremely high early in acute infection, often reaching 200 million IU/
mL (1 billion copies/ml)[4]. At this stage, HBeAg is also usually detected, hepatic
aminotransferase levels become elevated after HBsAg turns positive, and serum
bilirubin may also increase. In severe cases, hepatic production of albumin may
significantly decline and other evidence of hepatic dysfunction, such as prolonged
prothrombin time, may be present[5].
The differential diagnosis of acute hepatitis B infection includes exacerbation of
chronic hepatitis B, superinfection of hepatitis B carriers with hepatitis A, C, or D, and
acute hepatotoxicity caused by drugs or other toxins[1,2]. During exacerbations of
chronic hepatitis B, anti-HBc IgM may become transiently positive, making
differentiation from acute hepatitis B difficult. In such cases, previous test results or
a history of recent exposure may assist in the diagnosis of acute hepatitis B. In the
other conditions mentioned above, anti-HBc IgM is usually not present[5]. For the
patient discussed in this case, the initial laboratory workup should include
measurement of anti-HBc IgM. If this test is positive, the diagnosis of acute hepatitis B
is likely, although an acute exacerbation of chronic hepatitis B cannot be ruled out
without further information.
Chronic Immune Tolerant HBV
Immune tolerant hepatitis B infection refers to the clinical scenario of persistently
normal hepatic aminotransferase levels in the presence of high titers of circulating
HBeAg and HBV DNA. These patients with immune-tolerant hepatitis B usually have

become infected early in life through vertical or early horizontal infection. Such
infection most often occurs in areas with high rates of endemic infection, low rates of
maternal screening, and lack of widely available neonatal prophylaxis with HBV
vaccine and hepatitis B immune globulin[6]. Clinically, it is important to determine if
patients have evidence of immune tolerance, because such patients generally respond
poorly to antiviral therapy, and their management differs from patients with more
active hepatic inflammation[7].
Chronic Immune Active HBV Infection (Wild-Type)
Patients with chronic hepatitis B infection caused by wild-type virus (defined as a
naturally-occurring strain without known mutations) have high titers of circulating
HBeAg, almost always coupled with high titers of HBV DNA (defined as greater than
200,000 IU/ml). Indeed, prior to the widespread availability of HBV DNA assays, HBeAg
was considered to be the principle marker of HBV replication and high infectivity[3].
The laboratory profile of patients with chronic immune active (wild-type) hepatitis B
typically shows the following:
HBsAg positive longer than 6 months
HBeAg positive, anti-HBe negative
Serum HBV DNA greater than 200,000 IU/ml
Elevated or normal hepatic aminotransferase levels
The natural history for patients with immune active chronic hepatitis B is substantially
worse than that of patients in the inactive carrier state. Patients with chronic
hepatitis B who become inactive carriers either through spontaneous or treatmentinduced HBeAg seroconversion demonstrate improvement in clinical and biochemical
evidence of liver disease[6,8,9]. Furthermore, patients with HBeAg positive chronic
hepatitis B have a greater than six-fold increased relative risk of developing
hepatocellular carcinoma compared with HBeAg-negative inactive hepatitis B carriers
(relative risk, 60.2 versus 9.6)[10]. Patients with chronic immune-active HBV infection
usually have intermittently elevated, or persistently elevated liver enzymes.
Chronic Immune Active HBV (Precore and Core Promoter Mutants)
Some individuals with chronic hepatitis B are infected with a mutant HBV variant that
results in HBeAg negative chronic hepatitis. In such patients, viral mutations in the
precore or core promoter regions prevent HBeAg production in an otherwise normally
replicating HBV. Thus, these patients typically have high serum HBV DNA levels, but
negative HBeAg titers. The prevalence of precore or core promoter mutations is
highest among persons from Southern Europe and Asia, with prevalence estimates of
60 to 70%[11]. Chronic HBeAg-negative hepatitis B is diagnosed in a patient with the
following laboratory profile:
HBsAg positive longer than 6 months
HBeAg negative, anti-HBe positive
Serum HBV DNA greater than 20,000 IU/ml
Persistently elevated or intermittently normal hepatic aminotransferase levels
Accurate diagnosis of chronic HBeAg-negative hepatitis B is imperative because the

clinical management is markedly different from that of chronic inactive hepatitis B


carriers (who are also HBeAg negative but have low serum HBV DNA titers).
Chronic Inactive Carrier of HBV
Chronic inactive hepatitis B infection is defined as persistent HBV infection of the
liver without significant ongoing hepatic inflammation and necrosis[12]. Blood tests
on such patients are typically show four characteristic features:
HBsAg positive for longer than 6 months
HBeAg negative, anti-HBe positive
Serum HBV DNA less than 2,000 IU/ml
Persistently normal hepatic aminotransferase levels
The "chronic inactive carrier" state may persist for decades. Patients who remain in
this phase of infection have lower rates of disease progression and hepatocellular
carcinoma[13]. Generally, antiviral treatment is not indicated for such patients,
unless histologic or clinical signs of cirrhosis are present[12].
Case Summary
The patient's blood tests yielded the following results:
HBsAg positive, anti-HBs negative
Anti-HBc IgM negative, Anti-HBc IgG positive
HBeAg positive, anti-HBe negative
Serum HBV DNA 200 million IU/ml
ALT = 120 U/L, AST = 86 U/L, Albumin = 4.1 g/dL
Because the patient is HBeAg positive with a high viral load, he has chronic immune
active hepatitis B infection with wild-type virus. Because the ALT and AST are
elevated, he is not in the immunotolerant phase of infection, but instead has
evidence of hepatic inflammation.
http://depts.washington.edu/hepstudy/hepB/mgmt/hbsag/discussion.html

117) In a female who had Steroid Resistant ITP it was decided to perform
splenectomy. On day 3 post laparoscopic surgery patient had fever. Which of the
following scenarios is likely
a) Left lower lobe consolidation
b) Port site infection
c) Intraabdominal collection
d) Urine for pus should be sent
ANS=A
WHAT ARE THE COMPLICATIONS OF SURGERY TO REMOVE THE SPLEEN?
Even though the incisions are small, spleen removal is still a considered a major operation. Whilst
laparoscopic surgery is considered a relatively safe and low risk operation, all surgery carries a
number of serious complications. It must be stressed, these complications are very rare.
Complications are dealt with on a case by case basis. Some of these are:

General Risks:

Death: approx. 1/10,000 risk for all patients having this type of operation.

Allergic reaction or airway problems related to the anaesthetic.

Bleeding: usually occurs either during the operation or in the first 24 hours and may require
further surgery. You may require a blood transfusion.

Blood vessel problems: heart attack, stroke. This is very rare.

Infections: Wound, pneumonia, urine, intra-abdominal, IV line related.

Clots in the legs that may travel to the lungs. This may be fatal.

Wound pain, abnormal (keloid) scarring or hernia of the wound.

Bowel obstruction due to hernia or adhesions.

Risks Specific to Laparoscopic Splenectomy:

Injury to the tail of the pancreas resulting in a collection of fluid in the abdomen that may
require a further operations or drainage procedures.

Bleeding from the blood vessels that flow to the spleen requiring a return to the operating

theatre.

Significant distention of the stomach that may lead to a large vomit. Occasionally some of
this vomit may be inhaled into the lungs and cause lifethreatening pneumonia. This is why a tube will
be place via your nose into the stomach for the first day after the operation.

Because the spleen is very close to the left lung, partial collapse of the left lung is
quite common after splenectomy. The physiotherapist will work with you to prevent this. It is
very common to have a slight fever on the first 1-2 days after the operation because of this
lung collapse.

Injury to any organ in the abdomen: bowel, aorta, liver, stomach. This is rare.

Gas Embolism a bubble of carbon dioxide gets into a blood vessel and causes life
threatening heart problems. This is very, very rare.

Re-operation: if the surgeon has to re-operate for any reason, this may be done with keyhole
surgery or an open operation.

Life-threatening infections related to having no spleen (see below).

http://www.brisliver.com.au/laparoscopicsplenectomy.html
1. Collapse and consolation can occur independently or together
2. Collapse can be partial or complete
3. It is often not clear to what extent the appearance is due to collapse or consolidation or
both. The degrees of each are often unclear.
4. If a lobe is only partially collapsed and there is no accompanying consolidation, there
may be no increase in opacity
5. In cases of pure collapse, only when the collapse is virtually complete will there be a
significant increase in density of the affected lung
Left Lower Lobe (LLL) Collapse

The PA view will show a triangular area of increased opacity behind the left heart
shadow.

There may be loss of visualisation of the left hemi-diaphragm behind the heart

The lower lobes collapse


o
downward
o
medially toward the spine and
o
posteriorly

http://www.wikiradiography.net/page/Patterns+of+Collapse

ANS=C?
lap splenectomy is followed by fever & tachycardia on 3rd post op day .next best invesigation
finding
is:
a)pus
in
urine
b)LRTI
c)intraabdominal
collection
on
USG
d)pus in port site

ANS=B>C

Introduction
Surgical site infection (SSI) appear to be more frequent in splenectomized patients than might be expected and its
incidence can be explained neither by the extent of surgery nor by the risk of bacterial contamination of the
operating field.
Aim
Evaluation the local antibiotic prophylaxis using a gentamicin surgical implant in order to reduce SSI, particularly
subphrenic abscesses.
Introduction
The spleen is one of the most crucial elements of the human immune system, but its function has not been fully
recognized [1]. At first, it was believed that its removal did not entail any significant implications. Only later
observations of long-term infective complications after splenectomy, such as fulminating sepsis referred to as
overwhelming post-splenectomy infection (OPSI) revealed the risk. Because of them, it is currently known that
spleen removal causes significant impairment of the immune mechanisms [2, 3].**** Early postoperative, local,
infective complications constitute a somewhat less known problem. It has been observed that in patients after
splenectomy the surgical site becomes infected (surgical site infections SSI) much more often than could be
expected. Especially, abscesses occur frequently at the site of the removed spleen.****** Its incidence can be
explained neither by the range of the procedure nor by the risk of surgical field infection. The considerable risk of

abscess occurrence in the splenic site causes preventive actions. Perioperative systemic antibiotic prophylaxis does
not ensure complete protection [4]. Research on local use of antibacterial drugs has been undertaken in order to
prevent infections in the splenic site. In the literature concerning the issue no reports of studies of such kind were
found.

http://www.ncbi.nlm.nih.gov/pmc/articles/PMC3516976/

118) Which of the following is not an auto immune disorder?


a) Ulcerative colitis
b) Graves disease
c) Rheumatoid arthritis
d) SLE
A
119) Which of the following is false about TRALI
a) Develops within 24 hours
b) Mostly seen after sepsis and cardiac surgeries
c) Its a cause of non cardiogenic pulmonary edema
d) Plasma is more likely to cause it than whole blood
A
Transfusion-Related Acute Lung Injury (TRALI)
clinical TRALI
Author: Tanya Petraszko, MD, FRCP(C)
Description and Incidence
Transfusion-Related Acute Lung Injury (TRALI) is a syndrome characterized by acute
respiratory distress following transfusion. All plasma-containing blood products have
been implicated including rare reports of IVIG and cryoprecipitate. It is a rare
complication of allogeneic blood transfusion but the incidence has not been well
established due to difficulty in defining the syndrome and to variable reporting
mechanisms worldwide. Various studies have estimated the overall frequency of TRALI
to be between 1/1,120 and 1/57,810 units transfused. However, there is wide
discrepancy in the literature with the reported frequency is as low as 1/557,000 RBC
units and as high as 1/432 platelet units.
TRALI is associated with a high morbidity with the majority of patients requiring
ventilatory support. However, the lung injury is generally transient with PO2 levels
returning to pretransfusion levels within 48 -96 hours and CXR returning to normal
within 96 hours. TRALI is associated with a significant mortality rate, often
approximated at 5 to 10%. Given the gains in safety made within the blood component

production industry, particularly with respect to transmission of infectious diseases,


TRALI is now among the three leading causes of transfusion related fatalities along
with ABO incompatibility and bacterial contamination.
Clinical Presentation
Symptoms of TRALI typically develop during, or within 6 hours of a transfusion.
Patients present with the rapid onset of dyspnea and tachypnea. There may be
associated fever, cyanosis, and hypotension. Clinical exam reveals respiratory distress
and pulmonary crackles may be present with no signs of congestive heart failure or
volume overload. CXR shows evidence of bilateral pulmonary edema unassociated
with heart failure (non-cardiogenic pulmonary edema), with bilateral patchy
infiltrates, which may rapidly progress to complete "white out" indistinguishable from
Acute Respiratory Distress Syndrome (ARDS).
Treatment and Clinical Course
Treatment of TRALI is supportive. Mild forms of TRALI may respond to supplemental
oxygen therapy. Severe forms may require mechanical ventilation and ICU support. As
with ARDS there is no role for diuretics or corticosteroids. The majority of patients
recover within 72 to 96 hours and subsequently recover to their baseline pulmonary
function without apparent sequelae. However, some patients are slower to recover
and may remain hypoxic with persistent pulmonary infiltrates up to seven days. As
stated above, approximately 5 to 10% of cases are fatal in spite of aggressive
supportive care.
Differential Diagnosis
The differential diagnosis of acute lung injury after transfusion includes transfusionassociated circulatory overload (TACO), cardiogenic edema, allergic and anaphylactic
transfusion reactions, and bacteremia/sepsis due to transfusion of bacterially
contaminated blood products.
TRALI may be distinguished from TACO and cardiogenic pulmonary edema by the
absence of signs of circulatory overload such as a normal central venous pressure
(CVP) and normal pulmonary capillary wedge pressure (PCWP). Clinical response to
diuretics also suggests a diagnosis of TACO rather than TRALI. Allergic and
anaphylactic transfusion reactions may be manifest as hypotension and respiratory
distress but are marked by laryngeal edema or bronchospasm with wheezing and a
normal CXR. Transfusion transmitted bacteremia my present with fever, hypotension,
and culminate in severe sepsis with associated acute lung injury which may be
difficult to distinguish from TRALI. The presence of positive blood cultures is a useful
delineating finding.

Pathophysiology
The hallmark of acute lung injury (ALI) is that of increased pulmonary microvascular
permeability with increased protein in the edema fluid. This is true regardless of the
cause of the ALI.
It is hypothesized that TRALI may be precipitated by the infusion of donor antibodies
directed against recipient leukocytes. The infusion of donor anti-HLA (human
leukocyte antigens) or anti-HNA (human neutrophil antigens) antibodies is thought to
directly cause complement activation, resulting in the influx of neutrophils into the
lung, followed by neutrophil activation and release of cytotoxic agents, with
subsequent endothelial damage and capillary leak. Donor derived antibodies to HLA
class I antigens and neutrophils have been demonstrated in up to 89% of TRALI cases
examined in the literature.
An alternate hypothesis argues that TRALI is the result of at least two independent
clinical events: the first is related to the clinical condition of the patient (infection,
cytokine administration, recent surgery, or massive transfusion) that causes activation
of the pulmonary endothelium. This then leads to the sequestration of primed
neutrophils to the activated pulmonary endothelium. The second event is the infusion
of donor derived anti-HLA or anti-HNA antibodies directed against antigens on the
neutrophil surface and/or biological response modifiers (e.g., lipids) in the stored
blood component that activate these adherent, functionally hyperactive neutrophils,
causing neutrophil-mediated endothelial damage and capillary leak. Many studies in
the literature support this hypothesis which may explain how some TRALI reactions
occur in the absence of donor HLA/HNA antibodies, or why TRALI reactions do not
occur in all recipients of blood components from donors who are known to have these
antibodies.
Yet a third hypothesis suggests that high levels of donor derived vascular endothelial
growth factor (VEGF) or antibodies to class II HLA antigens residing on pulmonary
vascular endothelium may directly cause endothelial shape change and fenestration.
This theory purports to explain the syndrome in neutropenic patients.
Identifying TRALI
It is imperative that medical personnel and hospitals identify suspected cases of TRALI
and report them to Canadian Blood Services. The reporting of TRALI allows us to
better understand the true incidence of this reaction, in addition to its clinical course
and associated mortality. Further, by identifying cases of TRALI , CBS can take steps to
prevent further cases of TRALI by removing companion components of units that may
have caused the reaction and by investigating donors involved in these cases and
deferring them from further donations if they are found to be implicated as outlined
below.

Definition
Canadian Blood Services has adopted the definition put forth by the Canadian
Consensus Conference Panel on TRALI as outlined below. This definition is applied
consistently to all cases of TRALI reported to CBS and is used to determine whether
reported cases will be investigated.
Table 1: Canadian Consensus Conference Panel TRALI definitions
Term Definition
TRALI Acute lung injury (defined below) occurring within 6 hours of completion of
transfusion of blood component.
No pre-existing acute lung injury.
No other temporally associated risk factors for acute lung injury (see below).
Possible TRALI Acute lung injury (defined below) occurring within 6 hours of
completion of transfusion of blood components. No pre-existing acute lung injury. One
of more temporally associate risk factors for acute lung injury.
Table 2: Definition of Acute Lung Injury (ALI)
Term Definition
Acute Lung Injury New onset
Hypoxemia SpO2 <90% or Pa02/Fi02 < 300 mm Hg on room air, or other clinical
evidence of hypoxemia
Bilateral infiltrates on frontal chest X-ray
Because the diagnosis of Acute Lung Injury (ALI) can be difficult, it is important for
the transfusion service medical director and the patients physician to communicate
to determine, in particular, whether a patient has evidence of volume overload.
Although ALI and hydrostatic pulmonary edema may coexist, the latter is a more
common complication of transfusion and must be excluded in order for a diagnosis of
TRALI or possible TRALI to be made.
Prevention
Hospitals
It is unlikely that TRALI can ever be entirely prevented, but its frequency may be
reduced by the judicious use of blood components only for indications that are
justified based on sound medical evidence. Hospitals should have procedures in place
(e.g. blood utilization guidelines, blood conservation programs) which minimize
unnecessary transfusions. In addition, hospital medical staff must continue to have a
high index of suspicion in order to diagnose TRALI appropriately. All cases of TRALI or
possible TRALI should be reported to Canadian Blood Services (in addition to the
Provincial/Territorial Surveillance office as part of the TTISS program) who in turn is
required to report all possible TRALI or possible TRALI cases to Health Canada.

Reporting
Cases of suspected TRALI should be reported to CBS using standard procedure for
reporting Adverse Reactions. In order to correctly and consistently diagnose TRALI,
the following information must be included in the report:
Timing of transfusion with respect to symptom onset
Presence of other risk factors for acute lung injury (see Table 3)
CXR findings
Evidence of hypoxia: PaO2 or SaO2
Clinical indicators of volume status such as clinical evaluation, response to diuretics
(if given), or where available JVP, PCWP, CVP, echocardiogram report etc.
In addition to the standard procedure for reporting adverse event, and in order to
fully investigate and identify TRALI cases; patient samples must be sent for further
testing.
Blood Supplier
Recognizing that donor derived antibodies may be one of the causes of TRALI, CBS has
adopted a standardized, national donor management strategy in an attempt to
improve the safety of the blood supply for Canadians by reducing the risk of TRALI.
Secondary Prevention
Secondary prevention refers to the management of donors whose donations have been
temporally associated with a TRALI or possible TRALI reaction. In order to evaluate
such donors and their continued eligibility to donate, the following definitions apply:
Table 4: Definition of Donors Temporally linked with TRALI or possible TRALI
Term Definition
Implicated Donor A donor is implicated in TRALI if they have demonstrated antibodies
to an HLA Class I or II antigen or HNA; the antibody must have specificity for an
antigen present on the recipients WBCs or there must be a positive reaction noted
between donor serum and recipient WBCs, a positive crossmatch.
Associated Donor A donor is associated with a TRALI reaction if a blood component
was transfused during the six hours preceding the first clinical manifestation of
TRALI.
http://www.transfusionmedicine.ca/articles/transfusion-related-acute-lunginjury-trali

120) Best product to be given in multiple clotting factor deficiency and active
bleeding
a) FFP

b) Whole blood
c) Packed RBC
d) Cryoprecipitate
A

Following here is a brief, though at times technical, commentary and definition set
of common products made from donated Blood. There are others, however, these
are the ones that are most often are needed in the United States.
Whole Blood Products
Whole Blood

Red Blood Cell Components


Red Blood Cells (RBCs)
Washed Red Blood Cells
Leukoreduced Red Blood Cells
Pediatric/Divided RBC Units

Platelets
Platelets

Granulocytes
Granulocytes (Neutrophils)

Fresh Frozen Plasma


Fresh Frozen Plasma (FFP)

Cryoprecipitate
Cryoprecipitate (CRYO)

Factor Concentrates
Factor VIII Concentrates
Factor IX Concentrates
Antithrombin III

CMV Negative, Irradiated, and Leukoreduced Preparations


CMV Negative Blood and Components
Irradiated Blood and Components
Leukoreduced Blood and Components

Useful Links to Quality Information


Blood Components - Details from the University of Michigan
Summary
Short Summary Chart w/Shelf Life

Whole Blood Blood Products


Description - The product of one unit of donated and unadulterated Blood plus
ACD (anticoagulant/preservative). By definition, whole Blood contains one unit of

plasma and cells. Whole Blood can be stored, normally and conventionally, for 5
weeks. Factors V and VIII are labile and are significantly decreased after 7 days.
Indication - If "fresh" (less than 24 hours since drawn,) whole Blood is still
utilized in resuscitation of a patient who has been loosing a lot of Blood. Whole
Blood is not used for "routine" Blood transfusion when red cells (RBC) will
suffice. Since one unit of donated Blood can be broken down into one unit RBC,
one unit platelets, and one unit fresh frozen plasma (FFP), and more, the use of
whole Blood is considered to be a waste of resources.
Red Blood Cells (RBCs)
Description - One unit of red Blood cells (RBC) contains approximately 180ml
(range 150 to 210 ml) of red cells, 100ml of Optisol, and approximately 30ml
(range 10 to 50 ml) of plasma. As an average, in the United States, the total volume
of a RBC unit is 310 ml (range 270 to 350 ml). Blood Products
A unit of RBC is prepared from a whole Blood collection using a closed sterile
system. Blood is drawn into a bag containing the anticoagulant CPD. Most of the
platelet rich plasma is separated with a centrifuge and separated into an attached
container. 100 ml of an additive nutritive solution (Optisol) is added to RBC.
Optisol is a crystalloid solution containing sodium, dextrose, adenine and
mannitol. The Optisol supports red cell survival and extends the shelf life of the
unit to 42 days. The added fluid volume of the Optisol also reduces the unit's
hematocrit to ~57% (range 50 to 65%), thereby improving the flow characteristics
of the component. Optisol is also known as AS-5.
All RBC transfusions must be ABO/Rh compatible with the recipient. Packed red
Blood cells do not provide viable platelets or neutrophils, nor do they provide
clinically significant amounts of coagulation factors. RBC must be stored between
1 to 6C.
Indication - Red Blood cells are indicated for patients with symptomatic anemia
that is not treatable with specific therapy such as iron, vitamin B12 or with folic
acid.
Therapeutic Effect - In a 155-pound adult, one unit of RBCs can be expected to
increase the hematocrit by approximately 3% or the hemoglobin by 1 gm/dl.
Washed Red Blood Cells Blood Products
Description - Washed red Blood cells are red Blood cells washed with normal
saline to remove most of the plasma. Washed red Blood cells should not be
considered leukoreduced. Because the bag must be entered to introduce the saline,
washed red cells must be given within 24 hours of their preparation.
Indication - Washed red cells can be considered for patients who have had
repeated hypersensitivity reactions to Blood or components despite prophylactic

administration of antihistamines. It should be kept in mind, however, that the red


cell washing procedure may not reduce the proteins enough to prevent
hypersensitivity reactions (e.g. hypersensitivity to IgA). Controversial indicators
for washed red Blood cells include complement mediated immune hemolysis and
paroxysmal nocturnal hemoglobinuria. Blood Products
Therapeutic Effect - A unit of washed red Blood cells will raise the hematocrit
less than will a unit of red Blood cells because of an approximate 20% loss of red
cells from the unit during the washing process.
Leukoreduced Red Blood Cells
Description - Leukoreduced red Blood cell units contain leukocytes in a
specifically reduced amount. In the United States, Blood processing centers use
filtration to make leukoreduced red Blood cell units. Blood Products
Indication - The most common indication for leukoreduced red Blood cells is for
patients who have experienced two or more non-hemolytic febrile transfusion
reactions. Leukoreduced red cells are usually effective in preventing nonhemolytic febrile transfusion reactions for most patients.
Leukoreduced red Blood cells are also effective in prevention of CMV
transmission or HLA alloimmunization. Blood Products
Therapeutic Effect - Leukoreduced red Blood cells will have a slightly lower
therapeutic effect than red cells that have not been leukoreduced. Depending on the
filter used, there is a 10 to 15% loss of red cells with leukoreduction by filtration.
Pediatric/Divided RBC Units
Description - Pediatric/Divided red Blood cell units are prepared by separating a
CPD anticoagulated (containing no Optisol) packed red Blood cell unit into four
bags. Each pedi-pack contains approximately 45 to 50 ml of red Blood cells and
approximately 15 ml of plasma. This processing minimizes wasting Blood when
only small volume transfusion is required. In addition, it may reduce the recipient's
donor exposure because four units for transfusion are available from one unit of
donated Blood. Divided red cell units are issued when they are less than six days
old. This helps ensure adequate amounts of 2,3 DPG for optimal delivery of
oxygen to the tissues and relatively low plasma potassium levels when stored a
shorter period of time. All divided units are routinely prepared from units which
have been screened and found negative for HgbS. All divided units are irradiated.
They may or may not be serologically negative for CMV, but all divided units are
leukoreduced. Studies indicate leukoreduction by filtration is equivalent to CMV
screened negative for prevention of CMV transmission by transfusion.
Because of the small number of these Blood products (components) requested and
for inventory management purposes, divided RBC units are only available as type

O. These units may be transfused to any ABO type recipient in any clinical
circumstance, including intrauterine transfusion when the fetus ABO type may be
unknown, as well as for exchange transfusion in neonates when fetal-maternal
ABO incompatibility may be involved. Blood Products
Divided red Blood cells units have a higher hematocrit, approximately 72%, higher
than standard Optisol preserved RBC units. Unprotected and unprepared divided
Blood units expire 21 days after collection. Blood Products
Indication - Divided red Blood cell units are indicated for infants who require
small amounts of red cells. Blood Products
Therapeutic Effect - A divided red Blood cell unit will increase the
hematocrit/hemoglobin the same as a standard red Blood cell unit when corrected
for the weight of the child and the volume infused.
Platelets
Description - Platelets are cells essential for the coagulation of Blood. Platelet
products also contain plasma (coagulation factors), some red cells and some white
cells (leukocytes). Platelet products are usually cloudy and yellowish in color but
may occasionally have a pink tone because of the presence of residual red cells.
Platelets are stored at 68 to 75 Fahrenheit (room temperature) and require
continuous gentle agitation. They can be stored at the Blood center for up to five
days. When received for transfusion, both pooled and apheresis platelets will
expire in less than four hours. Since preparation for transfusion involves processes
such as pooling, volume reduction and leukoreduction which require entry into the
component, a four hour expiration is placed on the platelets once preparation is
started at the Blood center to avoid bacterial growth.
A Whole Blood Platelet Concentrate is prepared from whole Blood by an initial
soft centrifugation to separate the red cells from the platelet rich plasma. A second
harder centrifugation is used to concentrate the platelets that are then resuspended
in 60 ml of residual plasma. Each unit contains a specific ratio/quantity of platelets.
To provide an adequate dose of platelets for an adult, four to six platelet
concentrates of the same Blood type are pooled at the Blood center prior to issue.
Pooled platelets are generally issued ABO type compatible, but other types may be
substituted. One should avoid, if possible, giving type A platelets to an O recipient.
If the O recipient happens to have a high titer of anti-A, the post transfusion
platelet increment will be reduced. Platelets products contain an insufficient
number of red cells to cause an incompatibility reaction. There are sufficient
numbers of red cells, however, for an Rh negative person to be sensitized (develop
Rh antibodies) if they receive Rh positive Blood. There is very little risk of the
patient having an incompatibility reaction because the plasma in a pooled unit is
combined from different donors thereby reducing the possibility that isoagglutinins

(anti-A and/or anti-B) would be present in high titer. Due to the smaller Blood
volumes of infants and small children, ABO compatible or reduced volume ABO
incompatible platelets must be given.
Apheresis Platelets are obtained from one donor with the use of an apheresis
machine. Blood is drawn from a donors arm into a self contained, single use
Blood tubing/collection set which has been inserted into the apheresis machine.
Blood does not come into contact with the apheresis machine itself. Anticoagulant
is added to the Blood as it is drawn from the donor. The platelets are separated
from the red cells, leukocytes and most of the plasma by centrifugation. The red
cells, leukocytes and plasma are returned to the donor through his or her other arm,
and the platelets are retained in a collection bag for later transfusion to a patient.
The procedure takes approximately 60 to 90 minutes.
The majority of apheresis platelets collected contain less than a specific amount of
leukocytes and are labeled as leukocyte reduced.
One apheresis collection of platelets generally contains 200 to 400 ml of plasma.
Because of the possibility of a high titer of ABO antibodies in the donor plasma,
the unit is volume reduced in cases of minor ABO incompatibility. Apheresis
platelet concentrates can be collected from unselected community donors. This
yields a product known as a Random Apheresis Platelet (RAP). Alternately the
platelets may be drawn from a family or community donor who has been
specifically matched to the patient on the basis of HLA (Human Lymphocyte
Antigen) typing. This yields a product known asA MATCHED Apheresis Platelet
(MAP).
Random Apheresis Platelets are available in two doses: Standard and Large. The
standard dose contains a smaller average count of platelets (approximately
equivalent to four units of pooled platelets). The standard dose is generally ordered
for smaller patients, for those in whom a high platelet count is not required, and for
patients who respond well to transfusion. The large dose contains, as the name
would indicate, a greater average number of platelets, approximately equivalent to
six units of pooled platelets. The large dose is generally ordered for larger and
heavier patients, for those in whom a high platelet count is desired, and for those
who do not respond well to transfusion.
As many platelets as possible are collected from HLA matched apheresis donors,
therefore it is not necessary to specify dose when ordering these platelets.
Indications - Platelet transfusions are indicated for patients with bleeding due to
either thrombocytopenia, platelet dysfunction or some combination of the two
conditions. The point at which bleeding may occur varies depending on the
patients condition. The majority of patients with normal platelet function will not
experience bleeding until the platelet count drops below a certain point. In patients
with abnormal platelet function, usually caused by drugs (e.g. aspirin or semi-

synthetic penicillin), uremia or elevated split products of fibrinogen/fibrin,


bleeding may occur with higher platelet counts. In patients undergoing surgery,
bleeding may occur with relatively low platelet counts. Blood Products
In addition to evaluating platelet count and patient condition, bleeding time may
also be used in determining the need for platelet transfusions. A bleeding time
twice the upper normal limit may be an indication for a platelet transfusion in a
bleeding patient. HLA Matched platelets are indicated for patients who are
refractory (demonstrate a poor post-transfusion platelets increment) to random
donor platelets due to alloimmunization.
Patients with auto-immune thrombocytopenic purpura (ITP) should not receive
platelet transfusions unless bleeding is significant or life threatening. Platelet
transfusions given to patients with ITP will be rapidly removed from circulation by
the patients anti-platelet antibodies and thus will be, at most, only of transient
benefit.
Therapeutic Effect - Each unit of platelets prepared from donated whole Blood
contains a certain number of platelets and can be expected to increase the platelet
count of a 155-pound patient by a known approximate amount by one hour after
transfusion. Since the usual dose for adults with platelet related bleeding is a pool
of four to six units of platelet concentrates from whole Blood or one standard sized
unit of apheresis platelets, an increase in the platelet count by one hour after
transfusion is expected. Blood Products
Effect of Platelet Product and Patient Weight on Platelet Increment*
Standard apheresis Large apheresis
Single whole
Patient weight
of four pooled
or
Blood platelet
(in pounds)
whole
six pooled whole
concentrate
Blood platelets
Blood platelets
50
17,600
70,400
105,600
100
8,800
35,200
52,800
150
5,900
23,500
35,200
200
4,400
17,600
26,400
*Data is given as one hour post transfusion platelet increment
Patients demonstrating two consecutive platelet count increases of less than a
known standard range at one hour after transfusion of four to six units of pooled
platelets (or one unit of apheresis platelets) are considered refractory. Failure to
achieve hemostasis or the expected increment in the platelet count may signify a
refractory state. A refractory state to platelets may be caused by fever, sepsis, DIC,
or splenomegaly or an immune response to the platelets also referred to as platelet

alloimmunization. In patients with alloimmunization, HLA matched platelets may


be necessary to control bleeding due to thrombocytopenia.
Granulocytes (Neutrophils)
Description - Granulocytes (neutrophils) are obtained by an apheresis procedure
from an ABO-Rh compatible donor. Since there are large numbers of red cells in
granulocyte concentrates, compatibility testing must be performed between the
donor unit and the recipient. Granulocyte concentrates from donors who have been
stimulated by G-CSF, a practice currently under study, contain much larger
numbers of leukocytes. Granulocyte concentrates are always irradiated to prevent
graft versus host disease. Granulocytes should be administered as soon as possible
after collection. If this is not possible, storage should be at room temperature for no
longer than 24 hours after collection. A four hour expiration time is placed on the
granulocytes once issued from the Blood center. Blood Products
Indications - Granulocytes should be considered for patients with severe
neutropenia (<200/l) and a documented life-threatening bacterial or fungal
infection not responsive to appropriate antibiotic therapy. Additional indications
include neonates with clinical sepsis and patients with infections who have
neutrophil function defects. After granulocyte therapy is initiated, it is generally
continued once daily until the infection clears or the neutrophil count begins to
recover.
Therapeutic Effect - Even though there may be a clinical effect, there may not be
an increase in the recipients neutrophil count.
Fresh Frozen Plasma (FFP)
Description - Fresh frozen plasma (FFP) is the plasma removed from a unit of
whole Blood and frozen at or below 55 Fahrenheit within eight hours of
collection. FFP contains all coagulation factors in normal amounts and is free of
red cells, leukocytes and platelets. It is not a concentrate of clotting factors. One
unit is approximately 225 ml and must be ABO compatible with the recipients red
cells, Rh need not be considered.
Indications - FFP is indicated for patients with documented coagulation factor
deficiencies who are actively bleeding or who are about to undergo an invasive
procedure. Causes of such deficiencies include congenital deficiency, liver
disease, anticoagulation with warfarin or massive transfusion with red cells and
crystalloid/colloid solutions. Factor deficiencies severe enough to be clinically
significant are usually associated with prolongation of the coagulation screening
tests (prothrombin time, partial thromboplastin time) at least 1.5 times the
control value or an INR of 1.6. FFP is also indicated in treatment of thrombotic
thrombocytopenic purpura (TTP), usually in conjunction with plasma exchange.

FFP should not be used for volume expansion or nutritional support. Immune
globulin preparations are available for the provision of immune proteins instead of
FFP. Reversal of warfarin anticoagulation should be accomplished with Vitamin K
rather than FFP if two to three days can be allowed for clotting factors to return to
hemostatic levels. Massively bleeding patients may be given FFP along with red
Blood cells to prevent dilution of clotting proteins. Blood Products
Therapeutic Effect - One ml of FFP per 2.2 pounds of patient weight will raise
most clotting factors by approximately 1%. FFP should be used as soon as possible
after it is thawed and always within 24 hours after thawing. The amount of FFP
needed depends on the patients clotting factor levels, levels needed to achieve a
therapeutic effect, whether or not the patient is bleeding and the patients Blood
volume. Clotting factor activity should be estimated by specific coagulation factor
assays, or in emergencies, at least by coagulation screening tests.
Cryoprecipitate (CRYO)
Description - Cryoprecipitate (Cryo) is a low purity concentrate of three
hemostatic proteins prepared from donated whole Blood. A single bag of Cryo
contains an average of 100 units of factor VIII and von Willebrand factor and 150
to 250 mg of fibrinogen with some factor XIII and fibronectin. No compatibility
testing is required and ABO-Rh type is not relevant. However, due to their small
Blood volumes, children less than one year of age should be given ABO
compatible Cryo in case trace amounts of anti-A or anti-B are present. When Cryo
is ordered, units are thawed, suspended in sterile normal saline (20ml/bag) and
pooled. Once pooled, Cryo should not be chilled or refrigerated as the protein will
re-precipitate. The volume of a dose of Cryo depends upon the number of units
pooled. For young children who cannot tolerate a large volume or to increase the
fibrinogen concentration for fibrin glue preparation, Cryo can be suspended in 10
ml of saline per bag (reduced-volume Cryo). Cryoprecipitate is the only fibrinogen
concentrate available for intravenous use.
Indication - Cryoprecipitate is indicated for bleeding or imminent invasive
procedures for patients with significant hypofibrinogenemia (<100 mg/dl).
Commercial Clotting Factor Concentrates made with viral inactivation methods are
preferred over Cryo for hemophilia A and von Willebrand treatment.
The use of cryoprecipitate for the preparation of fibrin glue is increasing as
applications in neurosurgery, orthopedic and ENT surgeries are expanding.
Autologous units can be collected ahead of time and processed into Cryo to be
used for fibrin glue. Blood Products
Therapeutic Effect - When used for fibrinogen replacement, ten bags should
provide enough fibrinogen to raise the fibrinogen 60 to 70 mg/dl in a 155 pound

adult. Therapeutic effect can be monitored by fibrinogen levels and the patients
clinical response.
Note: Cryoprecipitate transfusions may be prepared from a designated donor for
some young or mildly affected patients with hemophilia A or von Willebrand
disease to limit potential viral exposure through transfusions. These single donor
Cryo products may have higher concentrations of factor VIII and von Willebrand
factor than regular donor Cryo because of DDAVP used to stimulate the apheresis
donor prior to collection. Multiple bags of high potency Cryo are then prepared
from one collection. Blood Products
A single unit (bag) of Cryo is usually adequate for the preparation of fibrin glue
unless more than 10 ml is needed.
Factor VIII Concentrates
Description - Factor VIII concentrates are a commercially prepared, lyophilized
powder purified from human plasma to treat patients with hemophilia A or von
Willebrands disease. Alternatively, recombinant (synthetic) protein is purified
from genetically engineered non-human cells grown in tissue culture. The quantity
of factor VIII coagulant activity is stated on the bottle. One factor VIII concentrate
unit equals the clotting activity in 1 ml of fresh plasma. Factor VIII concentrate is
cell free and is administered without regard to patient or donor ABO or Rh type. It
is heat treated and/or solvent detergent treated to reduce the risk of virus
transmission. Current processes appear to have eliminated the risk of HIV, HBV
and HCV transmission. concentrates differ in the purification procedures. Highly
purified factor VIII, e.g., preparations purified over a monoclonal antibody column
or current recombinant factor VIII concentrates, are stabilized by adding 98% of
pasteurized human albumin. Porcine factor VIII concentrate is available for
patients with high titer anti-human factor VIII 'allo' or autoantibody inhibitors.
Factor VIII concentrates are stored refrigerated at 35 to 45 Fahrenheit for up to
two years from the date of manufacture (expiration date will be indicated on each
vial). Some preparations may be kept at room temperature for extended periods.
Once reconstituted, it should not be refrigerated. Factor VIII concentrate should be
infused within four hours of preparation to reduce the risk of bacterial growth.
Vials are usually shipped to a hospital pharmacy, Blood service or nursing unit and
mixed there prior to use. Many patients or families receive them directly for home
care. Blood Products
Indication - Factor VIII concentrate is indicated for the treatment of bleeding or
imminent invasive procedures in patients with hemophilia A, (congenital factor
VIII deficiency) and for patients with low titer factor VIII inhibitors. Regular
prophylactic doses are often used, as well as daily doses in some hemophilic
inhibitor patients to try to induce immune tolerance. Patients with von

Willebrands disease respond to one specific, pasteurized intermediate purity


concentrate in which that factor activity is relatively preserved.
Therapeutic Effect - Dosage is dependent on the nature of the injury, the degree
of factor deficiency, the weight of the patient and the presence and level or absence
of factor VIII inhibitors. The half life of circulating factor VIII is eight to twelve
hours, therefore transfusions may need to be repeated every 12 to 24 hours to
maintain hemostatic levels. Following surgery, it is necessary to maintain
hemostatic levels for up to two weeks to prevent delayed bleeding and promote
wound healing in the hemophilic patient.
Factor IX Concentrates
Description - Factor IX concentrates are a commercially prepared, lyophilized
powder purified from human plasma to treat patients with hemophilia B. Crude
preparations, previously referred to as prothrombin complex concentrates, contain
coagulation factors II (prothrombin), IX, X and variable amounts of factor VII and
carry a risk of thrombosis associated with administration. Purified factor IX
concentrates, available since early 1991, essentially contain only factor IX and thus
have eliminated the risk of thrombosis. Since both products are used primarily to
treat factor IX deficiencies, the amount of Factor IX contained in each product is
stated on the label. Factor IX concentrates are heat treated to reduce the risk of
disease transmission, particularly HIV and hepatitis, and the purified forms
currently manufactured are treated sufficiently to inactivate hepatitis viruses B and
C. Prior to reconstituting, factor IX concentrates may be refrigerated at 35 to 45
Fahrenheit until the expiration date indicated on each vial, but should not be
frozen.
Indication - Factor IX concentrates are indicated for patients with hemophilia B
(factor IX deficiency), also called Christmas Disease, who are requiring treatment
of bleeding or about to undergo invasive procedures. Low purity factor IX
concentrate may be used in treatment of hemophilia A patients with high titer
inhibitors. It may be of value to patients with congenital factors X or II
(prothrombin) deficiency. "Activated" concentrates are also used for some factor
VIII inhibitor patients.
Factor IX concentrates should not be used for patients with acquired combined
deficiency of factor(s) II, VII, IX and/or X as plasma or whole Blood provides
safer, more effective treatment in these patients. Purified factor IX concentrate is
considerably more expensive than crude factor IX concentrates and is generally
indicated only for hemophilia B patients who require repeated infusion and/or are
at greater risk for thrombosis. A recombinant (synthetic) product was FDA
approved in early 1997 and is recommended for hemophilia B patients with
minimal or no prior exposure to plasma derived products or FFP. Low purity factor

IX concentrates are contraindicated for patients with liver disease, as there has
been an increased risk of thrombosis seen with the use of these products in this
group of patients.
Therapeutic Effect - Dosage will depend on the patient factor level and the
circumstances making the transfusion necessary. Treatment for bleeding generally
requires every 12-hour or daily infusions until symptoms resolve. For major
surgery in hemophilia patients, purified concentrate is indicated for ten to fourteen
days (until sufficient wound healing has taken place to prevent re-occurrence of the
bleeding condition). In hemophilia A patients with inhibitors, undergoing surgery
or experiencing major bleeding episodes, the treatment may consist of infusions of
the crude concentrates every eight to twelve hours. Blood Products
Antithrombin III
Description - Antithrombin III concentrates are commercially purified from
human plasma pools and lyophilized. They are provided as a powder in one vial
and a diluent (e.g. sterile water) inA COMPANION vial. Viracidal treatment
appears to eliminate the risks of HIV or hepatitis B or C transmission. Prior to
reconstituting, antithrombin III concentrates should be refrigerated (35 to 45
Fahrenheit) until they expire, but should not be frozen.
Indication - Antithrombin III concentrates are approved for and indicated in
reducing an acute increased risk of venous thrombo-embolic disorders in patients
with symptomatic, congenital antithrombin III deficiency. This will usually be
prophylactic therapy to correct levels from half-normal to around 100% during
surgical procedures or periods of increased risk from immobility, etc. A potential
role in acquired thrombic disorders or DIC is being investigated but has not been
established.
Therapeutic Effect - Due to a short life, long term prophylaxis with antithrombin
III infusions is not feasible (versus use of oral anticoagulants). The dosage is as
indicated in the paper included in the package. This Blood product is an 'order
item' norrnally available, in most areas, within two working days. Blood Products
CMV Negative Blood Components
Description - CMV is a herpes virus that resides in the white Blood cells of
persons who have been infected with the virus. There is a high prevalence of CMV
positive persons worldwide. Most persons that are CMV positive have no history
of illness.
CMV transmission to susceptible patients is effectively prevented by use of either
CMV seronegative, a donor determined to be negative for antibody to CMV, or
Leukoreduced, containing less than a certain range of leukocytes.

Cryoprecipitate and Fresh Frozen Plasma are cell free and have not been
implicated in CMV transmission. Blood Products
Indications - CMV negative Blood products are indicated for patients in the
following categories, regardless of CMV status of the mother:
P
r
e
m
a
t
u
r
e
i
n
f
a
n
t
s
;
Infants under four weeks of age; and,
Patients requiring intrauterine transfusion.

CMV negative Blood products are indicated for CMV negative patients in the
following categories:
B
o
n
e
m
a
r
r
o
w

o
r
o
r
g
a
n
t
r
a
n
s
p
l
a
n
t
r
e
c
i
p
i
e
n
t
s
(
i
f
t
h
e
m
a
r
r

o
w
o
r
t
h
e
o
r
g
a
n
d
o
n
o
r
i
s
a
l
s
o
C
M
V
n
e
g
a
t
i
v
e
)

;
Potential candidates for transplant;
AIDS or HIV infected patients;
Patients who have congenital immune deficiency;
Patients undergoing splenectomy; and,
Pregnant women.

If CMV status is pending in these patients, CMV negative components are


indicated. CMV negative components are not considered necessary for patients
receiving chemotherapy. Blood Products
Therapeutic Effect - In patients with compromised immune systems, a CMV
infection could result in a serious complication. CMV negative or leukoreduced
Blood products reduce this hazard.
Irradiated Blood Products
Description - Irradiated Blood products are exposed to approximately 2500 rads
of Gamma radiation to destroy the lymphocytes ability to divide. Transfusionassociated graft-versus-host disease (TA-GVHD) has not been reported from
transfusion of cryoprecipitate or fresh frozen plasma (FFP), thus these components
do not require irradiation. Fresh plasma (not frozen) for transfusion should be
irradiated if the patient is at risk for TA-GVHD.
Indications - The following indications for irradiation apply to patients with the
listed diagnoses.
Absolute Indication:
B
o
n
e
m
a
r
r
o
w
t
r

a
n
s
p
l
a
n
t
(
B
M
T
)
r
e
c
i
p
i
e
n
t
s
(
a
l
l
o
g
e
n
e
i
c
,
a
u
t
o

l
o
g
o
u
s
)
;
BMT or stem cell donors if allogeneic transfusion must be given prior to
completing the harvest;
Cellular (T-cell) Immune Deficiency (congenital or acquired);
Intrauterine transfusion;
Transfusions from family members (any degree);
Directed donors (when not identified as family members versus friends);
HLA-matched platelet transfusions.

Appropriate Indication:
H
e
m
a
t
o
l
o
g
i
c
m
a
l
i
g
n
a
n
c

i
e
s
(
l
e
u
k
e
m
i
a
s
)
;
Hodgkins Disease;
Non-Hodgkins Lymphoma;
Neonatal exchange transfusion;
Premature infants; and,
Certain solid tumors (neuroblastoma, glioblastoma).

Irradiation not considered as indicated: Blood Products


A
I
D
S
;
Most solid tumors;
Non-myeloablative chemotherapy recipients;
Routine immunosuppressive drugs (such as prednisone);
Solid organ transplant recipients;
Aplastic anemia (except if BMT); and,
Humoral immunodeficiency.

Therapeutic Effect - Irradiation destroys the ability of transfused lymphocytes to


respond to host foreign antigens thereby preventing graft vs. host disease in
susceptible recipients. Patients with functional immune systems will destroy

foreign lymphocytes, making irradiation of Blood and Blood components


unnecessary.
Leukoreduced Blood Components
Description - Cellular Blood components that contain less than a known and
accepted range of leukocytes (white Blood cells) are considered leukocyte reduced.
The leukocyte content of Blood components can be reduced by filtration. With
platelet preparations, filtration results in the loss of 10 to 30% of the platelets.
Single donor platelets prepared with the most modern apheresis machines will
already contain a miniscule amount of leukocytes (LRS platelets), can be labeled
as leukocyte reduced, and do not require filtration. Cryoprecipitate and fresh frozen
plasma do not contain intact or viable leukocytes making leukoreduction
unnecessary.
Indications - Leukoreduced Blood and components are indicated:
F
o
r
p
a
t
i
e
n
t
s
w
h
o
h
a
v
e
e
x
p
e
r

i
e
n
c
e
d
t
w
o
o
r
m
o
r
e
n
o
n
h
e
m
o
l
y
t
i
c
f
e
b
r
i
l
e

t
r
a
n
s
f
u
s
i
o
n
r
e
a
c
t
i
o
n
s
;
As a method of preventing transfusion transmitted CMV; and,
As a method of preventing platelet alloimmunization in some cases

SUMMARY
BLOOD
COMPONENT

Whole
Blood(autologous or
directed donations)

CONTENTS

VOLUM SHEL
E
F
LIFE*
*
Red Blood cells (RBC); plasma. White 520 ml
Blood Cells (WBCs); platelets not
35 days
viable after 24 hr. Factors V; VIII
significantly decreased after 2 days.
4o C
Hct 35%. 450mL Blood; 63 Ml
CPDA-1 anticoagulant

RBC w/ appx. 25 mL of plasma; 100 340 mL


42 days
mL of saline; additive solution
4o C
(adenine, mannitol). Hct 60%
Platelet concentrate Platelets; includes some WBC; 50 mL 50 mL 5 days
of plasma, a few RBC (Hct less than .
20o C
005)
Platelet pheresis
Platelets; includes some WBC; 300
300 mL 5 days
mL of plasma; a few RBC
200 C
Fresh frozen plasma Plasma proteins, all coagulation
225 mL 1 year
factors, complement
18o C
Cryoprecipitate
150 mg of fibrinogen, at least 80 units 15 mL 1 year
of factor VIII, von Willebrand factor,
18o C
factor XIII, fibronectin
* Note 1 - these are not "packed" red cells. Packed red cells have a Hct of 7080%.
** Note 2 - these times do not consider cryo-freezing technologies.
Red cells (AS-1)

121) A girl comes with Sydenhams chorea and acute rheumatic fever is suspected.
Other major criteria of Rheumatic fever (arthritis,skin rashes, subcut nodules and
carditis) are absent. No evidence of sore throat?? Best investigation to prove
rheumatic etiology is
a) ASL S
b) ASL O
c) Throat culture?
d) ??
B
122) Best non invasive investigation to check for viable myocardium is
a) Echocardiogram
b) FDG-18 PET CT
c) MRI
d) Thallium - 201 - ???
B
PETWhilst there are a number of tracers which can beused with PET for viability
assessment, F-18 FDG hasbecome the one almost universally employed for
clinicalpurposes. The seminal report on the diagnostic utility ofFDG PET for viability
assessment in humans waspublished by the UCLA group in 1986.27 PET has

higherspatial resolution than SPECT though still insufficientfor adequate assessment of


tracer distribution across themyocardial wall. The most commonly employed technique is to perform resting blood flow imaging witheither N-13 ammonia or Rb-82 Cl
and compare this withmetabolic imaging of myocardial glucose uptake with F-18 FDG.
Less commonly, the FDG study may beperformed in isolation without a blood flow
study, with[50% relative uptake taken to indicate viability. Thistechnique optimally
requires use of a metabolic clamp-ing procedure to standardize regional glucose
utilization.This technique is based on the observation thatviable myocardium
preferentially switches from theusual free fatty acids to glucose as the preferred
energysubstrate. FDG uptake into the myocardium is depen-dent on the insulinsensitive glucose transporters. Thesecan be activated by either oral glucose loading
oradministration of insulin (together with some glucose toavoid hypoglycemia) prior to
tracer administration.FDG is effectively fixed in the myocardium
throughphosphorylation by hexokinase, with minimal reversetransport. It clears from
the blood such that images ofthe myocardium can be acquired from 45-minute
postinjection onward.Myocardial segments are assessed with PET asshowing: (1)
normal flow/metabolism (viable), (2) mild matched reduction in flow/metabolism
(subendocardialscar), (3) severe matched defect, (transmural scar), or(4) mismatch
(apparent reduction in resting flow butpreserved glucose utilization) (Figure 1). All
exceptpattern 3 indicate a degree of retained viability in themyocardium. The
presence of mismatch can predictfunctional recovery after revascularization even
whencontractile reserve appears exhausted on functionaltesting.28In regards to
ability of FDG PET to predictimprovement in LV function, positive predictive valuein
one study was 86%, negative predictive value was100% and accuracy was 90%.29Earlier
vs later revascularization following identi-fication of viable myocardium by FDG
imaging has beenshown to be more beneficial for patient outcome.30,31The extent of
viability in the LV needed to predictimprovement in mortality after revascularization
onFDG imaging varies in different studies between 7% ofthe LV,32 10%33, and 20%.34
However, this extentshould be seen rather as a continuum, where increasingextent
implies both increased risk and increased poten-tial for recovery.34,35Some issues
with PET imaging are: it is less widelyavailable than SPECT. Second, myocardial
extraction ofFDG can be reduced in diabetic patients, as glucoseentry into the
myocardium is dependent on the insulin-sensitive glucose transporters. This is
apparent particu-larly in some type I diabetics, where additional lateimaging after
insulin administration may be necessary toobtain diagnostic images.FDG imaging has
also been performed usingSPECT acquisition but this has not been widely adopteddue
to the increasing availability of PET tomographswith their higher resolution.The
expanding potential for both SPECT and PETimaging not only in identification of
myocardial viabil-ity but also in increasing our understanding of its naturein both
basic science and clinical correlates has beenrecently reviewed.36Prognostic Risk of
Radiation Exposure fromTracer Studies in Patients UndergoingViability TestingThe
potential theoretical hazard of exposure toionizing radiation from tracer studies vs
other testmodalities in medical diagnosis is a frequently raisedissue. It is worth
considering that existing data suggestlong lead times for the development of such

possibleradiation-related events in the order of decades.37 In thecohort of patients


with depressed LV function undergo-ing viability testing the 2-year cardiac survival
rate canbe very low, i.e., only a few years.
https://www.asnc.org/media/PDFs/CME%20Instruction%20&
%20Article.July.pdf
---------------------------------------SPECTSPECT tracers include thallium-201 and the Tc-99m-based tracers, with Tc-99m
sestamibi being the most widely used of these. Thallium retention in
myocardiumreflects the intracellular potassium pool and is dependenton the integrity
of the Na/K pump in the cell membrane. Tc-99m sestamibi binds to the mitochondrion
and reflects itsintegrity as a marker of viability. There exist a proliferationof protocols
and diagnostic criteria for these tracers inregards to viability assessment. This
complexity meansthere is no single standardization of criteria, which can be asource
of confusion to clinicians and frustration to inves-tigators including those wishing to
perform collaborativestudies or pooled analyses. These diagnostic criteria may
bebased on relative resting tracer uptake, uptake after nitrateadministration, late
redistribution or improvement after restreinjection (for thallium), or defect
reversibility betweenstress and rest. SPECT has low spatial resolution andcannot
assess transmurality of tracer distribution. On theother hand it is widely available.
There are a multitude ofpublished SPECT viability studies

123) Which of the following statements is not true regarding H1N1


a) Pregnant woman with sore throat can be treated without diagnostic test under
category B
b) People on long term steroids cant receive Oseltamivir
c) Category B concerns with
d) ??
B
Category-B: In addition to all the signs and symptomsmentioned under Category-A, if the patient
hashigh grade fever and severe sore throat, he/shemay REQUIRE home isolation and Oseltamivir;
In addition to all the signs and symptomsmentioned under Category-A, individuals havingone or
more of the following high risk conditionsshall be treated with Oseltamivir: Children less than 5
years old; Pregnant women; Persons aged 65 years or older; Patients with lung diseases, heart
disease, liver disease, kidney disease, blood disorders,diabetes, neurological disorders, cancer and
HIV/AIDS
Patients on long term cortisone therapy. No tests for H1N1 are REQUIRED for Category-B (i)and
(ii). All patients of Category-B (i) and (ii) should confinethemselves at home and avoid mixing with
publicand high risk members in the family.

Category-C: In addition to the above signs and symptoms ofCategory-A and B, if the patient has
one or more ofthe following: Breathlessness, chest pain, drowsiness, fall in blood pressure, sputum
mixed with blood, bluishdiscolouration of nails; Irritability among small children, refusal to accept
feed; Worsening of underlying chronic conditions. All these patients mentioned above in CategoryC REQUIREtesting, immediate hospitalization andtreatment including Oseltamivir

124) Hypertensive hemorrhage most commonly affects


a) Basal ganglion
b) Thalamus
c) Brain stem
d) Cerebrum
A
125) A man comes with aphasia. He is unable to name things and repetition is poor.
However comprehension, fluency and? Is unaffected. He is probably suffering from
a) Anomic aphasia
b) Transcortical sensory aphasia
c) Conduction aphasia
d) Broca aphasia
c
CONDUCTION APHASIA= Caused by damage to the arcuate fasciculus, the structure that transmits
information between Wernicke's area and Broca's area. Auditory comprehension is near normal, and
oral expression is fluent with occasional paraphasic errors. Repetition ability is poor

http://cirrie.buffalo.edu/encyclopedia/en/article/9/
Table:
Characteristic
Features of
Aphasia
Auditory
Type of Repetitio Namin
comprehensio Fluency
Presentation
aphasia
n
g
n
Wernicke's mildmod mild defective
fluent
Individuals with Wernicke's aphasia
aphasia
severe
paraphasic may speak in long sentences that
have no meaning, add unnecessary
words, and even create new "words"
(neologisms). For example,
someone with Wernicke's aphasia
may say, "You know that smoodle
pinkered and that I want to get him
round and take care of him like you

Transcortical
sensory
good
aphasia

Conduction
poor
aphasia

Anomic
aphasia

mild

Broca's
aphasia

mod
severe

want before", meaning "The dog


needs to go out so I will take him for
a walk". They have poor auditory
and reading comprehension, and
fluent, but nonsensical, oral and
written expression. Individuals with
Wernicke's aphasia usually have
great difficulty understanding the
speech of both themselves and
others and are therefore often
unaware of their mistakes. They are
also often unaware of their
surroundings, and may present a risk
to themselves and others around
them.
Similar deficits as in Wernicke's
mod
poor
fluent
aphasia, but repetition ability
severe
remains intact.
Caused by damage to the arcuate
fasciculus, the structure that
transmits information between
Wernicke's area and Broca's area.
poor
relatively good fluent
Auditory comprehension is near
normal, and oral expression is fluent
with occasional paraphasic errors.
Repetition ability is poor.
Anomic aphasia, is essentially a
difficulty with naming. The patient
may have difficulties naming certain
words, linked by their grammatical
type (e.g.difficulty naming verbs
and not nouns) or by
mod
their semanticcategory
mild
fluent
severe
(e.g. difficulty naming words
relating to photography but nothing
else) or a more general naming
difficulty. Patients tend to produce
grammatic, yet empty, speech.
Auditory comprehension tends to be
preserved.
mod mild difficulty non-fluent, Individuals with Broca's aphasia
severe
effortful, frequently speak short, meaningful
slow
phrases that are produced with great
effort. Broca's aphasia is thus
characterized as a nonfluent aphasia.

Transcortical
motor
good
aphasia

mild
mild
severe

Global
aphasia

poor

poor

Transcortical
mixed
moderate poor
aphasia
Subcortical
aphasias

poor

poor

Affected people often omit small


words such as "is", "and", and "the".
For example, a person with Broca's
aphasia may say, "Walk dog"
meaning, "I will take the dog for a
walk". The same sentence could also
mean "You take the dog for a walk",
or "The dog walked out of the yard",
depending on the circumstances.
Individuals with Broca's aphasia are
able to understand the speech of
others to varying degrees. Because
of this, they are often aware of their
difficulties and can become easily
frustrated by their speaking
problems. It is associated with right
hemiparesis, meaning that there will
be paralysis of the patient's right
arm, leg, and face.
Similar deficits as Broca's aphasia,
except repetition ability remains
intact. Auditory comprehension is
generally fine for simple
conversations, but declines rapidly
non-fluent
for more complex conversations. It
is associated with right hemiparesis,
meaning that there will be paralysis
of the patient's right arm, leg, and
face.
Individuals with global aphasia have
severe communication difficulties
and will be extremely limited in
their ability to speak or comprehend
language. They may be totally
non-fluent nonverbal, and/or only use facial
expressions and gestures to
communicate. It is associated with
right hemiparesis, meaning that
there will be paralysis of the
patient's right arm, leg, and face.
non-fluent

Similar deficits as in global aphasia,


but repetition ability remains intact.
Characteristics and symptoms
depend upon the site and size of

subcortical lesion. Possible sites of


lesions include the thalamus,
internal capsule, and basal ganglia.

126) A young lady with C/F of hyperthyroidism with elevated T4 and TSH= 8.5. Further
examination reveals bitemporal hemianopia. Next step of management
a) Start antithyroid drugs, And do urgent MRI brain
b) ??
c) ??
d) ??
A

127) AIIMS MAY 2015


Hyperdynamic state includes all except
a) Anemia
b) Beri beri
c) Cor Pulmonale
d) AV fistula
ANS=A
A controversial question
hyperdynamic circulation (e.g. febrile states, severe anaemia, thyrotoxicosis, chronic cor pulmonale
and others)
REF-Principles and Practice of Medicine a TB for Students and Practitioners
BYDR BIRENDRA PRASAD
Front Cover
Jaypee Brothers Publishers
----------------------------------------------------------------------Water hammer pulse is commonly found when a patient has aortic regurgitation. It can also be seen
in other conditions which are associated with a hyperdynamic circulation. A more comprehensive list
of causes follows:
1. Physiological
Fever
Pregnancy
2. Cardiac lesions
Aortic regurgitation
Patent ductus arteriosus
Systolic hypertension
Bradycardia
Aortopulmonary window

Aneurysm of sinus of Valsalva


3. Syndromes or High output states
Anemia
Cor pulmonale
Cirrhosis of liver
Beriberi
Thyrotoxicosis
Arteriovenous fistula
Paget's disease
4. Other causes
Chronic alcoholism

https://en.wikipedia.org/wiki/Watson%27s_water_hammer_pulse
------------------------------------------------------Hyperdynamic circulation is abnormally increased circulatory volume. Systemic vasodilation and the
associated decrease in peripheral vascular resistance results in decreased pulmonary capillary
wedge pressure and decreased blood pressure, often presenting with a collapsing pulse. In effort to
compensate the heart will increase cardiac output and heart rate, which accounts for the decreased
pulse pressure and sinus tachycardia.[1] The condition sometimes accompanies septic shock,
preeclampsia, and other physiological and psychiatric conditions.
Possible causes[edit]
Kidney disease
Hypervolemia
Anemia
Anxiety
Aortic Regurgitation[2]
AV fistulae
Beriberi
Dysautonomia
Erythroderma
Exercise
Liver failure
Hydrocephalus[3]
Hypercapnia
Paget's disease
Portal hypertension
Pregnancy
Pyrexia
Thyrotoxicosis
Vasodilator drugs

https://en.wikipedia.org/wiki/Hyperdynamic_circulation

128) Which of the following is not a part of Duke Criteria for infective endocarditis
a) Splenomegaly

b) Fever > ? 100.4 Celsius


c) IV drug user
d) Blood culture positive
a
Major criteria :
Positive blood culture for Infective Endocarditis
Typical microorganism consistent with IE from 2 separate blood cultures, as noted below:
viridans streptococci, Streptococcus bovis, or HACEK group (what's this?), or
community-acquired Staphylococcus aureus or enterococci, in the absence of a primary focus
or
Microorganisms consistent with IE from persistently positive blood cultures defined as:
2 positive cultures of blood samples drawn >12 hours apart, or
all of 3 or a majority of 4 separate cultures of blood (with first and last sample drawn 1 hour apart)
Evidence of endocardial involvement
Positive echocardiogram for IE defined as :
oscillating intracardiac mass on valve or supporting structures, in the path of regurgitant jets, or on
implanted material in the absence of an alternative anatomic explanation, or
abscess, or
new partial dehiscence of prosthetic valve
or
New valvular regurgitation (worsening or changing of preexisting murmur not sufficient)
Minor criteria :
Predisposition: predisposing heart condition or intravenous drug use
Fever: temperature > 38.0 C (100.4 F)
Vascular phenomena: major arterial emboli, septic pulmonary infarcts, mycotic aneurysm,
intracranial hemorrhage, conjunctival hemorrhages, and Janeway lesions
Immunologic phenomena: glomerulonephritis, Osler's nodes, Roth spots (what's this?), and
rheumatoid factor
Microbiological evidence: positive blood culture but does not meet a major criterion as noted above
or serological evidence of active infection with organism consistent with IE
Echocardiographic findings: consistent with IE but do not meet a major criterion as noted above
Excludes single positive cultures for coagulase-negative staphylococci, diphtheroids, and
organisms that do not commonly cause endocarditis.

http://www.medcalc.com/endocarditis.html

129) Which of the following cannot cause reduced DLCO


a) Emphysema
b) Asthma
c) ?Pulmonary thromboembolism
d) Pneumonia
c

Causes: Decreased DLCO


Restrictive Lung Disease
Idiopathic Pulmonary Fibrosis
Asbestosis
Sarcoidosis
Berylliosis
Hypersensitivity pneumonitis
Langerhans Cell Histiocytosis X
Miliary Tuberculosis
Silicosis (advanced stage)
Pneumonectomy
Bronchiolitis Obliterans with Organizing Pneumonia
Obstructive Lung Disease
Cystic Fibrosis
Emphysema
Silicosis (early stages)
Normal lung function tests
Chronic Pulmonary Embolism
Congestive Heart Failure
Interstitial Lung Disease prior to PFT restrictive lung changes
Primary Pulmonary Hypertension
Connective tissue disease with lung involvement
Dermatomyositis or Polymyositis
Inflammatory Bowel Disease
Rheumatoid Arthritis
Systemic Lupus Erythematosus
Systemic sclerosus
Wegener granulomatosis
V. Causes: Increased DLCO
Asthma
Left-to-right cardiac shunt
Polycythemia Vera (see below)
Pulmonary Hemorrhage
VI. Causes: Normal DLCO
Restrictive Lung Disease
Cystic Fibrosis
Emphysema
Silicosis (early stages)
Obstructive Lung Disease
Alpha-1-Antitrypsin Deficiency
Asthma
Bronchiectasis
Chronic Bronchitis

VII. Causes: Falsely Abnormal DLCO (if Hemoglobin not 15 g/dl)


See corrected DLCO for Hemoglobin Above
Falsely elevated DLCO
Polycythemia Vera
Early Congestive Heart Failure
Falsely reduced DLCO
Anemia

http://www.fpnotebook.com/lung/Lab/DfsngCpcty.htm

130) Lymphatic spread most commonly seen in which type of thyroid carcinoma
a) Papillary
b) Medullary
c) Follicular
d) Lymphoma
a
131) A young girl with a mass in lower abdomen involving umbilical and hypogastric
regions. On examination it is cystic and mobile and the examiner is unable to
insinuate fingers between the mass and the pelvic bone. Likely diagnosis
a) Duplication of intestine
b) Mesenteric cyst
c) Omental cyst
d) Ovarian cyst
ANS=D
If the mass is in the suprapubic region:Is it abdominal or pelvi-abdominal? Insinuate
your hands between the mass and the inguinal ligaments on either side and try to
pushyour hands medially so that they touch each other above the pubic bone (as in
testing for engagement of the foetal head in a pregnant woman):a) If your hands
touch each other, the mass ispurely abdominal.b) If you cannot make your hands
touch each otherand the mass lies in-between them, the mass ispelvi-abdominal
http://mmkenawi.ucoz.com/3/KENAWI-s_History_Taking_Clin_Exa

132) Mallory Weiss syndrome mainly affects which vessel


a) Left gastric artery
b) Right gastric artery
c) Phrenic vein
d) Coronary vein

A
133) Incision preferred for diaphragmatic surgery
a) Circumferential
b) Radial
c) Vertical
d) Transverse
a
134) Which is not true regarding the basis of functional division of Liver
a) Based on Portal Vein & Hepatic Vein
b) Divided into 8 segments
c) Three major & Three minor fissures
d) 4 sectors
a
The Couinaud classification (pronounced kwee-NO) is used to describe functionalliver anatomy. It
is the preferred anatomy classification system as it divides of the liver into eight independent
functional units (segments) rather than relying on the traditional morphological description based
on the external appearance of the liver. The segments are numbered I to VIII.
The separation of the segments is based on the fact that each has its own dual vascular inflow,
biliary drainage and lymphatic drainage. In general each segment is wedged shaped with the
apex directed towards the hepatic hilum (porta hepatis). At the apex of each segment a single
segmental branch of the portal vein, hepatic artery and bile duct enter; in the periphery of each
segment there is venous outflow through the hepatic veins, which run in 3 vertical planes that
separate the segments:
right hepatic vein located in the right intersegmental fissure, divides the right lobe into anterior
and posterior parts
middle hepatic vein lies in the main lobar fissure, divides the liver into right and left lobes (or
right and left hemiliver): this vertical plane runs from the inferior vena cava to the gallbladder
fossa and is known as Cantlie's line
left hepatic vein located in the left intersegmental fissure, divides the left lobe into a medial and
lateral parts
A fourth horizontal plane further divides the liver, known as the portal plane where theportal
vein bifurcates and becomes horizontal, dividing the liver into superior and inferior units.

Segments
segment I is

the caudate lobe situated posteriorly around the IVC and different to the other 7
segments. It may receive its supply from both the right and the left portal vein and is drained
directly into the IVC by one or more small hepatic veins
The remainder of the segments (II to VIII) are numbered in a clockwise fashion starting
superiorly in the left hemiliver :
segments II and III are lateral to the left hepatic vein and falciform ligamentwith II superior and
III inferior to the portal plane

segment IV lies

medial to the falciform ligament, between the left and middle hepatic veins. It is
subdivided into IVa (superior) and IVb (inferior) subsegments. Easy tip: IVa above and IVb below
the portal plane. Segment IV includes the quadrate lobe.
Segment V to VIII make up the right hemiliver and are easier to describe:
segment V is located below the portal plane between the middle and right hepatic veins
segment VI is located below the portal plane lateral to the right hepatic vein
segment VII is located above the portal plane lateral to the right hepatic vein
segment VIII is located above the portal plane between the middle and right hepatic veins
Each hepatic vein therefore drains multiple adjacent segments, those that are bounded by the
hepatic vein.
The division of the liver into self-contained units allows the surgical resection of individual
segments and sections (two or more adjacent segments) without damaging those segments
remaining. Hence for the liver to remain viable, resections occur along the hepatic veins and
portal veins in the planes that define the boundaries of these segments.

History and etymology


It was first described by C Couinaud in 1957. The notion of the Couinaud liver segments being
based on the arrondissements (administrative districts) of Paris is a radiological urban myth 4.
http://radiopaedia.org/articles/couinaud-classification

135) A pregnant lady was stabbed in the right side of the chest. She is yelling for help
on entering the casualty. On examination she has tachycardia and BP was 90/60 mm
Hg, breath sounds are decreased on the right side. What is the first consideration for
her
a) Intubate her
b) Establish iv line and start saline
c) Needle decompression in 2nd intercostal space
d) oropharungeal ariway

ANS=B>C
AS MENTION IN SCENARIO THE PT IS TALKING SO THE AIR WAY (A) IS PATENT
.BREATHING (B) IS EFFECT CAUSE SHE HAS DEC.BREATH SOUND SO POSSIBILITY
hemothorax and that's a life threatening situation that needs a prompt action.
Decompress
with
wide
bore
needle.
Once decompressed, insert an intercostal chest drain under water seal.

This pt has rt haemothorax he need chest tube but first thing is to stabilize the pt more ever she is pregnant so B is
more correct

136) 3yr old child comes with hydrocoele of the hernia sac. Management will include
a) Herniotomy

b) Herniorrhapy
c)
d) ??
A
DIFFICULTIES [s7]WITH INGUINAL HERNIAS IN CHILDREN If he has a HERNIA and a
HYDROCELE, gently draw the mass upwards, and dissect off its outer coverings down
to the hydrocoele sac, so as to expose it. Don't disturb his testis. Do a herniotomy for
the hernia. To do this you will have to divide his processus vaginalis. If you can find it,
tie it; if you cannot find it, this is not important.
If his SAC SPLITS up to and perhaps through his inguinal ring, this is inconvenient. Be
specially careful, as you search for something to sew together, that you don't tie his
vas.
If his TESTIS ATROPHIES later, you have probably interfered with its blood supply. This
is one of the commonest complications. His parents, who may have difficulty
accepting that one testis can function as efficiently as two, will not be pleased. His
other testis should however be normal, so reassure them.
If his hernia RECURS (unusual), try to refer him. If you are inexperienced, reoperation can be difficult.
http://www.meb.uni-bonn.de/dtc/primsurg/docbook/html/x4414.html

137) In a pt with thrombocytopenia, what is the target platelet count after platelet
transfusion to perform an invasive procedure?
a) 40,000
b) 50,000
c) 30,000
d) 60,000
B
138) In the case of esophageal cancer prognosis is best determined by
a) Age of patient
b) Cellular differentiation
c) Length of involvement
d) T stage

ans=D
139) MC and acceptable method of bariatric surgery is
a) Biliopancreatic division
b) Biliopancreatic division with ?ileostomy
c) Laparoscopic gastric banding
d) Roux en- Y procedure
D
Established Types of Bariatric Surgery
Gastric Bypass Surgery (Roux-en-Y)
gastric bypass surgery
Gastric bypass, also called Roux-en-Y or RNY for short, is the most commonly performed bariatric
surgery in the United States.
An entire section of our web site is dedicated to this popular and effective procedure, so well save
most of our discussion about it for those pages (link is below). The surgery is mainly restrictive in
nature, but it also has elements of malabsorption.
From a very high level perspective, to perform gastric bypass surgery the surgeon...
Cuts and staples the top portion of the stomach to create a small pouch at the end of the esophagus
Leaves the remainder of the stomach attached to the top of the small intestines
Goes further down the small intestine, cuts it, and attaches it to the pouch
Takes the end of the small intestine that is still connected with the non-pouch portion of the stomach
and attaches it to the bottom of the Roux limb. This allows the digestive juices produced by the
stomach to meet up with the food in the intestines.
---------------------------------------------------------------Adjustable Gastric Banding (Lap Band Surgery)
lap band surgery
This is the second most popular U.S. bariatric surgery procedure and is restrictive in nature. Its
been used in Europe since the 90s, but wasnt approved by the FDA in the States until 2001.
The lap band surgery procedure involves the sewing of a silicone and Silastic band around the top of
the stomach. A balloon around the inner surface of the band (imagine the inside of a bicycle tire) is
connected to a tube that leads to a half-dollar-sized port above the abdominal muscles but below the
skin.
During follow up visits, your doctor will add or remove saline solution (salt water) to make it tighter or
looser. The tighter it is, the less hungry you feel and vice-versa.

http://www.bariatric-surgery-source.com/types-of-bariatric-

140) A 60 yr old lady with family history of breast cancer comes with blood stained
discharge from nipple. Next best step
a) Ductoscopy
b) Nipple discharge cytology

c) Sono - mammogram
d) MRI

ANS=C
Until 2010 the consensus was that all women with PND should have mammography if they were
over the age of 35 years in order to identify ipsilateral lesions [11]. However, in the ASSOCIATION
of Breast Surgery's diagnostic guidelines document published in November 2010 the recommended
age for mammography was revised to patients over 40 years of age [12]. Ultrasound scan may be
helpful in visualising retro-areolar lesions that can potentially be biopsied. Other approaches used in
some centres in evaluating PND include ductal aspiration, ductography and ductoscopy. Our own
current standard practice, after triple assessment, in appropriate cases that present with PND is to
carry out diagnostic microdochectomy (DMD) if the presentation involves a single duct, or total duct
excision (TDE). If a concurrent mass is detected on clinical examination and/or imaging it would
REQUIRE tissue sampling as part of the triple diagnostic workup in order to establish the diagnosis

http://www.agialpress.com/journals/oajost/2014/101037/

141)
A lady who is 9 weeks pregnant comes with a 2.5cm mass in the upper and outer quadrant of left breast.
Ultrasound
failed
to
show
any
abnormality.
The
ideal
management
will
be
to
a)
Finger
guided
core
biopsy
b)
Aspirate
the
cyst
and
reassure
c)
Work
up
after
pregnancy
completed
d) Do mammography

ANS=A

The spectrum of breast cancer pathology is the


same as non-pregnant
DCIS
Locally Advanced
No increase in incidence of inflammatory cancer
Page 7
Diagnosis Of Breast Cancer in
Pregnancy and Lactation
Delayed diagnosis a major issue
Delay due to - changes in the breast being
attributed to the pregnancy
- reluctance/concern about
investigations
- breast changes decrease
sensitivity of assessment
Delay Minimised by Regular Breast
Examinations
Page 8
Diagnosis Of Breast Cancer in

Pregnancy and Lactation


Imaging - Ultrasound
Ultrasound is the imaging modality of choice
to assess a breast lump or localised change in
breast consistency
Pregnancy related changes make U/S less
sensitive so repeat imaging at a short interval
may be warranted
Taylor et al. 100% U/S abnormal (23/23)
Page 9
Page 10
Diagnosis Of Breast Cancer in
Pregnancy and Lactation
Imaging - Mammography
Low radiation dose to foetus (0.00004Gy)
Sensitivity of 63-78%
Helps assess DCIS related calcification
Screens other breast tissue (both sides)
Page 11
Diagnosis Of Breast Cancer in
Pregnancy and Lactation
Imaging - MRI
Technically difficult after 20 weeks as required
to lie face down
Increased blood flow to breasts will decrease
sensitivity and specificity
Gadolinium crosses placenta and associated
with rat foetal abnormalities, so alternate
contrast agent would be needed.
Page 12
Diagnosis Of Breast Cancer in
Pregnancy and Lactation - Biopsy
Fine Needle Cytology
- least invasive
- difficult interpretation
Core Biopsy
- more material
- better tissue characterisation
Open Biopsy
- indeterminate core biopsy
- inadequate needle target
- ? anaesthetic
http://www.sah.org.au/assets/files/PDFs/GP%20Conf%20Presentations/Breast%20Cancer%20in%20Pregnancy.pdf
Pregnancy-Associated Breast Cancer (PABC)
Breast cancer diagnosed during pregnancy or within the first year after delivery is referred to as pregnancyassociated breast cancer. It accounts for 12 % of all newly diagnosed breast cancers each year and affects one out
of every 3,000 live births. Typically presenting as a painless palpable lump, diagnosis is often delayed as physical
examination of the pregnant and lactating breast is challenging. Overall, PABC is biologically aggressive with the

majority of these tumours being oestrogen and progesterone receptor negative and Her2-neu receptor positive.
The biological nature of these tumours consequently bodes a poorer prognosis. During pregnancy, ultrasound
remains the optimal tool for imaging. However, if a suspicious finding is seen on the ultrasound in a pregnant
patient, mammography should be performed. Imaging findings on both mammography and ultrasound of PABC is
identical to cancer presenting in a non-pregnant or non-lactating woman. Ultrasound may reveal a circumscribed
to spiculated hypoechoic to complex mass with irregular shape, posterior shadowing and non-parallel orientation
(Fig. 14). Though cancers usually show posterior acoustic shadowing, in one study of 22 cases, posterior acoustic
enhancement was seen in 12 of the 22 patients, likely reflecting the necrotic/cystic degeneration seen in these
aggressive tumours [8]. Other associated findings include thickening of the Coopers ligaments, oedema, skin
thickening and axillary lymphadenopathy. Mammography permits evaluation of microcalcifications, multifocality
and multicentricity. Typically, a mass with or without associated pleomorphic calcifications is seen (Fig. 14).
Secondary findings might include architectural distortion, nipple or skin retraction and lymphadenopathy. MRI is
not typically done in pregnant patients, but can be safely performed in lactating patients. MRI is done for staging
and to evaluate extent of disease. MRI may reveal a circumscribed to irregular mass with type II or type III
enhancement kinetics. Other findings include non-mass-like enhancement and ductal enhancement
--------------------------------------------

http://www.ncbi.nlm.nih.gov/pmc/articles/PMC3781252/
A palpable mass in a patient younger than 30 years should be evaluated initially with ultrasonography,16 and
mammography is typically performedonly if sonographic and/or clinical features are suspicious for malignancy. If
the patient is 30 years or older, typically mammography is performed first, followed by ultrasonography.
Simple cysts are primarily asymptomatic, although they can be palpable as soft mobile masses when they are
large. Relaxed cysts, which are oval (flat), are usually asymptomatic. Tense, round cysts may require aspiration for
symptomatic relief. A clearly benign cyst recommended for aspiration because of symptoms is still appropriately
coded BI-RADS 2, benign1 and is not included as a positive imaging finding for auditing purposes (Edward A. Sickles,
MD, personal communication, 2009).
The fluid of simple cysts can be clear or cloudy and yellow or greenish black. Cysts that communicate with ducts
can produce nipple discharge. Greenish-black nipple discharge is typical of fibrocystic change communicating with
the nipple and does not require further evaluation. Clear or milky nipple discharge only with stimulation is usually
physiologic and does not require further evaluation.17 Spontaneous clear nipple discharge can be caused by
papillomas or cysts or rarely by cancer (with 6% malignancy rate in one series18) and merits further evaluation
with mammography and ultrasonography and rarely MR imaging or galactography. Bloody nipple discharge is
typically caused by benign or malignant papillary lesions, with a 24% rate of malignancy in one series

http://www.ncbi.nlm.nih.gov/pmc/articles/PMC3020576/
142) Most common shunt for hydrocephalus is
a) Ventriculoperitoneal
b) Ventriculopericardial
c) Ventriculopleural
d) Lumboperitoneal

a
143) Oncotype dx test is done to check the usefulness of the following in breast
cancer
A. Chemotherapy in ER -ve
B. Hormone therapy in ER +ve
C. Herceptin in HER2 +ve
D. ???
B
The Oncotype DX test is a genomic test that analyzes the activity of a group of genes that can affect how a cancer
is likely to behave and respond to treatment. The Oncotype DX is used in two ways:
to help doctors figure out a womans risk of early-stage, estrogen-receptor-positive breast cancer coming back
(recurrence), as well as how likely she is to benefit from chemotherapy after breast cancer surgery
to help doctors figure out a womans risk of DCIS (ductal carcinoma in situ) coming back (recurrence) and/or the
risk of a new invasive cancer developing in the same breast, as well as how likely she is to benefit from radiation
therapy after DCIS surgery
The Oncotype DX test is the only one of the four genomic tests for breast cancer (MammaPrint, Mammostrat, and
Prosigna are the others) with results that have been validated with much vigorous research.
While the results of the studies on the MammaPrint, Mammostrat, and Prosigna tests are promising, these three
tests aren't widely used to help make treatment decisions. Right now, the Oncotype DX test is the only genomic
test for early-stage breast cancer that is included in the National Comprehensive Cancer Center Network (NCCN)
and the American Society of Clinical Oncology (ASCO) treatment guidelines. Because the Oncotype DX test for DCIS
is relatively new, its not yet included in the ASCO or NCCN DCIS treatment guidelines.
The NCCN is an alliance of the world's leading cancer centers. These NCCN centers collaborate on research,
guidelines, and education to improve the care of people diagnosed with cancer. ASCO is a national organization of
oncologists and other cancer care providers. ASCO guidelines give doctors recommendations for treatments that
are supported by much credible research and experience.
Because strong results are available and the Oncotype DX test is included in the NCCN and ASCO treatment
guidelines, it is the only genomic breast cancer test widely used to make treatment decisions.
The results of the Oncotype DX test, combined with other features of the cancer, can help you make a more
informed decision about whether or not to have chemotherapy to treat early-stage hormone-receptor-positive
breast cancer or radiation therapy to treat DCIS.
Who is eligible for the Oncotype DX test?
You may be a candidate for the Oncotype DX test if:
youve recently been diagnosed with stage I or II invasive breast cancer
the cancer is estrogen-receptor-positive

there is no cancer in your lymph nodes (lymph node-negative breast cancer)


you and your doctor are making decisions about chemotherapy.
Most early-stage (stage I or II), estrogen-receptor-positive breast cancers that havent spread to the lymph nodes
are considered to be at low risk for recurrence. After surgery, hormonal therapies such as an aromatase inhibitor or
tamoxifen are prescribed to reduce the risk that the cancer will come back in the future. Whether or not
chemotherapy is also necessary has been an area of uncertainty for patients and their doctors.
If youve been diagnosed with early-stage, estrogen-receptor-positive breast cancer, the Oncotype DX test can help
you and your doctor make a more informed decision about whether or not you need chemotherapy. (Some research
also suggests the test may help postmenopausal women with estrogen-receptor-positive breast cancer that has
spread to the lymph nodes make chemotherapy decisions. Talk to your doctor if you are in this group.)
You also may be a candidate for the Oncotype DX test if:
youve recently been diagnosed with DCIS
youre having lumpectomy to remove the DCIS
DCIS is the most common form of non-invasive breast cancer. DCIS usually is treated by surgically removing the
cancer (lumpectomy in most cases). After surgery, hormonal therapy may be recommended if the DCIS is hormonereceptor-positive. Radiation therapy may be recommended for some women. Doctors arent always sure which
women will benefit from radiation therapy.
If youve been diagnosed with DCIS, the Oncotype DX test can help you and your doctor make a more informed
decision about whether or not you need radiation therapy.
How does Oncotype DX work?
The Oncotype DX genomic test analyzes the activity of 21 genes that can influence how likely a cancer is to grow
and respond to treatment.
Looking at these 21 genes can provide specific information on:
the likelihood that the breast cancer will return
whether youre likely to benefit from chemotherapy if youre being treated for early-stage invasive breast cancer
whether youre likely to benefit from radiation therapy if youre being treated for DCIS
So, the Oncotype DX test is both a prognostic test, since it provides more information about how likely (or
unlikely) the breast cancer is to come back, and a predictive test, since it predicts the likelihood of benefit from
chemotherapy or radiation therapy treatment. Studies have shown that Oncotype DX is useful for both purposes.
Oncotype DX test results assign a Recurrence Score a number between 0 and 100 to the early-stage breast
cancer or DCIS. You and your doctor can use the following ranges to interpret your results:
Recurrence Score lower than 18: The cancer or DCIS has a low risk of recurrence. The benefit of chemotherapy for
early-stage breast cancer or radiation therapy for DCIS is likely to be small and will not outweigh the risks of side
effects.
Recurrence Score between 18 and 31: The cancer or DCIS has an intermediate risk of recurrence. Its unclear

whether the benefits of chemotherapy for early-stage breast cancer or radiation therapy for DCIS outweigh the
risks of side effects.
Recurrence Score greater than 31: The cancer or DCIS has a high risk of recurrence, and the benefits of
chemotherapy for early-stage breast cancer or radiation therapy for DCIS are likely to be greater than the risks of
side effects.
You and your doctor will consider the Recurrence Score in combination with other factors, such as the size and
grade of the cancer, the number of hormone receptors the cancer cells have (many versus few), and your age.
Together you can make a decision about whether or not you should have chemotherapy or radiation therapy.

http://www.breastcancer.org/symptoms/testing/types/oncotype_dx
144) During laparoscopic inguinal hernia repair a tacker was place below and lateral
to the iliopubic tract. Postoperatively the patient complained of pain. This is due to
the involvement of
a) Lateral cutaneous N. of thigh
b) Ilioinguinal N.
c) Genitofemoral N.
d) Obturator N.
A
LATERAL FEMORAL CUTANEOUS NERVE
This nerve is the most commonly injured nerve during laparoscopic hernia repair.
Injury of this nerve may be normal with inexperienced surgeons performing
laparoscopic hernia repair.
Course
It arises from L2-3, emerges in the lateral fringe of the psoas muscle, courses along
the iliac fossa, lateral towards the iliac vessels. It passes below or with the inguinal
ligament, where it lies free in a fibrous tunnel 1cm to the medial side of the anteriorsuperior iliac spine. Broin et al detailed its course in cadavers and found it was a
mean distance of 6.6 cms from the Inferior Epigastric vessels and 5.6 cms in the
Internal Inguinal ring as it passes below the Iliopubic Tract.
Section of sensory supply to the upper lateral thigh. Common Site of damage is to
prevent nerve injury during, dissection and stapling ought to be over the Iliopubic
Tract.
Post-Injury Pain
Injury results in pain and numbness within the higher lateral thigh and is called
Meralgia Paraesthetica.

Sensory Changes
Injury results in pain and numbness within the upper lateral thigh and it is known as
Meralgia Paraesthetica
http://www.laparoscopyhospital.com/anatomy-laparoscopic-hernia.html

145) A middle aged man with RTA and bleeding from scalp.He is unconcious.A card in his pocket
reveals that he is known diabetic on glimepride+metformin,two tablets daily.What should be the next
step?
a)send blood for tests,start iv glucose and send pt for CT scan
b)start normal saline and send for CT
c)send blood for test,wait for report& if hypogglcemic start glucose
d)secure bleeeding first then send for CT scan
ans=D, Milton M. Madrigal,neurosurgeon wrote=D + Saline. We face a case of TBI in a Diabetic
patient. First ABCD - what means in this case secure the bleeding, then send to the CT scan... And
in the meantime start with normal saline... Why not? After that... and depending on the different
scenarios... then to treat all the aetiological factors.
D???.....Because in ATLS sequence is ABCD.......Circulation comes first...and securing hemostasis
should be the priority......Rest comesafterwards
Diabetic patients with traumatic brain injury: insulin deficiency is associated with increased mortality.
Ley EJ1, Srour MK, Clond MA, Barnajian M, Tillou A, Mirocha J, Salim A.
Author information
Abstract
BACKGROUND:
Hyperglycemia after traumatic brain injury (TBI) is an independent predictor of mortality. Insulin
deficiency, as opposed to elevated blood glucose, might be the reason for increased mortality. TBI
patients with diabetes mellitus (DM) were analyzed to determine how insulin deficiency affects
mortality after TBI.
METHODS:
NTDB version 7 was queried for patients with isolated moderate to severe TBI (head abbreviated
injury score [AIS]3 with AIS3 for other body regions). Demographics and outcomes were
compared between TBI patients with insulin-dependent DM (IDDM), noninsulin-dependent DM
(NIDDM), and those without DM. Logistic regression analysis was used to investigate the
relationship between mortality and DM.
RESULTS:
Overall, 51,585 patients with isolated moderate to severe TBI were analyzed. Mortality was 14.4%
and 8.2% in patients with and without DM, respectively (p<0.0001). Although head AIS scores were
similar, patients with DM had a statistically higher Glasgow coma scale (GCS) at presentation
compared with patients without DM (GCS score 12.4 vs. GCS score 10.9; p<0.0001). After
multivariable logistic regression analysis, DM was an independent predictor for mortality (odds ratio

1.5, confidence interval 1.29-1.74, p<0.0001). When comparing TBI patients with IDDM to NIDDM,
mortality was 17.1% for IDDM and 13.0% for NIDDM (p=0.025).
CONCLUSION:
DM is a significant predictor for mortality after moderate to severe TBI. Insulin deficiency is a likely
contributor to increased mortality after TBI as IDDM patients have higher mortality than NIDDM
patients who have higher mortality than no-DM patients.
------------------------------------------------------

http://www.ncbi.nlm.nih.gov/pubmed/21610428
146) According to the 2010 WHO criteria what are the characteristics of normal semen
analysis
a) Volume 1.5ml, count 15 million, morphology 4% progressive motility 32%
b) Volume 2.0ml, count 20 million, morphology 4% progressive motility 32%
c) Volume 1.5ml, count 20 million, morphology 4% progressive motility 32%
d) Volume 2.0ml, count 15 million, morphology 40% progressive motility 32%
A
In WHO 2010, the new normal values are based on data from men with proven fertility, men who
were known to help their partners conceive in the previous 12 months. Following a large analysis of
semen parameters from over 4000 men in 14 countries, a new set of 5th percentile parameters was
recommended. Below are the comparisons of the old and new reference values:
Parameter

WHO 1999

WHO 2010

Volume

2 ml

1.5 ml

Concentration

20 million/ml

15 million/ml

Progressive motility 50%

32%

Normal forms

4%

14%

http://ivfmd.net/new-world-health-semen-analysis-parameters/
147) What is the level of proteinuria to diagnose the lady to have severe
preeclampsia
a) 20mg
b) 200mg
c) 300mg
d) 2000 mg
d

148) Carbetocin dosage during labour is


a) Single 100 microgram dose over 1 minute
In 2009, the Society of Obstetricians and Gynecologists Canada, which produces
national clinical guidelines on important women's health issues, recommended that a
bolus of carbetocin 100 mcg into your vein should be used at elective cesarean
delivery instead of oxytocin infusion for the prevention of bleeding after you deliver
your baby. Similar to oxytocin, carbetocin has side effects that are dose-related.
Although 100 mcg has been the recommend dose, studies in nonlaboring women
suggest that doses lower than 100 mcg may be used to achieve the same degree of
uterine contractility with less side effects. So far, the ideal dose to be used in
cesarean sections for labouring women who have failure to progress in labour (failure
of your cervix to dilate adequately to 10cm or the baby's head not descending the
birth canal) has not been determined. This study is designed to determine the
minimum carbetocin dose required during cesarean delivery for 'failure to progress' to
achieve
the
best
effect.
https://clinicaltrials.gov/ct2/show/NCT01725243
----------------------------------------------------------------------------------------------------------------Can
J
Anaesth.
2015
Aug;62(8):866-874.
Epub
2015
Apr
10.
Carbetocin at Cesarean delivery for labour arrest: a sequential allocation trial to
determine
the
effective
dose.
Nguyen-Lu N1, Carvalho JC, Farine D, Seaward G, Ye XY, Balki M.
Author
information
Abstract
PURPOSE:
The aim of this study was to estimate the effective dose 90% (ED90) of carbetocin to
provide adequate uterine tone at Cesarean delivery (CD) for labour arrest.
METHODS:
We conducted a double-blind dose-finding study of carbetocin using a biased-coin upand-down design in women undergoing CD for labour arrest under epidural anesthesia.
Forty healthy term pregnant women who had received at least three hours of oxytocin
infusion during labour were recruited for the study. Carbetocin was administered
intravenously upon delivery of the anterior shoulder of the fetus. The first patient
received 20 g, and the dose for the subsequent patient was determined according to
the response of the previous patient as per the biased-coin allocation scheme using
increments or decrements of 20 g (maximum 140 g). Uterine tone was assessed by
the obstetrician and rated as satisfactory or unsatisfactory throughout the
intraoperative period. The primary outcome was satisfactory uterine tone with no
need for additional uterotonic drugs intraoperatively. Secondary outcomes included
use of additional uterotonic drugs postoperatively in the first 24 hr, estimated blood
loss,
and
adverse
effects.
RESULTS:
The ED90 of carbetocin to produce adequate uterine tone was estimated at 121 g
(95% confidence interval [CI]: 111 to 130; 99% CI: 108 to 133) using the truncated
Dixon and Mood (DM) method. The isotonic estimator of ED90 was 140 g; however,
the observed response rate across all doses was < 90%. Also, the 95% CI of the DM

estimator is likely to have lower than expected coverage, while the 99% CI may have
about 90% coverage. Therefore, these results should be interpreted with caution. The
overall median (range) estimated blood loss was 1,014 (104-2,436) mL. The overall
incidence of hypotension and tachycardia were 45% and 57.5%, respectively. At a dose
of 140 g, the incidence of tachycardia and intraoperative arrhythmias was 76% and
14%,
respectively.
CONCLUSION:
The ED90 of carbetocin at CD for labour arrest, as determined in our study, should be
interpreted with caution since it may be underestimated. This dose is higher than the
currently recommended dose of 100 g at elective CD and should not be used
routinely given the uncertainty regarding its efficacy and the high incidence of
arrhythmias at higher doses. This trial was registered at ClinicalTrials.gov, number:
NCT01725243.
http://www.ncbi.nlm.nih.gov/pubmed/25860126
Recommendations
Prevention of Postpartum Hemorrhage
1. Active management of the third stage of labour (AMTSL) reduces
the risk of PPH and should be offered and recommended to all
women. (I-A)
2. Oxytocin (10 IU), administered intramuscularly, is the preferred
medication and route for the prevention of PPH in low-risk vaginal
deliveries. Care providers should administer this medication after
delivery of the anterior shoulder. (I-A)
3. Intravenous infusion of oxytocin (20 to 40 IU in 1000 mL, 150 mL
per hour) is an acceptable alternative for AMTSL. (I-B)
4. An IV bolus of oxytocin, 5 to 10 IU (given over 1 to 2 minutes), can
be used for PPH prevention after vaginal birth but is not
recommended at this time with elective Caesarean section. (II-B)
5. Ergonovine can be used for prevention of PPH but may be
considered second choice to oxytocin owing to the greater risk of
maternal adverse effects and of the need for manual removal of a
retained placenta. Ergonovine is contraindicated in patients with
hypertension. (I-A)
6. Carbetocin, 100 g given as an IV bolus over 1 minute, should be
used instead of continuous oxytocin infusion in elective Caesarean
section for the prevention of PPH and to decrease the need for
therapeutic uterotonics. (I-B)
7. For women delivering vaginally with 1 risk factor for PPH,
carbetocin 100 g IM decreases the need for uterine massage to
prevent PPH when compared with continuous infusion of oxytocin. (I-B)
8. Ergonovine, 0.2 mg IM, and misoprostol, 600 to 800 g given by
the oral, sublingual, or rectal route, may be offered as alternatives
in vaginal deliveries when oxytocin is not available. (II-1B)
9. Whenever possible, delaying cord clamping by at least 60 seconds
is preferred to clamping earlier in premature newborns (< 37 weeks
gestation) since there is less intraventricular hemorrhage and less

need for transfusion in those with late clamping. (I-A)


10. For term newborns, the possible increased risk of neonatal
jaundice requiring phototherapy must be weighed against the
physiological benefit of greater hemoglobin and iron levels up to
6 months of age conferred by delayed cord clamping. (I-C)
11. There is no evidence that, in an uncomplicated delivery without
bleeding, interventions to accelerate delivery of the placenta
before the traditional 30 to 45 minutes will reduce the risk of PPH.
(II-2C)
12. Placental cord drainage cannot be recommended as a routine
practice since the evidence for a reduction in the duration of the
third stage of labour is limited to women who did not receive
oxytocin as part of the management of the third stage. There is no
evidence that this intervention prevents PPH. (II-1C)
13. Intraumbilical cord injection of misoprostol (800 g) or oxytocin
(10 to 30 IU) can be considered as an alternative intervention
before manual removal of the placenta. (II-2C)
Treatment of PPH
14. For blood loss estimation, clinicians should use clinical markers
(signs and symptoms) rather than a visual estimation. (III-B)
15. Management of ongoing PPH requires a multidisciplinary
approach that involves maintaining hemodynamic stability while
simultaneously identifying and treating the cause of blood loss. (III-C)
16. All obstetric units should have a regularly checked PPH
emergency equipment tray containing appropriate equipment. (II-2B)
17. Evidence for the benefit of recombinant activated factor VII has
been gathered from very few cases of massive PPH. Therefore
this agent cannot be recommended as part of routine practice. (II-3L)
18. Uterine tamponade can be an efficient and effective intervention to
temporarily control active PPH due to uterine atony that has not
responded to medical therapy. (III-L)
19. Surgical techniques such as ligation of the internal iliac artery,
compression sutures, and hysterectomy should be used for the
management of intractable PPH unresponsive to medical
therapy. (III-B)
Recommendations were quantified using the evaluation of
evidence guidelines developed by the Canadian Task Force on
Preventive Health Care (Table 1).
http://sogc.org/guidelines/active-management-of-the-third-stage-of-labourprevention-and-treatment-of-postpartum-hemorrhage/

149) Dose of dexamethasone to prevent RDS in newborn is

a) 12 mg 12 hrly 2 doses
b) 12 mg 24 hrly 2 doses
c) 6 mg 24 hrly 4 doses
d) 6 mg 12 hrly 4 doses
D
Guideline on the use of Antenatal Corticosteroids to PreventRespiratory Distress
Syndrome Preterm birth accounts for 10% of births in Malaysia and results in about
75% ofneonatal deaths not associated with congenital malformation.1 The
neonatalmorbidity in surviving infants is high.The use of corticosteroids in preterm
birth has been responsible for a significantreduction in neonatal mortality as well as
morbidity from respiratory distresssyndrome (RDS) and intraventricular haemorrhage
(IVH).2The initial guidelines were convened in 1995, as there was a need to emphasis
theusage of antenatal steroids as it was felt that at that time there was some
hesitationfor its usage. This has however improved over the years. The
representatives asbelow reviewed the original recommendations in August 2001 and
there wasextensive discussion on three issuses.i.The choice of antenatal steroids as
there was current data to suggest thatbetamethasone may have a better outcome
than dexamethasone.3However, because this data is a Grade B recommendation, it
was felt thatthe negative effect of dexamethasone should not be highlighted.
Thisissue will constantly be reviewed as evidence becomes available.ii.The dosage of
dexamethasone per literature is 4 doses of 6mg given 6hours apart. However it is
widely given as 2 doses of 12mg given 12 hoursapart. There is no clinical data to
support this regimen
http://www.acadmed.org.my/view_file.cfm?fileid=200
150) A 32 yr old P2L2 lady comes 5 days after unprotected sexual intercourse. Best
management for contraception is
a) Levonorgestrol 0.75mg
b) Copper IUCD
c) 2 tablets of high dose OCP, repeated after 24 hrs
d) Laparoscopic tubectomy
B
2.5.2 Intrauterine device (IUD)
A trained healthcare provider can insert a Copper T IUD within 5-7 days of
unprotected sex.
This is the most effective EC method but is not suitable for masses since infrastucture
and training is required. CuT use is undesirable in young nulliparous women or in
women with multiple sex partners. There is a risk of pelvic inflammatory disease and
subsequent infertility following CuT use especially in high-risk women. Sometimes,
irregular bleeding associated with IUD insertion may mask diagnosis of early
pregnancy.

Advantages :
higher efficacy rates (over 99%)4
very few compliance issues
no systemic effects
can be used as ongoing long-term contraception
suitable for clients with history of thrombo-embolic disease, stroke, heart attack and
women who have difficulty ingesting ECP due to vomiting
may be suitable for women who present later than 120 hours and before 7 days of
unprotected intercourse 4 .
can be used as a third-chance when EC pill has been vomited or the woman is unable
to take the full dose due to intractable nausea or other side effects.
Disadvantages :
unsuitable for women with multiple sex partners and those with active reproductive
tract infection
relatively contraindicated in nulliparous women
not suitable for women with previous ectopic pregnancy
the infrastructure and trained personnel may not be available everywhere
irregular bleeding or spotting associated with CuT insertion may be confused with
bleeding due to abortion or ectopic pregnancy and vice versa, delaying the diagnosis.
http://www.aiims.edu/aiims/events/Gynaewebsite/ec_site/manual/1_2_1.ht
m
151) A G6+0+0 lady with h/o recurrent missed abortions at 14-16wks comes to you
with abortion. Which of the following tests is not warranted
a) Lupus anticoagulant
b) Anticardiolipin ab
c) VDRL for wife and husband
d) Fetal karyotype
D
152) In Galactorrhea - amenorrhea syndromes, which is the investigation you should
advise (apart from serum prolactin)
a) TSH
b) LH
c) Urinary ketosteroids
d) HCG
A
LABORATORY TESTS
Laboratory studies may include a serum pregnancy test, a prolactin level, renal

function tests and a thyroid-stimulating hormone level.


Because prolactin levels are influenced by stress and breast stimulation, blood should
not be drawn immediately after a breast examination. Rather, it should be drawn at
least one hour after the examination and when the patient is relaxed. If the initial
prolactin level is borderline, the level should be repeated one or two times because
of the great fluctuation in prolactin levels throughout the day. A level greater than
200 ng per mL (200 g per L) is almost always associated with a prolactinoma or other
prolactin-secreting tumor.
Serum cortisol, growth hormone and insulin-like growth factor levels should be
obtained if the patient has signs or symptoms of Cushing's disease (cushingoid
features) or growth hormone excess (acromegalic features)
http://www.aafp.org/afp/2001/0501/p1763.html

153) Drug not given in PCOD


a) Tamoxifen
b) Clomiphene
c) Oral contraceptive
d) ?Metformin
a
154) A lady underwent vaginal hysterectomy for ? Ca cervix. Following the surgery
after her urethral catheter was removed, she complained of urinary incontinence. On
examination she had normal voiding as well as continuous incontinence. Methylene
blue dye was instilled in her bladder through her urethra and she was given oral
Phenazopyridine. After some time her pad was checked and it showed yellow staining
at the top portion, but not the middle or bottom portions. She is likely to have
a) Ureterovaginal fistula
b) Vesicovaginal fistula
c) Urethrovaginal fistula
d) Vesicouterine fistula
a

Double dye test


From Wikipedia, the free encyclopedia
"Double dye" redirects here. For the concept in numismatics, see double die.
Double dye test is useful for diagnosing vesicovaginal or ureterovaginal fistulae. For this test, the patient takes
oral phenazopyridine (Pyridium) 200 mg TDS, and indigo carmine or methylene blue is filled into the empty urinary
bladder via a urethral catheter. Pyridium turns urine orange in the kidneys, and methylene blue (or indigo carmine)
turns urine blue in the bladder.
A tampon is placed into the vagina. If the tampon turns blue, vesicovaginal fistula is suspected. If the tampon turns
orange, ureterovaginal fistula is suspected. If the tampon turns blue and orange, suspect a combination of
vesicovaginal and ureterovaginal fistulae. It is important to be alert for leakage around the catheter, which may

spill back into the vagina creating the false impression of a fistula. It is also important to ensure that adequate
distension of the bladder occurs as some fistulae do not leak at small volumes; conversely, some fistulae with an
oblique track through the bladder wall may leak at small volumes, but not at capacity. Direct inspection of leaking
dye in vagina in lithotomy position is better than the traditional 'three swab test' as multiple fistulae may be
located in this way

https://en.wikipedia.org/wiki/Double_dye_test

155) A 16 yr old girl was brought with primary amenorrhoea. Her mother mentioned
that she started developing breast at the age of 12. She was prescribed OCPs 2 yrs
back by a doctor with no effect. She was having normal stature and was a football
player. On examination breast was well developed(Tanners stage 5) and pubic hair
was minimal (Tanners stage 1). What is the diagnosis
a) Premature ovarian failure
b) Turners syndrome
c) Total androgen resistance
d) Mullerian ageness
c
Androgen insensitivityAndrogen insensitivity is the third most commoncause of
primary amenorrhea. The patient is a malepseudohermaphrodite: she has testes and a
XYkaryotype, but is phenotypically female. She hasnormal growth and development,
although thebreasts are abnormal (small pages , less glandulartissue), underdeveloped
labia minora, less deepvagina and no uterus. Body hair, axillary and pubichair are
absent or sparse. More than half of thesewomen have inguinal hernias which contain
thetestes. Testes should be removed around age 1618years because cancer might
develop. There is nowithdrawal bleeding after the progestational chal-lenge test or
the combined oral contraceptive pill.An ultrasound should be able to establish the absence of a uterus
https://www.glowm.com/pdf/Chap-08_Lagro.pdf
156) A young lady with 6 weeks amenorrhea had nausea and vomiting with severe
abdominal pain. Her BP was100/80 mmHg. Examination revealed a 5 x 5cm adnexal
mass. What is the plan of management?
a) Plan for immediate laparoscopic surgery
b)
c) give methotrexate
d) Give IV fluids, keep NPO and observe for 4-5 days
A
resuscitation first
Then laparoscopy to get the correct diagnosis.
if no laparoscopic facility, USG and diagnostic tapping should be done.
Size 5x5cm is large for 6weeks amenorrhoea patient with ectopic pregnancy , I think. But may be if it
ruptured and sealed with omentum
So twisted ovarian cyst is also suspected.
===================================================
In women of childbearing age who have not had a hysterectomy, including those on contraceptives,

those with an intrauterine device in place, and those with partners who have had a vasectomy, a
pregnancy test via urine -HCG should be performed. When the diagnosis is unclear and a
gynecological cause is included in the differential diagnosis, a gynecology consult should be
considered. In all pregnant patients with abdominal pain, a gynecology consult is strongly
recommended to optimize fetal and maternal outcomes. If diagnostic uncertainty exists, laparoscopy
with direct visualization of the pelvis and abdominal cavity is often the best modality for investigating
pelvic pain in women.

http://www.ncbi.nlm.nih.gov/pmc/articles/PMC3313456/
Age-Related Differential Diagnosis of the Pelvic Mass: Reproductive Age-Group After menarche,
adnexal masses will most likely be follicular or corpus luteum cysts of the ovary. Follicular cysts arise
during normal ovarian function, when amature follicle fails to rupture at ovulation or from bleeding
into the follicle atovulation. Corpus luteum cysts result from hemorrhage within a persistent
maturecorpus luteum. They can attain a larger size than follicular cysts (5-10 cm) and aremore likely
to cause a dull, aching unilateral pelvic pain. When corpus luteumcysts rupture or bleed, they may
cause abrupt severe pelvic pain. If the pregnancy test is positive in a patient with a pelvic mass,
possibilities include intrauterine pregnancy, ectopic pregnancy (see related article in thisHandbook)
or much less commonly, trophoblastic disease. Neoplasms are rare but the most common
neoplasm in this age group is the benign cystic teratoma or dermoid cyst. They are usually 5-10 cm
in diameter andin 15% of cases, are bilateral. They are slow growing and usually the patient
isunaware of their existence. They may contain teeth, hair, sebaceous material andneural elements
Fibroids can be found in the uterus, ovary, cervix, pelvic ligaments or other pelvic organs. They
occur in 30% of premenopausal women and are 3x more common inblacks. Symptoms include
pelvic pain, bladder pressure, dysmenorrhea andmenorrhagia. Their size can vary from mm to
absolutely huge masses filling theentire abdominal cavity. Leiomyosarcomas can occur in these
tumors but theseare very rare (<0.1%), usually presenting at age 50-55. If the patient has severe
pelvic pain, an adnexal mass and a negative pregnancy test, consideration should be given to the
diagnosis of ovarian torsion. Here theovary twists on itself (usually because of an OVARIAN CYST)
causing pain which maywax and wane initially but will remain constant and severe once it has
completedtorsion. Complete torsion is a surgical emergency and if not corrected, willrequire
oophorectomy. Endometrial implants outside the uterus may develop in patients with
endometriosis. Endometriosis most commonly occurs in white, nulliparouswomen between the ages
of 35-45. These implants may occur on the ovaries andoccasionally can form large cysts filled with
chocolate colored, syrupy fluid.These cysts are called chocolate cysts or endometriomas. Patients
usually havesx such as cyclic pelvic pain or pressure and/or dyspareunia. In salpingitis, the
fallopian tube becomes distended with pus forming a hydrosalpinx which is detected on physical
examination as a tubular adnexalmass. The ovary may become involved in a tubo-ovarian abscess
(TOA).Generally, the patient will be febrile and complaining of pelvic pain. Sertoli-Leydig cell
tumors, ovarian neoplasms, adrenal neoplasms, and polycystic ovary syndrome (PCOS) can cause
masculinization or defeminization to varyingdegrees. These changes include hirsutism, clitoral
enlargement, frontal balding,acne, coarsening of the voice, and enlargement of shoulder girdle
muscles.Occasionally, the enlarged ovaries in PCOS can be detected on physicalexamination.
Granulosa cell tumors can cause increased feminization with precocious puberty, menorrhagia, or
post-menopausal bleeding

157) A lady with abdominal mass was investigated. She was found to have b/l ovarian masses
with smooth surface. On microscopy they revealed clear cells with signet ring shapes. Diagnosis
a) Dysgerminoma
b) Krukenberg tumour
c) Primary Adenocarcinoma of the ovaries
d) ?

B
158) What are the cut off values in 2 hr OGTT for FBS, PP1 hr and PP 2 hrs respectively
a) 92 , 182, 153
b) 90, 180, 153
c) 95, 180, 155
d) 92, 180, 153
ans=A
http://www.perinatology.com/Reference/Reference%20Ranges/2OGTT.htm
Normal blood values for a 75-gram oral glucose tolerance test used to check for type 2
diabetes in those who are not pregnant:
Fasting: 60 to 100 mg/dL
1 hour: less than 200 mg/dL
2 hours: less than 140 mg/dL
The examples above are common measurements for results of these tests. Normal
value ranges may vary slightly among different laboratories. Some labs use different
measurements or test different samples. Talk to your doctor about the meaning of
your specific test results.
What Abnormal Results Mean
A glucose level than is higher than normal may mean you have pre-diabetes, diabetes,
or gestational diabetes.
Between 140 and 200 mg/dL is called impaired glucose tolerance. Your doctor may
call this "pre-diabetes." It means you are at increased risk of developing diabetes over
time.
A glucose level of 200 mg/dL or higher is used to diagnose diabetes.
Serious stress to the body, such as from trauma, stroke, heart attack, or surgery, can
raise your blood glucose level. Vigorous exercise can lower your blood glucose level.
Some medicines can raise or lower your blood glucose level. Before having the test,
tell your health care provider about any medicines you are taking.
http://www.nlm.nih.gov/medlineplus/ency/article/003466.htm

159) In a child surgery was done for EHBO with ?hepatojejunal anastomosis. Post operative
bilirubin level after 2 weeks was 6 mg/dl from a pre operative level 12mg/dl. The reason for this
could be
a) Normal lowering of bilirubin takes time

b) Delta bilirubin
c) Anastomotic stricture
d) ??
A
ANS=A
*****We have shown that the outcome of the Kasai PEwas predictable by assigning the patients
postoperativelyinto 2 groups based on the values of the direct bilirubinlevel (DB). Groups were defined
by stratifying the pa-tients at 6 months post-PE into a successful (DB dropped to 2.0 mg/dL, no liver
transplant) or failed (DB 2mg/dL, death or liver transplant) outcome.****** The positivepredictive
values (PPV) for the outcome 2 and 5 yearspostoperatively, based on the 6 months
postoperativestatus, were high. Eighty-five percent of our patientsreached a follow-up time of 5 years
postoperatively, inwhich case the PPV of the status at 6 months postoper-atively proved to be 95%.
Although satisfactory biledrainage can be obtained with PE in most patients, liverfunction can
deteriorate progressively with a possibleturning point in late adolescence, indicating that as thelength
of follow-up increases, comprehensive clinicalassessment should still be regularly performed.2,3 However, in patients aged 20 years or more, long-term fol-low-up still may be essential. The negative
predictivevalues (NPV) of the status at 6 months postoperativelywere lower, being 76% at 2 years
postoperatively. Withlonger follow-up (at 5 years post-Kasai), our stratification at 6 months was able
to predict failure at a rate of74%. Thus, patients who were predicted to undergo aliver transplantation
or die can still improve and have asuccessful outcome. Overall, the PPV and NPV of thestatus at 6
months postoperatively were useful predictorsof outcome. These data are similar to those found
byOhama et al4. In that review, the authors found that theserum bilirubin level 3 months after the
Kasai operationcan be used to predict long-term survival rate and thetime when the onset of liver
failure is likely to occur.4These findings, however, were in contrast to anotherstudy by LopezSantamaria et al5 which did not find acorrelation between an increased postoperative bile flowand a
good outcome.In our review, one valid preoperative factor for deter-mining outcome of a Kasai PE
appeared to be thepresence of bridging fibrosis in the liver biopsy. Fibrosispredicted a poor outcome;
however, this covariate wasnot as significant as the number of cholangitic episodes.Nevertheless, Fig 1
shows graphically, that for any num-ber of cholangitic episodes, the presence of fibrosisincreased the
relative risk of an unsuccessful outcome.Bridging fibrosis in the liver biopsy at the time of
PErepeatedly has been described as a negative risk factorfor a successful outcome of the Kasai
operation.6-8 It hasbeen shown that BA patients with extensive liver fibrosisat surgery will have
cirrhosis in 86% of cases without theadditional effect of other risk factors like cholangiticepisodes.9
Another extensive prospective study con-firmed this observation.6 However, these investigatorsfound
that the efficacy of PE was less influenced by theinitial degree of liver fibrosis than by the histology of
theporta hepatis (eg, the relationship between the bile ductremnants and the portal and arterial
vessels). In thisstudy, fibrosis was seen only as a secondary criterion indetermining prognosis
compared with the bile duct pat-tern. A low level of fibrosis was of no predictive value ifthere were not
sufficient bile duct remnants in the portahepatis. Our results were not analyzed by grouping theminto
different portal plate patterns, instead, only the ductsize was taken into account

http://www.unige.ch/medecine/FACULTEETCITE/newsletter/novembre2010/p
df6.pdf

Significance of serum delta-bilirubin in patients with obstructive jaundice.


Kozaki N1, Shimizu S, Higashijima H, Kuroki S, Yamashita H, Yamaguchi K, Chijiiwa K, Tanaka M.
Author information
Abstract
BACKGROUND:
Delta-bilirubin is a bilirubin covalently bound with albumin, which is nontoxic and excreted neither in
urine nor in bile. We previously reported that the percentage of delta-bilirubin increased after biliary
drainage and that the rapidly excretable bilirubin fraction (total minus delta-bilirubin) was a better
parameter to predict the effectiveness of biliary decompression in the dog model. The aim of the
present study was to elucidate whether it is applicable to humans.
MATERIALS AND METHODS:
The serum bilirubin concentration was measured and its fractions were analyzed by high-performance
liquid chromatography in 22 patients with obstructive jaundice before and after biliary drainage. In
addition, the patients were subgrouped into good and poor drainage groups according to the decline
index of serum bilirubin to examine the significance of delta-bilirubin.
RESULTS:
The concentration of total bilirubin decreased from 14.1 mg/dl before biliary drainage to 5.4 mg/dl 28
days after drainage. During this period, the percentage of conjugated bilirubin steeply declined from
47.1 to 8.8% and that of excretable bilirubin from 63.4 to 28.6%. In contrast, the proportion of serum
delta-bilirubin increased from 36.6 to 71.4%. There was an inverse correlation between percentage of
delta-bilirubin and total bilirubin concentration (r = -0.69, P < 0.01). In the good drainage group, the
percentage of delta-bilirubin increased above 60% within 7 days after biliary drainage, but it did not
reach 60% by 28 days in the poor drainage group. A decreasing rate of total bilirubin minus deltabilirubin, the excretable bilirubin fraction, was a better index than that of total bilirubin to assess the
efficacy of biliary drainage (P< 0.01).
CONCLUSIONS:
The increase in the percentage of serum delta-bilirubin indicates an effectiveness of biliary drainage in
man. An analysis of serum delta-bilirubin for 7 days can distinguish the good drainage patients from
the poor drainage patients.
Copyright 1998 Academic Pre
===================================
Development of strictures of hepaticojejunal anastomoses (HJA) is observed in 6-30% of patients and
mortality after repeated reconstructive interventions ranges from 13% to 25%. Double balloon
enteroscopy (DBE) allows one to visualize the zone of Roux-en-Y anastomosis after reconstructive
operations on the bile ducts for differentiation between stricture of HJA and recurrent cholangitis.

160) Fluid of choice in diarrhea in an infant


a) ORS
b) Salt water
c) Sugar water
d) Dextrose

161) A mother comes with history of neural tube defect in first child. What is the amount of folic
acid you will prescribe during preconceptional counselling (In micrograms / day)
a) 4
b) 40
c) 400
d) 4000
C
Countries and Organizations United for Neural Tube Defects Prevention
Birth Defects COUNT is CDCs global initiative to significantly reduce death and lifelong disability
resulting from neural tube defects.
What are neural tube defects and why are they a public health concern?
Neural tube defects are serious birth defects of the brain and spine. They are a major cause of death
and lifelong disability worldwide.
The two most common neural tube defects are spina bifida (defect of the spine) and anencephaly
(defect of the brain). They happen very early in pregnancy when the neural tube, which becomes the
brain and the spine, does not close properly.
Spina bifida can cause lifelong disabilities that range from mild to severe.
Almost all babies born with anencephaly will die shortly after birth.
In the United States, 3,000 pregnancies are affected by neural tube defects every year (1).
Hispanic women have higher rates of neural tube defects than non-Hispanic women in the United
States. (2)
Worldwide there are more than 300,000 babies born with neural tube defects each year. (3)
The total lifetime direct cost of care for a child born in the United States with spina bifida is estimated
to be $706,000. (4)
What has been done?
CDC has worked to prevent neural tube defects over the past two decades and has led the way in
folic acid research.**** Folic acid, a B vitamin, is known to reduce the risk of a pregnancy being
affected by a neural tube defect when a woman takes 400 micrograms daily before and during early
pregnancy.****** By 1998 in the United States, folic acid was added to all cereal grain products
labeled as enriched, like breakfast cereals and corn grits. This process is called folic acid
fortification. Before folic acid fortification, about 4,100 pregnancies were affected by a neural tube
defect each year in the United States. After fortification, this number declined to about 3,000. The
number of deaths each year due to neural tube defects also declined from 1,200 before folic acid
was added to 840 after folic acid was added. (1)

http://www.cdc.gov/ncbddd/folicacid/global.html

162) Regarding Japanese encephalitis vaccine, what is not true

a) Not given for infants less than 6 months


b) 2 primary doses given to children in the 1-3 yr age group
c) Booster doses are given after 1 yr and repeated every 3 yrs
d) effect start from second week onwards
e) protection lasts for five years
f)in endemic areas vaccination is given to cover children between one to nine yrs of age
ans=E>D?
Type of vaccine and schedules:
1) Live attenuated vaccine (SA 14-14-2 strain). In China, the first dose is given subcutaneously at
age 8 months, followed by a booster dose at 2 years of age. In some areas, an additional booster is
offered at 67 years of age. However, protection for several years may be achieved with a single
dose of this vaccine, and in many countries one dose without subsequent boosters is recommended.
2) Inactivated, Vero cell-derived, alum-adjuvanted vaccine (SA 14- 14-2 strain). Primary
immunization consists of two intramuscular doses, 4 weeks apart. A booster is recommended after 1
year.
3) Inactivated Vero cell-derived vaccines (Beijing-1 strain). Primary immunization consists of three
doses at days 0, 7 and 28, or two doses given preferably 4 weeks apart (0.25 ml for children
heart
years, 0.5 ml for all other ages). One booster is recommended 1214 months after
emoticon
completion of the primary immunization and thereafter every 3 years.
4) Live chimeric vaccine (with yellow fever 17D as backbone). A single dose is recommended; the
need for and timing of a possible booster dose have not yet been determined
Adverse reactions: Occasional mild local or systemic reactions
Contraindications and precautions:
A hypersensitivity reaction to a previous dose is a contraindication.
In principle, the live attenuated vaccine should be avoided in pregnancy unless there is a high risk of
exposure to the infection

http://www.who.int/ith/vaccines/japanese_encephalitis/en/
??The results revealed that even a single dose of the test vaccine was sufficient to elicit the
immuneresponse. On 28th day, the subjects who had received asingle dose were 98.67%
seroprotected and 93.14%seroconverted (4 fold) for 50 - 1 years, whereas thecorresponding
figures for the reference vaccine were77.56% and 57.69%, respectively (p-value <0.001)
-------------------------------------------------The committee reviewed the efficacy of this vaccinein India. A small casecontrol study from
Lucknow, Indiafound an efficacy of 94.5% (95% CI, 81.5 to 98.9) after asingle dose of this vaccine
within 6 months after itsadministration [16]. However, data from post marketingsurveillance (PMS) in
India (ICMR unpublished study)showed that protective efficacy of the vaccine in India isnot as high
as that seen in Nepal. According to the study,sero-conversion rates at 28 days after vaccination
were73.9% and 67.2% in all individuals and in those who werenonimmune pre-vaccination,
respectively. The protectiveefficacy of the vaccine at one year was 43.1% overall and35% for those
who were non-immune pre-vaccination,respectively [17]. Preliminary results of a recent casecontrol
study show an unadjusted protective effect of62.5% in those with any report of vaccination
[17].According to this report, the JE vaccine efficacy hasbeen around 60% in Uttar Pradesh and

around 70% inAssam

http://medind.nic.in/ibv/t13/i12/ibvt13i12p1095.pdf
How long does the Japanese encephalitis vaccination last?
The duration of protection is unknown. For persons aged 17 years and older, a booster dose may be
given if the primary two-dose vaccination series was given one year or more previously and there is
continued risk of exposure. Although studies are being conducted on the need for a booster dose for
children, data are not yet available.

http://www.cdc.gov/japaneseencephalitis/qa/
163) A 1month old child with conjugated bilirubinemia and intrahepatic cholestasis. On Liver
biopsy and staining with PAS red coloured granules were seen inside the hepatocytes. Probable
diagnosis is
a) Alpha1 Antitrypsin deficiency
b) Congenital hepatic fibrosis
c) ?
d) ??
??
A
A few other causes of pediatric cholestasis have distinctive histologic findings.
Alpha-1-antitrypsin deficiency is associated with a broad range of liver disease, both
in children and adults (Chapter 9). It is responsible for 7% to 10% of cases of neonatal
cholestasis, and, conversely, between 10% and 20% of infants with a deficient (PiZZ)
phenotype develop clinical evidence of liver disease, although many others have
minimal disease only characterized by laboratory abnormalities.178,181 The outcome
is variable: some patients develop early cirrhosis and die of liver failure, a few may
progress to chronic disease later in life, and many appear to recover, demonstrating
only minimal evidence of hepatic disease.167,173,176
The histologic hallmark of alpha-1-antitrypsin deficiency is the presence of
eosinophilic globules in periportal hepatocytes (Figure 6-19). These PAS-positive and
diastase-resistent globules represent dilated sacs of endoplasmic reticulum stuffed
with alpha-1-antitrypsin. They may be small and difficult to see in neonatal livers, but
are consistently present after about three months and have even been noted in a 20
week fetus.171,174,179,182 Alpha-1-antitrypsin globules can be confused with other
PAS-positive material, but immunohistochemical techniques can be used to
specifically confirm their presence.169 Measuring serum levels and determining the
phenotype can definitively establish the diagnosis.
Three other metabolic disorders -- galactosemia, hereditary tyrosinemia type I, and
hereditary fructose intolerance -- are all characterized by a broadly similar histology
(Chapter 9). Although not specific, these changes are suggestive and should prompt
appropriate biochemical analysis. In addition to canalicular cholestasis, early changes
include prominent fatty change and acinar transformation of liver-cell plates, often in
the form of cholestatic rosettes (Figure 6-20). Giant multinucleated hepatocytes are
variably present, and portal fibrosis and bile ductular proliferation also noted. With
advancing disease, intralobular fibrosis and fibrous septa progressively develop and,
without appropriate treatment, cirrhosis may result, particularly in tyrosinemia and

galactosemia.168,175,177,180
Liver dysfunction associated with total parenteral nutrition occurs most frequently in
premature infants. The pathologic picture is variable and nonspecific, but may
include canalicular cholestasis, fatty change, giant multinucleated hepatocytes, and
portal inflammation and fibrosis, which may progress to cirrhosis.15,170,172 The
overlap with neonatal hepatitis and biliary atresia may lead to differential diagnostic
problems. Benign recurrent intrahepatic cholestasis is characterized by
nonprogressive, repetitive attacks of acute cholestasis and may have an onset in
childhood (Chapter 4). The histologic pattern is that of pure cholestasis -- canalicular
cholestasis unaccompanied by other morphologic changes.
http://tpis.upmc.com/tpislibrary/dlp/CHAP6h.html
REF=Biopsy Interpretation of Pediatric Lesions
By Aliya N. Husain

164) [PICTURE] Acne rosacea


165) 26 yr old man from Bihar comes with juicy looking papules over face and back of neck
[PICTURE] which were hypopigmented and normoaesthetic with no nerve thickening. History
of prolonged fever in childhood was present. Diagnosis
a) Tuberculoid leprosy
b) Post Kala Azar dermal leishmaniasis
c) Lepromatous leprosy
d) ?

B
In Africa, PKDL by far mostly occurs in Sudan. It is much less common inEthiopia,
Kenya or Uganda. The reason for this is not clear; differences inthe parasite or the
genetic background of the population may be of im-portance. Up to 50-60% of VL
cases develop PKDL, usually within 0-6months after treatment. Some patients do not
have a previous history ofVL and probably had subclinical VL infection. Clinical
presentation In contrast with VL, the patient is generally well, except in severe

cases.The initial presentation is usually with some papules around the mouth;these
increase in number and size and spread further to cover most of theface. The most
common presentation is a maculopapular rash with papulesoccurring on a macular
background. The papules may be small resemblingmeasles; others increase in size and
may be called nodules; these may beco-me confluent. PKDL is often described in 3
grades of density and spread oflesions. Patients may present with a macular rash only,
but this is much lesscommon as e.g. in Bangladesh. The macular rash seems not to
follow theclassical spread as in the papulonodular form. Other more uncommon presentations include a patchy distribution of plaques and the verrucous type.Ulceration
is not a feature, but there may be sloughing of heavily affectedparts of the skin; in
case of mucosal involvement ulcers may form.
--------------------------------------Post-kala-azar dermal leishmaniasis (PKDL) is a complication of visceralleishmaniasis
(VL) or kala-azar. It is common in areas endemic for VL cau-sed by L. donovani. These
include countries in Africa in particular Sudanand in Asia, Bangladesh and to a lesser
extent India. PKDL may also spora-dically occur in L. infantum or L. chagasi endemic
areas, mainly the Medite-rranean countries and Latin America.The condition is
characterized by the occurrence of a skin rash after anepisode of VL; the interval
varies according to the endemic area. The rashis usually in the face, from which it
may or may not spread to other partsof the body. In contrast to VL, the patient is not
ill and PKDL is not fatal. Inthe Sudanese type, self cure is the rule while in Bangladesh
and India, allcases are treated.Risk factors for PKDL are not well known; previous
treatment of VLwith inadequate dosage of drug and the drug used, malnutrition, HIV
in-fection and young age may play a role.The importance of PKDL is twofold:
Clinical: patients develop a rash that may last for weeks or months; in particular in
small children, the rash may become generalized andsevere with mucosal lesions in
the mouth, causing general dis-comfort. Epidemiological: smears or biopsies taken
from the lesions may show Leishmania parasites and there is evidence that the
sandflyvector may take up these parasites while taking a blood meal andthus PKDL
patients may play an important role in transmission(anthroponotic transmission). It is
thought that VL occurs in cy-cles with epidemics of thousands of cases, followed by a
period ofseemingly low transmission. It is likely that chronic PKDL patientswho
harbour parasites may play an important role in subsequentupsurges in VL
cases.Diagnosis is usually clinical by the triad of the typical rash, its distributionand
the previous episode of VL. There are however, often difficulties andexceptions: many
patients do not have a previous episode of VL and therash may mimic other common
skin conditions. In addition, the presentationin Africa and Asia is quite different with
the maculopapular form and typicalspread being the most common in Sudan and the
macular form being muchmore common in Bangladesh, often with a more atypical
distribution.Parasites may be found in the lesions but this requires a skin smear
orbiopsy; in papular or nodular PKDL the parasites can usually be demons-trated but
in the macular type they are scanty. Serological diagnosis isnot very helpful as most
patients will have a previous history of VL andantibodies may persist as a result and
therefore a positive test may bedifficult to interpret

http://www.who.int/iris/bitstream/10665/101164/1/9789241504102_eng.pd
f
166) A lady came with unilateral white skin lesions with leukotrichia. [PICTURE] Shows 2
white spots 1 x 1cm each over left side of chin and neck. Diagnosis
a) Segmental vitiligo
b) Piebaldism
c) ?
d) ??
B
ans=A
Segmental vitiligo occurs as a single white patch in 90%, but in a few, a second or rarely a third
segment is affected. Each segment affects one side of the body. It arises in a young person and
after the first year, generally remains stable. It is an example of cutaneous mosaicism. The
segments often follow Blaschko lines, but occasionally have dermatomal pattern (following nerve
distribution), phylloid pattern (leaf-like) or checkerboard pattern. The border of the white patch can
be irregular or less often, smooth. White hair is often noted within segmental vitiligo.

http://dermnetnz.org/colour/vitiligo.html

167) A lady came with complaints of a bluish lesion over left side of forehead and left eye.
[PICTURE] Shows Irregular bluish lesion in Left superior conjunctive and ? forehead.
Diagnosis
a) Nevus of Ota
b) Nevus of Ito
c) Beckers nevus
d) Mongolian spot

Naevi of Ota and Ito


What are these naevi?
A naevus (plural naevi, American spelling nevus) is a circumscribed and stable malformation of a
component of the skin. Naevi are often present at birth, when they are often called birthmarks.
Naevi composed of the pigment cells that produce melanin (melanocytes) are called melanocytic
naevi.
Naevus of Ota, naevus of Ito and naevus of Hori are melanocytic naevi with slate-brown or blue/grey
colouring. The naevus cells are found deep within the dermis, a form of dermal melanocytosis.

Naevus of Ota is on the forehead and face around the eye area. Hyperpigmentation of parts of
the eye may occur: sclera, cornea, iris, retina;

Naevus of Hori is similar to naevus of Ota but is on both sides of the face

Naevus of Ito is on the shoulder and upper arm area (shoulder girdle).

Dermal melanocytosis can also occur elsewhere on the body, including inside the mouth.

How does dermal melanocytosis arise, and who is at


risk?
It is not known why dermal melanocytosis occurs. Specific mutations have been detected within the
dermal melanocytes, most often GNAQ or GNA11. Researchers have suggested that hormones play a
part in their development. The role of ultraviolet radiation is thought to be small, as it does not reach
deep dermal melanocytes.
Naevus of Ota is much more common than naevus of Ito. These naevi are present at birth in 50% of
cases but may appear during adolescence or adult life. Naevus of Hori is not present at birth and is
therefore a form of acquired melanocytosis.
Naevi of Ota and Ito are most commonly found in Asian populations; 0.2-0.6% of Japanese people
have nevi of Ota. They appear more frequently in females. Both forms of naevi are uncommon in
Caucasians.

What are the signs, symptoms and complications of


dermal melanocytosis?
In all forms of dermal melanocytosis:

Colour may vary to include brown-violet, violet-blue or blue-green hues

Naevi present in childhood may slowly grow and darken until adulthood is reached

Colour or perceived colour of naevi may change according to personal and environmental
conditions, e.g. fatigue, menstruation,HOT

weather

If affecting the eye, melanocytosis rarely causes glaucoma

Malignant melanoma very rarely develops within dermal melanocytosis, and has usually
been reported in Caucasians. Ocular melanoma has been reported in the choroid, brain,
orbit, iris, ciliary body, and optic nerve in association with a nevus of Ota.

How is the diagnosis of dermal melanocytosis made?


The diagnosis of dermal melanocytosis is usually made by observing typical discolouration of the skin.
It is classified according to the site affected.
Some patients may undergo skin biopsy, which confirms the presence of melanocytes in the dermis.
Other skin conditions resulting in bluish or grey coloured skin may be considered. These include:

Blue naevus

Drug-induced pigmentation, eg due to minocycline, a tetracycline antibiotic

Melasma

Postinflammatory pigmentation

Lichen planus pigmentosus and ashy dermatosis

What treatments are available for melanocytosis?


Treatment of a melanocytosis includes:

Cosmetic camouflage to cover the disfiguring markings.

Laser treatment (usually using 1064nm Q switched Nd:YAG or QS ruby laser)


and intense pulsed light (IPL). These work by destroying the dermal melanocytes.
Multiple treatments are necessary, often with a combination of devices. Laser treatment is
more effective in light skinned individuals than in those with dark skin. Unfortunately
recurrence is common after laser clearance, sometimes resulting in a darker hue.

If the eye is affected, regular eye examinations should be arranged to detect glaucoma. Any change in
a naevus should be assessed by a dermatologist.

http://www.dermnetnz.org/lesions/naevus-ota-ito.html
168) The patient came with h/o bullae involving >30 % body surface area and erosions of the
lips for the past 7 days. [PICTURE] Shows ?Forearm with extensive reddish bullae. What is
probable regarding the etiology

a) Drugs
b) Bacterial infection
c) Viral infection
d) ?
A
Drug induced
Induction of SJS is most often due to drugs, especially sulfonamides, phenytoin, barbiturates, lamotrigine,
aminopenicillins, nonnucleoside reverse transcriptase inhibitors (e.g., nevirapine), and carbamazepine.
Widespread dusky macules and significant mucosal involvement are characteristic of SJS, and the cutaneous lesions
may or may not develop epidermal detachment. If the latter occurs, by definition, it is limited to <10% of the body
surface area (BSA). Greater involvement leads to the diagnosis of SJS/TEN overlap (1030% BSA) or TEN (>30%
BSA).
In addition to primary blistering disorders and hypersensitivity reactions, bacterial and viral infections can lead to
vesicles and bullae. The most common infectious agents are HSV (Chap. 216), varicella-zoster virus (Chap. 217),
and S. aureus (Chap. 172).
Staphylococcal scalded-skin syndrome (SSSS) and bullous impetigo are two blistering disorders associated with
staphylococcal (phage group II) infection. In SSSS, the initial findings are redness and tenderness of the central
face, neck, trunk, and intertriginous zones. This is followed by short-lived flaccid bullae and a slough or exfoliation
of the superficial epidermis. Crusted areas then develop, characteristically around the mouth in a radial pattern.
SSSS is distinguished from TEN by the following features: younger age group (primarily infants), more superficial
site of blister formation, no oral lesions, shorter course, lower morbidity and mortality rates, and an association
with staphylococcal exfoliative toxin (exfoliatin), not drugs. A rapid diagnosis of SSSS versus TEN can be made by
a frozen section of the blister roof or exfoliative cytology of the blister contents. In SSSS, the site of
staphylococcal infection is usually extracutaneous (conjunctivitis, rhinorrhea, otitis media, pharyngitis, tonsillitis),
and the cutaneous lesions are sterile, whereas in bullous impetigo, the skin lesions are the site of infection.
Impetigo is more localized than SSSS and usually presents with honey-colored crusts. Occasionally, superficial
purulent blisters also form. Cutaneous emboli from gram-negative infections may present as isolated bullae, but
the base of the lesion is purpuric or necrotic, and it may develop into an ulcer (see Purpura, below).

http://clinicalgate.com/cutaneous-drug-reactions-2/
169) A pt came with h/o itchy tense blisters whose roof settled as above. [PICTURE] Shows big
blisters on ?forearm which have subsided with the roof sticking to the floor without much
damage. Diagnosis

a) Bullous pemphigoid
b) Pemphigus vulgaris
a
Bullous pemphigoid (BP) is the most common autoimmune disease that results in subepidermal
blistering. This condition typically presents with generalized itchy eruptions with blisters and can be
associated with significant morbidity. In the early stages, the blisters can have quite a polymorphic
clinical appearance in the absence of vesicles and bullae.
Considered a disease of the elderly, BP usually does not develop until after age 60 years.1,2 Each
year, approximately 6 to 13 new diagnoses of BP are made per 1 million people.1 Interestingly,
patients older than age 90 years are at an approximately 300-fold greater risk for developing BP
than are patients younger than age 60 years.1 There also appears to be a higher predominance of
the disease in men than in women.

170) A child came with similar lesions over elbows, shaft of penis and [PICTURE] Shows ?
Pinhead papules over right elbow. Diagnosis
a) Lichen nitidus
b) Lichen planus
c) Scabies
d) ??

Lichen nitidus
What is lichen nitidus?
Lichen nitidus is an uncommon inflammatory skin condition that usually presents with tiny skincoloured bumps in children. Although it has been considered a variant of
nitidus is now believed to be a separate and distinct entity.

lichen planus, lichen

Who gets lichen nitidus and why?


Lichen nitidus usually begins in childhood, particularly preschool and school-aged children, or in young
adult life. It affects both sexes equally and occurs in all races.
The cause of lichen nitidus is unknown.
Lichen nitidus has been reported in association with:

Lichen planus

Crohn disease

Down syndrome

Atopic dermatitis

HIV infection

Juvenile chronic arthritis

Congenital megacolon

It is sometimes familial.

Clinical features of lichen nitidus

Typically lichen nitidus presents as a localised bumpy skin eruption involving the flexor aspects of the
forearms, backs of hands, the penis, chest, abdomen and buttocks, although any site can be involved
including the face.
The papules (bumps) have the following characteristics:

pinpoint to pinhead size 1-2mm diameter

round or polygonal shape

skin coloured, less commonly yellow, red-brown or violet

hypo- or hyperpigmented in dark skinned patients

flat-topped

shiny

scaly

rarely, tiny blisters (vesicles)

rarely, a central depression (umbilication)

clustered in groups, which can become confluent to form plaques

often found over elbows and knees so they may resemble psoriasis

localised or rarely generalised

usually asymptomatic but they can be itchy

The Kbner or isomorphic phenomenon is common, appearing as lichen nitidus papules arranged in a
line along scratch mark sites.
The palms and soles may be affected. This may be unilateral or bilateral, localised or in combination
with lesions in more typical sites. Lichen nitidus confined to the palms seems to affect an older age
group (age range 37-52 years) and tends to be very persistent.
Lichen nitidus can involve the lining of the mouth, perhaps more commonly than is reported. The
clinical features in this location are:

Tiny grey-white flat papules inside the cheeks (buccal mucosa)

White plaques on the tongue and hard palate

May resemble oral lichen planus

Changes in the nails are seen in approximately 10%, particularly in adults. These may include:

Pitting

Longitudinal linear ridging

Splitting of the free nail edge

Rippling

Redness of the proximal nailfold

Reported clinical variants of lichen nitidus include:

Generalised lichen nitidus

Haemorrhagic/purpuric lichen nitidus: the generalised purpuric form begins on tops of feet and
around the ankles before progressing up the legs and becoming generalised over months. A
palmar purpuric form may show accentuation at sites of mechanical stress

Vesicular (blistering) lichen nitidus affects the palms and resembles pompholyx

Actinic lichen nitidus appears on sun-exposed sites usually in dark-skinned patients and tends
to be seasonal, recurring in summer

Linear lichen nitidus papules arise in a line

A keratoderma form of lichen nitidus affects palms and soles, resembling chronic eczema with
fissuring

Perforating lichen nitidus is mostly reported on hands and forearms; it has umbilicated papules

How is lichen nitidus diagnosed?


Although the clinical presentation will suggest the diagnosis, lichen nitidus usually requires a

skin

biopsy to be confirmed. The histology of lichen nitidus is very typical and described as the
claw and ball appearance:

Focal dense lymphohistiocytic infiltrate in the upper dermis very close to the epidermis.

Langhans giant cells are often present in the infiltrate.

Rete ridges of the epidermis are elongated and clutch the infiltrate.

Red blood cells are seen just under the epidermis in the haemorrhagic/purpuric form.

Eosinophilic dermal material with some cell nuclei may be seen within the epidermis in the
perforating variant.

Treatment of lichen nitidus


Usually no treatment is required, as lichen nitidus does not usually cause any symptoms and resolves
within 12 months in two-thirds of cases. Post-inflammatory hyperpigmentation can persist for months,
or in some cases, for years.
Treatment may be requested by patients with:

itch

generalised rash

involvement of cosmetically sensitive sites

prolonged course

Treatments reported to have given some relief and/or resolution include:

narrowband UVB
phototherapy and photochemotherapy (PUVA)

Light: including sunlight,

Topical corticosteroids

Topical calcineurin inhibitors (tacrolimus and

Contact sensitisation with DNCB or

pimecrolimus)

diphencyprone

In severe cases, systemic treatments may be considered, including:

Corticosteroids

Ciclosporin

Retinoids

Astemizole (antihistamine, a selective H1 antagonist)

Itraconazole

Isoniazid (antibiotic often used to treat tuberculosis)

acitretin

http://dermnetnz.org/scaly/lichen-nitidus.html
171) A man with Leprosy came with the following lesion. [PICTURE] Shows a single target
lesion of about 7cm diameter over ? forearm. Diagnosis
a) LL
b) BL
c) BT
d) BB

Target lesions can be observed in the BB form of leprosy


REF=Leprosy: A Practical Guide
Front Cover
Enrico Nunzi, Cesare Massone
Springer Science & Business Media, 27-Apr-2012 - Medical - 384 pages

172) A 7 month pregnant lady comes with the following lesions all over the body. [PICTURE]
Shows multiple small ?pustules over skin. Which of these drugs is the appropriate treatment
a) Cyclosporine
b) Azathioprime
c) Methotrexate
d) Retinoids

A
Impetigo herpetiformis (IH) is a rare dermatosis of pregnancy. There is a continuing
debate whether it represents a form of pustular psoriasis triggered by the pregnancy
status or just an entity itself [1, 2, 3, 4]. The natural course of IH is commonly
resolved in the postpartum period. The major difficulty remains handling the disease
during pregnancy, especially when there is no response to systemic steroids.
Resistance to the steroid scheme served as a stimulus to consequently discuss the use
of cyclosporine as a therapeutic option for this condition.
Go to:
Case Report
A 27-year-old female, gravida 1, para 0, in week 22 of pregnancy, presented with an
eruption consisting of annular erythematosquamous plaques with an active polycyclic
elevated border comprised of superficial micropustules. Lesions were pruritic and
widely distributed on the face, trunk and extremities (fig. (fig.1,1, fig. fig.22).
Fig. 1
Fig. 1
Annular erythematosquamous plaques with an active polycyclic elevated border.
Fig. 2
Fig. 2
Erythematosquamous lesions with peripheral micropustules on the trunk.
The patient had had similar lesions in the past, with flare-ups and remissions since
adolescence. A family history of psoriasis was also REPORTED. Her mother and
grandmother had a long-standing history of plaque-type psoriasis. Histology revealed a
subcorneal collection of neutrophils, a psoriasiform epidermis and perivascular
infiltration of lymphocytes and a few neutrophils in the dermis, typical features of
pustular psoriasis (fig. (fig.3).3). Direct immunofluorescence of perilesional skin with
C3, IgG, IgA and IgM was negative.

Fig. 3
Fig. 3
Subcorneal collection of neutrophils, psoriasiform epidermis. HE. 25.
The initial treatment decision was to avoid any systemic regimen and attempt to
control the disease flare-up with topical steroids and emollients. There was no
response and the patient, having generalized lesions, was admitted to the Obstetrics
Department. Systemic steroids, at an initial dose of prednisolone 60 mg/day for 3
weeks failed to control the disease and a partial response was achieved only at a dose
of 80 mg/day. Eight days after this partial improvement, new pustular lesions
presented on the forehead, chest and buttocks. At this time point, week 27 of
pregnancy, cyclosporine at a dose of 4 mg/kg body weight was added in order to
prevent the development of new lesions and to help tapering from high steroid
dosages.
An initial response with no further eruptive pustules was observed and the eruption
remained stable for almost 1 month. Prednisolone was tapered to a minimum of 15
mg/day and in combination with cyclosporine served as a stable regimen until the day
of cesarean section, in week 32 of gestation. A healthy preterm male, 1,590 g, was
born. Throughout the time of hospitalization, all laboratory exams, including full
blood count, calcium, glucose, albumins, progesterone levels and electrolytes were
unremarkable. Blood pressure was within normal limits. Ophthalmologic examinations
were performed every week. Sonography of the fetus did not reveal any
abnormalities.
Three weeks after delivery, showing no signs of improvement, cyclosporine tapering
and the addition of methotrexate at a dose of 15 mg subcutaneously per week was
decided. The patient presented with only residual hyperpigmentation 2 months later.
Go to:
Discussion
IH typically presents during the third trimester of pregnancy and resolves during the
postpartum period [5]. It is a rare pustular dermatosis related to pregnancy with no
more than 200 REPORTED cases up to now [6]; it may recur in future pregnancies or
with menstrual exacerbation [5]. In a significant number of cases, a history of
psoriasis is well documented. It is a matter of debate whether it represents a pustular
form of psoriasis or a separate entity related to pregnancy in cases when there is no
personal or family history of psoriasis [3], when the lesions clear during the
postpartum period and never rebound [2], or when they recur in subsequent
pregnancies [1, 4].
Pathogenesis remains unclear for those who consider IH a separate entity. A possible
triggering role in the onset of the disease is attributed to high levels of progesterone
during the last trimester of pregnancy, low levels of calcium and reduced levels of
epidermal skin-derived antileukoproteinase activity implicated in the formation of
epidermal pustules [5].

Clinically, IH presents with erythematous plaques bordered by tiny pustules in a


herpetiform pattern. The centrifugal distribution of micropustules may spread all over
the body and evolves into desquamation and subsequent hyperpigmentation. Histology
is characterized by the presence of subcorneal spongiform pustules, parakeratosis and
psoriasiform hyperplasia of the epidermis, all findings of pustular psoriasis. Absence of
a perivascular infiltrate by eosinophils and of necrotic keratinocytes differentiates IH
from acute generalized exanthematous pustulosis.
Despite the typical micropustules of IH, differential diagnosis may include
autoimmune bullous diseases, like herpes gestationis, IgA pemphigus or linear IgA
dermatosis. Histology and direct immunofluorescence have a critical role in
distinguishing the above-mentioned entities. In many cases of IH, both mother and
fetus are threatened. Maternal complications may be seen in unrecognized and
unmonitored cases and consist of delirium, tetany due to hypocalcemia and
convulsions. More severe is the impact on the fetus, mainly due to placental
insufficiency. The major fetal risks are stillbirth, neonatal death and fetal
abnormalities [5].
Systemic corticosteroids are considered the gold standard in the treatment of IH,
although their efficacy in the control of the disease varies. The most common scheme
is with prednisolone at a dosage of 1530 mg/day, which may be increased to 80
mg/day in refractory cases. Prednisolone, though nonteratogenic, has been associated
in a limited number of cases with macrosomia, gestational diabetes and premature
rupture of the membranes [7].
Cyclosporine has been administered in 13 patients with IH in the literature [6, 7, 8, 9,
10, 11, 12, 13, 14, 15, 16, 17]. Reviewing the above-mentioned cases, in 12 out of 14
cases, including ours, cyclosporine was not used as a monotherapy in the treatment of
IH but in combination with systemic corticosteroids (table (table1).1). Prednisolone
dosage was between 20 and 80 mg/day. Fluocortolone and dexamethasone were used
in 2 patients. Cyclosporine was administered at a dosage of 27.5 mg/kg/day. It is of
interest that in only 1 case a complete resolution was achieved before delivery. The
outcome of the combined scheme (prednisolone + cyclosporine) was a stable
condition (3/14), a partial response (3/14) or a marked improvement (4/14). Four of
the cases were resistant to therapy and regression was observed after delivery. In
conclusion, in 10 out of 14 cases (71.4%) cyclosporine helped to achieve control of the
disease, not a clearance of the lesions.
Table 1
Table 1
Published cases of IH treated with cyclosporine
Despite placental transfer, cyclosporine seems to be rather safe. Children exposed in
utero to cyclosporine have been shown to have normal renal function. Use of the
lowest active dose and careful monitoring of the fetus is advisable, though, as there is
still a weak possibility of early rupture of the membranes [18]. In the majority of IH

cases, total regression of lesions must be expected after delivery and moreover, if
there is persistence, with an alternative regimen, such as methotrexate [19] or
infliximab [16]. In our patient, 3 weeks after cesarean section, methotrexate at a
dose of 15 mg subcutaneously was proposed, resulting in a complete clearance.
In conclusion, cyclosporine in the treatment of IH seems to represent an alternative
therapeutic option when there is resistance to systemic corticosteroids or a steroidsparing agent in order to taper high dosages of steroids. It must also be considered in
addition to corticosteroids, in order to control flare-ups of the disease
http://www.ncbi.nlm.nih.gov/pmc/articles/PMC3635965/
173) Which anesthetic agent can cause pain on IV administration
a) Thiopentone
b) Propofol
c) Ketamine
d) Midazolam
B
174) Which of the following combinations can be used for day care surgery
a) Remifentanyl, Midazolam, Propofol
b) Fentanyl, Midazolam, Thiopentone sodium
c) Morphine, Midazolam, Propofol
d) Morphine,
a
175) Muscle relaxant that can be used in a pt with high S. Bilirubin (value?) and S.Creatinine of
4.5mg/dL
a) Vecuronium
b) Atracurium
c) Pancuronium
d) ??
B
176) In a patient with coronary artery disease with pulse rate of 48/min and low BP. Which of
these anesthetic agent is contraindicated
a) Ketamine
b) Dexmedetomidine
c) Etomidate
d) ??
ANS=B

Dexmedetomidine [1 vial $70.00] ADMINISTRATION ROUTES:


IV
ALTERNATIVE NAMES: Precedex
ICU INDICATIONS: 1. agitation
Note: Dexmedetomidine is an expensive drug with a limited evidence base to
support its use. It should only be administered with ICU Consultant authorisation.
PRESENTATION AND ADMINISTRATION:
IV:
200mcg/2ml vial Dexmedetomidine must be diluted in 0.9% sodium chloride solution
prior to administration. Preparation of solutions is the same, whether for the loading
dose or maintenance infusion. To prepare the infusion, withdraw 2 ml of
dexmedetomidine and add to 48 ml of 0.9% sodium chloride injection to a total of 50
ml. Shake gently to mix well. Dexmedetomidine has been shown to be compatible
when administered with the following intravenous fluids and drugs: Hartmanns 5%
dextrose 0.9% sodium chloride H a n d l i n g Procedures Store at room temperature
DOSAGE: Dexmedetomidine HCl should be administered using a controlled infusion
device. Dexmedetomidine dosing should be individualized and titrated to the desired
clinical effect. It is generally initiated with:
A loading infusion of 1 mcg/kg over 10-20 minutes, (optional) followed by A
maintenance infusion of 0.2-0.7 mcg/kg/hr.
The rate of the maintenance infusion should be adjusted to achieve the desired level
of sedation. Note: doses of up to 1.4mcg/kg/hr are supported by clinical trial data.
Since dexmedetomidine clearance decreases with severity of hepatic impairment,
dose reduction should be considered in patients with impaired hepatic function
DOSAGE IN RENAL FAILURE AND RENAL REPLACEMENT THERAPY: Dose as in normal
renal function
DOSAGE IN PAEDIATRICS: There have been no clinical studies to establish the safety
and efficacy of dexmedetomidine in paediatric patients below 18 years of age.
Therefore, dexmedetomidine is not recommended for use in this population.
Dexmedetomidine
Version 30/08/2010

160Version 30/08/2010
CLINICAL PHARMACOLOGY: Dexmedetomidine is a relatively selective alpha2adrenoceptor agonist with sedative properties.
CONTRAINDICATIONS: 1. severebradycardia
WARNINGS Clinically significant episodes of bradycardia and sinus arrest have been
associated with dexmedetomidine administration
PRECAUTIONS
General
Some patients receiving dexmedetomidine have been observed to be arousable and
alert when stimulated. This alone should not be considered an evidence of lack of
efficacy in the absence of other clinical signs and symptoms. Caution should be
exercised when administering dexmedetomidine to patients with advanced heart
block and/or severe ventricular dysfunction. Because dexmedetomidine decreases
sympathetic nervous system activity, hypotension and/or bradycardia may be
expected to be more pronounced in hypovolemic patients and in those with diabetes
mellitus or chronic hypertension and in the elderly.
In situations where other vasodilators or negative chronotropic agents are
administered, co-administration of dexmedetomidine could have an additive
pharmacodynamic effect and should be administered with caution. Transient
hypertension has been observed primarily during the loading dose in association with
the initial peripheral vasoconstrictive effects of dexmedetomidine. Treatment of the
transient hypertension has generally not been necessary, although reduction of the
loading infusion rate may be desirable.
Laboratory Tests:
No tests in addition to routine ICU tests are indicated
Drug/Laboratory Test Interactions:
None known.
IMPORTANT DRUG INTERACTIONS FOR THE ICU Co-administration of dexmedetomidine
with anesthetics, sedatives, hypnotics, and opioids is likely to lead to an
enhancement of effects.
ADVERSE REACTIONS

Body as a Whole:
Fever, rigors
Cardiovascular system:
Hypotension, hypertension, bradycardia, tachycardia, atrial fibrillation
Gastrointestinal system:
Nausea,Vomiting
Respiratory system:
Hypoxia, dry mouth
Metabolic and endocrine system:
Hyperglycemia, acidosis
Haematological system:
Anaemia
http://lifeinthefastlane.com/book/critical-care-drugs/dexmedetomidine/
177) Judet view of X-ray is for
a) Pelvis
b) Calcaneum
c) Scaphoid
d) Shoulder
A
Judet's views are generally only performed as a supplementary view. In cases of acute
injury, they can be useful in demonstrating or confirming acetabular fractures. In a clinic
referral situation, Judet views may be requested to follow-up fracture healing.
Judet views are basically 45 degree obliques of the affected hip. The 45 degree angle is
best achieved by rolling the patient. Alternatively, tube angulation (with careful consideration
of grid line orientation) is a legitimate alternative. In patients with acute injuries, rolling the
patient will be very painful. Consultation directly with the referring doctor (or preferably the
orthopaedic surgeon) will ensure that your approach to performing these views is safe for
the patient.
If the Judet views are performed using a bedside technique, a non-grid technique will
ensure that the image is not marred by grid cut-off. The image quality which can be
achieved with this technique is limited due to excessive scatter radiation. Transfer of the

patient onto the X-ray table should be performed with the consent of the referring doctor
(and the patient).
http://www.wikiradiography.net/page/Judet+Views

178) Shenton line is seen in X ray of


a) Knee
b) Shoulder
c) Elbow
d) Hip
D
179) IOC for acute appendicitis in children
a) USG
b) MRI
c) CECT
d) X ray
a
180) Investigation of choice in stress fracture
a) MRI
b) CT
c) Bone scan
d) X ray
A
Nuclear Medicine and Radiologic Imaging in Sports Injuries
edited by Andor W.J.M. Glaudemans, Rudi A.J.O. Dierckx, Jan L. Gielen, Johannes Zwerve
More recently MRI has replaced scintigraphy as the investigation of choice for stress fracture

181) Investigation of choice for biliary atresia in a 2 month old is


a) Hepatic scintigraphy
b) ERCP
c) USG
d ??intra-operative cholangiography
e)CT

ANS=AD
DISCUSSION
A thorough literature search revealed less than 100 cases of EHBA associated with CDC reported till
date [2-8] of which majority were Type 1-EHBA (76%) and only 21% were Type 3-EHBA (our case)
[2]. CDC associated with Type 1-EHBA has been described only in neonates who are also found to
have distal atresia of the CBD during intra-operative cholangiogram [3]. EHBA with CDC should be
differentiated from Type 1-cystic EHBA which also has a prenatal origin but the fibro-inflammatory
cyst wall does not have biliary epithelial lining nor does it have continuity with the biliary tree [9].

As regards to the etiopathogenesis of CDC, the well accepted Babbitts hypothesis of anomalous
pancreaticobiliary union resulting in acquired CDC holds true only for children and young
adolescents and cannot explain the presentation of neonatal/ infantile CDC [10]. In 1974, Landing
hypothesized that neonatal hepatitis, biliary atresia, and choledochal cyst represented a spectrum of
manifestations of what he called infantile obstructive cholangiopathy. He proposed that the
panductular sclerosis of EHBA and the extrahepatic distal obstruction responsible for the formation
of the CDC were due to a perinatal infectious cholangitis (most likely of viral origin), which
progressed to a fibrotic scarring of the biliary tract [11]. Only Landing's hypothesis can explain the
possible etiopathogenesis of coexisting EHBA and CDC.
A noticeable finding was the early onset of jaundice in this baby (day 3 of life). This is usually seen in
Biliary Atresia Splenic Malformation (BASM) (polysplenia, situs inversus and portal vein anomalies)
syndrome which has an incidence of 10-15% [2, 5, 12]. EHBA associated with CDC has also been
shown to present at birth and even diagnosed antenatally, though the prognosis is better than that of
BASM [5].
^^^^^As we are all aware, no single preoperative investigation can diagnose EHBA with cent percent
accuracy. Surgical exploration and intra-operative cholangiography form the mainstay of
confirmatory diagnosis of EHBA. A good ultrasound by an experienced and focussed radiologist can
provide valuable information in the workup of neonatal obstructive jaundice.^^^^^^^^^ Humphrey et
al have demonstrated the following highly sensitive and specific ultrasound features for preoperative
diagnosis of EHBA - triangular cord sign (sensitivity 73% and specificity 100%), abnormal
gallbladder wall (sensitivity 91% and specificity 95%) and shape (sensitivity 70% and specificity
100%), and an absent common bile duct (sensitivity 93% and specificity 92%). In addition, they
postulated for the first time, that jaundiced infants with EHBA had a significantly larger hepatic artery
diameter when compared to those without EHBA [13]. These findings have been simulated by other
authors [14].
The most pertinent sonographic finding in a case of CDC is a well defined cyst below the
portahepatis with or without intrahepatic biliary radicle dilatation. In present case, the 4 x 4 cms
subhepatic cyst was missed on ultrasonography though the patent CBD was identified, classifying
the EHBA as Type 3. The combination of a detailed ultrasonogram and hepatic scintigraphy
should raise the preoperative suspicion of EHBA associated with CDC as the subhepatic cyst
seen on sonography will not accumulate the hepatic radioisotope on scintigraphy nor will
there be any intestinal activity seen. Moreover, an experienced and aware sonologist can
differentiate between EHBA associated with CDC, and infantile CDC, by the larger diameter of
the cysts, presence of intra-hepatic biliary dilatation and normal gall bladders in the latter
group [1].
Infantile CDC should also be distinguished from bile lakes which are usually located in the central
part of the liver and are seen in long standing cirrhohis/ cholestasis with or without EHBA. They are
pseudocysts, lacking an epithelial lining and do not communicate with the biliary tree. They can also
be seen after KPE for EHBA [15].
MRCP is an additional tool in our armamentarium for the diagnostic workup of infantile CDC. Liver
biopsy can help in differentiating EHBA from neonatal hepatitis but it is an invasive procedure and
may be complicated with bleeding. Since the patients in our setup often present late, we do not
routinely perform preoperative liver biopsy (which would take another week to get reported at our
institute). With a negative workup for medical causes of neonatal jaundice, and failure of excretion of

hepatobiliary radionucleotide into the intestine on scintigraphy even at 24 hours; our protocol is to
proceed with laparotomy and intraoperative cholangiogram which was done in this case.
The importance of per operative cholangiogram, which is still the GOLD standard for diagnosis of
EHBA, cannot be overemphasized. We were able to identify this rare coexistence during surgery
only because of the findings on cholangiography. Vice versa, absence of bile flow during surgery for
CDC should also raise the suspicion of coexisting proximal EHBA which can easily be missed on
naked eye examination [8]. Another important intra-operative finding is an abnormal gall bladder
(atretic or hypoplastic) during surgery for CDC. Often these abnormalities may not be appreciated on
ultrasonography. If the gall bladder is abnormal, intra-operative cholangiography is a must to rule out
proximal or distal EHBA [3]. Even though, in our case, the gall bladder looked normal and was 2.7
cms long.
The outcome is usually similar to that of EHBA if diagnosed and operated on time. Delay in
diagnosis increases the chances of failure. It is unfortunate that though the KPE was functioning, we
lost this child to a completely treatable etiology like cholangitis.
EHBA associated with CDC is a rare cause of neonatal obstructive jaundice. Combined
interpretation of a focussed ultrasound and hepatic scintigraphy can diagnose this condition
preoperatively. We recommend intra-operative cholangiography for all cases of either EHBA or
infantile CDC. The presence of an abnormal gall bladder and absence of bile flow during surgery for
CDC should prompt the surgeon to further explore the possibility of proximal biliary atresia.

http://www.jneonatalsurg.com//index./jns/article/view/7/43

182) Patient with h/o tachyarrhythmias is on Implantable cardioverter defibrillator. He develops


shock. Best method to know the integrity of ICD is to do
a) Contrast CT
b) MRI
c) USG
d) Plain Radiograph
d
183) Expansion of the ?space is seen in
a) Intramedullary tumor
b) Extramedullary intradural tumor
c) Intradural tumor
d) Extradural tumor
b
184) Earliest diagnosis of pregnancy can be established by
a) USG of fetal cardiac activity
b) Fetal cardiac Doppler study
c) HCG levels
d) ?

C
Early pregnancy
Dr Matt A. Morgana

and Dr

Frank Gaillarda

et al.

Early pregnancy roughly spans the first ten weeks of the first

trimester.

Radiographic features
Antenatal ultrasound
0-4.3 weeks: no ultrasound findings
4.3-5.0 weeks:
possible small gestational sac
possible double decidual sac sign (DDSS)
possible intradecidual sac sign (IDSS)
5.1-5.5 weeks:
gestational sac should be visible by this time
5.5-6.0 weeks
yolk sac should be visible by this time
gestational sac should be ~6 mm in diameter

double bleb sign


>6.0 weeks

fetal pole may be identifiable on endovaginal ultrasound (1-2 mm)


fetal heart rate (FHR) should be ~100-115 bpm
gestational sac should be ~10 mm in diameter
6.5 weeks
crown rump length (CRL) should be ~5 mm
7-8 weeks
CRL is between 11-16 mm
cephalad and caudal poles can be identified
8-9 weeks
CRL is between 17-23 mm
limb buds appear
head can be seen as separate from the body
9-10 weeks
CRL is between 23-32 mm
fetal heart rate 170-180 bpm
fetal movement can be seen
a round hypoechoic structure in the fetal brain represents a developingembryonic/fetal
nuchal translucency may begin to be seen

rhombencephalon

Transvaginal/endovaginal (TV/EV) scanning

intradecidual sac sign (IDSS): possible early sign on a TV scan


when the MSD measures 8 mm a yolk sac should be visible
when the MSD measures 16 mm, an embryo should be visible
when the MSD measures 25 mm, an embryo must be visible
when the CRL measures >7 mm, an embryo must show cardiac activity
an embryo should be seen <=14 days after a scan with a gestational sac without a yolk sac
an embryo should be seen <=11 days after a scan with a gestational sac and ayolk sac
Transabdominal (TA) scanning

when the MSD measures 20 mm a yolk sac should be visible


when the MSD measures 25 mm, an embryo must be visible
CT/MRI

Many times, early pregnancy is unintentionally imaged by CT or sometimes MRI is done for some concurrent
pathology, and its important to know the imaging findings 3.

fluid-filled cystic structure in endometrial cavity (well identified on MRI, and may be visible on CT especially on
delayed post-contrast images)
developing placenta seen as curvilinear enhancing structure
fetal pole may be seen in delayed first trimester imaging
corpus luteal cyst may be visible in one of the ovaries
unilocular <3 cm cyst with irregular crenated and enhancing walls
Differential diagnosis to be considered with a positive urinary pregnancy test includes

ectopic pregnancy
missed abortion
gestational trophoblastic disease
If urinary pregnancy test is negative similar findings may suggest submucosal
products of conception.

fibroid or retained

http://radiopaedia.org/articles/early-pregnancy
185) A smoker is worried about the side effects of smoking. But he doesnt stop smoking
thinking that he smokes less and takes a good diet. This thinking is called as
a) Self exemption
B)cognitive behaviour
c)self-protection
d) distortion
A
Pictorial warning labels (PWLs) disrupt brand symbolism by presenting alternative messages and
challenging the connotations branding creates [26]. PWLs have reduced tobacco packagings
appeal [3], increased awareness of the harms attributable to smoking, stimulated message
processing, and elicited higher levels of cessation-linked behaviours than text-only warnings
[27,28]. Because smoking causes fatal and debilitating illnesses, health warnings aim to increase
knowledge of the widespread harms directly attributable to smoking and prompt cessation.
However, while these messages resonate with older smokers, many of whom have experienced
symptoms of illness, they may have less effect on younger smokers, who reportedly see the
messages as lacking relevance and realism [29]. This lack of salience enables young adult
smokers to employ self-exempting strategies that diminish the risks shown, thus rationalising and
supporting their continued smoking [30,31]. As smoking prevalence peaks among this
demographic [32], there is an urgent need to develop warnings young adults see as salient, and
that deter initiation, reduce progression to addiction. and encourage cessation
http://www.ncbi.nlm.nih.gov/pmc/articles/PMC3694466/
http://adstandards.com.au/cases?ref=0147/14
186) A woman comes to the psychiatrist with history of spending a lot of time in washing her
hands. She is distressed about it but says that she cannot stop the practice and spends a lot of
time on it. This has started affecting her social life as well. What is the best mode of treatment in
her
a) Cognitive behavioural therapy
b) Exposure and response prevention
c) Systematic desensitization

d) ?
A
187) Which of these is not a part of catatonia
a) Akathesia
b) Ambivalence
c) Ambitendency
d) Akinesia
a
188) Key symptom for alcohol withdrawal syndrome is
a) Visual hallucinations
b) Sleep disturbance
c) Tremors
d) ?
A
Emergencies in Adult Nursing
By Philip Downing
p80

189) A person with histrionic, shy, avoidant personality comes under which cluster
a) A
b) B
c) C
d) D
ans=c
There are 10 main disorders which are classed in three clusters by DSM IV of
behaviour types, which are:
Cluster A personality disorders: these are disorders in which odd or eccentric
behaviour is considered to be central. For example in schizotypal, schizoid and
paranoid personality disorders.
Cluster B personality disorders: this group includes personality disorders in which
dramatic and erratic emotional responses are common. Borderline, antisocial,
histrionic and narcissistic personality disorders belong in this cluster.
Cluster C personality disorders: personality disorders which are classified as belonging
to cluster C are those in which anxious and fearful behaviour are central including
avoidant, dependent and obsessive compulsive personality disorders.
190) Uncertainty and excessive sense of responsibility seen in

a) Generalized anxiety disorder


b) OCD
c) Depression
d) Personality disorder
ans=b
A range of etiological theories have been proposed for OCD (e.g., psychological, biological and
neuropsychological). However, cognitive-behavioral models of OCD have generated a large body of empirical support
and have led to the development of effective treatments (see Frost & Steketee, 2002). Other etiological theories for
OCD have supported biological and structural brain abnormalities (see Pigott & Seay, 1996) and neuropsychological
factors such as memory deficits (e.g., Tallis, Pratt, & Jamani, 1999). However, inconsistencies of findings and
interference of confounding variables make interpretation difficult. For instance, while some OCD sufferers show
abnormal brain structures, many do not (Cottraux & Gerard, 1998; Riffkin et al., 2005). Further, neuropsychological
deficits (e.g., visuo-spatial memory impairment) may be a reflection of OCD symptoms rather than causal factors,
such that impaired performance on specific memory tasks may reflect uncertainty and inability to make decisions
rather than neurological deficits per-se (Otto, 1992). Moreover, such models have difficulty accounting for the
effectiveness of behavioral and cognitive treatment in OCD.
Cognitive models suggest that dysfunctional beliefs and maladaptive appraisals underlie unhelpful strategies in
the management of intrusive phenomena. Such strategies lead to extreme reactions to specific intrusive thoughts,
images, or urges resulting in obsessive and compulsive symptoms (Clark & Purdon, 1993; de Silva & Rachman,
1998; Rachman, 1998; Salkovskis, 1985). Empirical research has indicated that the vast majority of the population
experience intrusions at times, and that the difference between common intrusive thoughts and "obsessions" is in
terms of the frequency, intensity, duration, discomfort and misinterpretations elicited by the thoughts, rather than in
the content of the intrusions (Rachman & de Silva, 1978; Salkovskis & Harrison, 1984).
^^^^^Recent cognitive-behavioral research by the Obsessive Compulsive Cognitions Working Group
(OCCWG, 1997) has focused on six main belief domains that play an important role in the development of
obsessions from intrusive thoughts: inflated personal responsibility, overimportance of thought, beliefs
about the importance of controlling one's thoughts, overestimation of threat, intolerance for uncertainty and
perfectionism.^^^^^ More recently, the OCCWG (Steketee et al., 2003; Taylor, Kyrios, Thordarson, Steketee, &
Frost, 2002) reported not only a high degree of association between the identified belief domains and OC symptoms,
but also high intercorrelations between scales measuring the six domains. Further, examination of the factor structure
of a scale measuring these cognitive domains identified three larger factors (inflated sense of
responsibility/overestimation of threat, perfectionism/intolerance for uncertainty and importance/control of thoughts),
but, again, these were highly intercorrelated (OCCWG, 2005). This raises questions about possible higher order
cognitive vulnerabilities that may account for such high intercorrelations.
Further, a substantial proportion of individuals with OCD do not show high levels of these dysfunctional beliefs
(Taylor et al., 2006), suggesting that other beliefs may be important in this disorder. Finally, cognitive theories have
been criticized for not sufficiently addressing the motivational base of the disorder (O'Kearney, 1998, 2001) although
counter arguments have been raised (e.g., Salkovskis & Freeston, 2001).

http://www.psych-it.com.au/theses/article.asp?page=200

Causes of GAD
Early life experiences[edit]
Anxiety risk factors include family history (e.g. of anxiety)[44] and parenting factors including
parental rejection, lack of parental warmth, high hostility, harsh discipline, high maternal negative

affect, anxious childrearing, modelling of dysfunctional and drug-abusing behaviour, and child abuse
(emotional, physical and sexual).[45]

Biological vulnerabilities[edit]
Research upon adolescents who as infants had been highly apprehensive, vigilant, and fearful finds
that their nucleus accumbens is more sensitive than that in other people when deciding to make
an action that determined whether they received a reward.[46] This suggests a link between circuits
responsible for fear and also reward in anxious people. As researchers note, "a sense of
'responsibility', or self agency, in a context of uncertainty (probabilistic outcomes) drives the
neural system underlying appetitive motivation (i.e., nucleus accumbens) more strongly in
temperamentally inhibited than noninhibited adolescents".[46] Anxiety is also linked and
perpetuated by the person's own pessimistic outcome expectancy and how they cope with feedback
negativity.[47] Temperament and attitudes (e.g. pessimism) have been found to be risk factors for
anxiety.

https://en.wikipedia.org/wiki/Anxiety

OCD
identifying cognitive processes
The therapist would discuss with Ella the process of cognitive fusion, her overinflated
sense of responsibility and other beliefs about her intrusive thoughts or urges. She
would be helped to differentiate between thoughts and actions with intention. This
might involve behavioural experiments to test out theory B and being alone with
children. This is akin to traditional exposure, but because of the detailed discussion
and experiments on the nature of intrusive thoughts, it may be more acceptable and
less distressing than just facing your fears. The cognitive behavioural model is
presented in some detail and referred to throughout treatment the aim for the
patient is not that she stops having intrusive thoughts but that she alters her
relationship with her thoughts and develops an understanding of why some strategies
increase or decrease her symptoms. Further discussion would focus on the
assumptions involved in her appraisals of her thoughts (e.g. the therapist might
question the mechanism of how thinking can make an event happen and question the
validity of an intrusive thought and how it can reflect an actual event). This may lead
to a mental experiment of trying deliberately to induce bad actions or events (e.g.
having thoughts of causing the therapist to have a serious accident before the next
appointment or having thoughts of harming the therapist while holding a knife against
his or her neck).
Safety behaviours and compulsions
The therapist might conduct a functional analysis of the unintended consequences of

Ellas avoidance, safety behaviours and compulsions and how they prevent
disconfirmation of her worries. Patients tend to believe that their distress and worry
will continue unless they carry out their safety behaviours or compulsions. This can be
tested out in a behavioural experiment by asking the patient deliberately to perform
one of their less disturbing obsessions. For example Mark, a man with OCD who fears
being contaminated by HIV (see below), might be encouraged to perform his
compulsion (e.g. checking and reassurance-seeking) on one day and on the next to
resist the urge, on each occasion monitoring the degree of his distress or worry and
the effect on his confidence in his memory. He would then be asked to compare the
two experiences. Another experiment might involve the paradoxical effect of thought
suppression on the frequency of the intrusive thoughts. This may lead to an
experiment that involves asking the patient to record the frequency of a neutral
thought under two conditions, with and without thought suppression. This may later
be extended to their own intrusive thought. Behavioural experiments may appear to
be the same as exposure but with a rationale of testing out certain beliefs about
safety behaviours and making predictions about what would happen were they not
performed.
Distancing
An important strategy is to help the patient distance themselves from their thoughts
or urges and to cease to engage in (buy into) them. A metaphor for thoughts and
urges are cars on a road. If one engages with the cars (thoughts) then one might stand
in the road and try to divert them (and get run over) or try to get into a car and park
it. However, even when one has managed to divert or to park one car there are always
more cars to be dealt with. The key is to acknowledge the thoughts (and thank ones
mind for its contribution to ones mental health), but not to attempt to stop them or
to control them. The goal is to embrace intrusive thoughts and urges, to walk along
the side of the road, and to engage with life. This means always experiencing traffic
noise in the background intrusive thoughts never go away and reflect a persons
worries. If a patient struggles with distancing themselves from their intrusive
thoughts, this may be linked to beliefs about the consequences of not responding to
them (e.g. a person who fears being contaminated may believe that they would lose
control and go mad). In general, patients are taught to notice and experience their
thoughts and feelings without trying to evaluate them or trying to avoid or control
them.
Beliefs about contamination
Thoughtobject fusion (described above) can be used to enhance exposure to
contamination. For example, a person can put the tiniest drop of a contaminant (e.g.
urine, saliva, semen) into a large volume of water, so that the water has thoughts of
contamination. The solution can then be transferred to a hand-held spray with which
they can contaminate themselves or possessions for which they are responsible. This
may lead them to question the process of thoughtobject fusion, and the usefulness of
excessive washing or cleaning and any specific predictions that they had made

beforehand.
Beliefs about intolerability of uncertainty
The need for certainty is a common theme in OCD, especially for events in the distant
future that are impossible to disprove. For example Mark, the man with OCD
mentioned earlier, demands to know for certain whether he is HIV positive, despite
repeated reassurance from negative tests or positive explanations for his symptoms.
Such patients always have a nagging doubt the blood sample could have been
accidentally switched, there could be a new type of HIV which has not yet been
discovered, the sero-conversion has not yet occurred and so on. Mark is demanding a
100% guarantee or absolute certainty, which is of course impossible. However, while
he continues to believe that he has to be 100% certain, he will focus on the possible
doubts. Obviously the feared situations are possible, but they are highly improbable.
It is important not to get involved in a detailed analysis of probabilities but to help
the patient to focus on the process and recognise the link between the demand for
certainty and their distress and further doubt. This will help them to step back and
focus on the much higher likelihood of a poor quality of life if they continue to seek
reassurance. Patients can be helped to tackle their beliefs using humour: we can
guarantee two things in life death and taxes! A third guarantee is that while the
patient continues to demand a guarantee that a feared consequence will not occur
they will continue to disturb themselves with their symptoms.
Beliefs about terminating compulsive behaviours
As has already been noted, people with OCD tend to use problematic criteria such as
being comfortable, just right or totally sure to terminate a compulsion. Patients
can be taught that they are diverting increasing amounts of attention and trying too
hard to determine the indeterminable (e.g. whether one can be totally sure
everything has been done to make something clean). Patients who check their
memory have special difficulty. They are demanding to have a perfect or totally clear
picture of everything they have done, in the order that it happened. Socratic
questioning can be used to illustrate the impossibility of this demand and how each
check creates further ambiguous data and more scope for doubt. The criterion of
feeling comfortable is particularly impossible to achieve when confronting disturbing
fears about the harm that one may cause. In this case, patients should be helped to
focus their attention away from their subjective feelings and towards the external
world (e.g. what they can see with their eyes or feel with their hands). Alternatively,
patients may be shown that demanding to feel comfortable or confident before they
can terminate a compulsion or confront a fear is akin to putting the cart before the
horse. Patients usually need to do tasks uncomfortably and unconfidently before they
can achieve comfort and confidence in doing them.
http://apt.rcpsych.org/content/13/6/438
191) A person was diagnosed to have Schizophrenia and for that was receiving antipsychotic

medications. Following this he developed rigidity and inability to move his eyes. But he was able
to respond to Which of the following drugs should be given iv for this condition
a) Promethazine
b) Haloperidol
c) Risperidone
d) Diazepam
A
ans-diazepam
Studies suggest that for haloperidol, the dose REQUIRED for acute poisoning is
relatively high compared to therapeutic doses. Overdose does not occur with depot
injection, as this is administered by a healthcare professional who is trained to give
the injection.
The symptoms seen in haloperidol overdose are generally exaggerations of the known
drug effects and adverse reactions, which would include the following:
The most prominent feature of haloperidol overdose is severe extrapyramidal
reactions such as tremor, rigidity and an intense feeling of physical restlessness,
referred to as akathisia. These symptoms can be treated using a slow intravenous (IV)
injection of 5 mg biperiden injection, which can be repeated after a few hours if
needed. Oral or intramuscular biperiden may also need to be continued for several
days or weeks. If the extrapyramidal side effects are causing the patient distress, oral
lorazepam (0.5 mg to 1 mg) can be administered to relieve anxiety and upset as well
as the extrapyramidal side effects. This treatment can be repeated every 4 to 6
hours, if necessary.
Anticholinergic side effects include dry mouth, constipation, difficulty urinating, and
paralytic ileus, which refers to slowing of the peristaltic movement in the gut. To
correct these symptoms, slow physostigmine can be administered, although this drug
needs to be used cautiously as it can cause seizures
192) A man comes with history of abnormal excessive blinking and grunting. He says he
has no control over his symptoms which have risen in frequency of late. This has
started affecting his social life making him depressed. Which of the following
medications should be used in him
a) Carbamazepine
b) SSRIs
c) Risperidone
d) ?
Oral Medications for Treatment of Blepharospasm
and Other Cranial Dystonias #
Mark S. Ledoux, M.D., Ph.D.
University of Tennessee Health Science Center
Memphis, Tennessee
Given the marked benefit of botulinum injections, the use of oral medications for the

treatment of blepharospasm and other cranial dystonias has garnered little interest
from clinical researchers over the past 15 years. For example, at the most recent
International Congress of Parkinson's Disease and Movement Disorders Meeting in
Chicago, not one of the 5 research abstracts on blepharospasm dealt with the subject
of oral pharmacotherapy. Among 46 additional abstracts related to the treatment of
dystonia, only 2 were focused on oral medications: (1) a small study of dronabinol for
cervical dystonia did not generate optimistic results and (2) the other abstract
described positive effects of trihexyphenidyl on dystonic posturing in two girls with
Rett syndrome. The vast majority of "Dystonia: Treatment" abstracts presented
information on deep brain stimulation (DBS) and botulinum toxins. At the present
time, many intractable cases of craniocervical dystonia (typically a consequence of
immuno-resistance to botulinum toxins) are being treated with DBS without
consideration of oral pharmacotherapy.
Most pharmacological treatments for neurological and other medical disorders can be
characterized as symptomatic. However, there is growing interest in developing
medications to prevent disease onset and slow disease progression. For example, Teva
Pharmaceutical Industries has recently released data showing that the monoamine
oxidase inhibitor rasagiline slows down the rate of disease progression in Parkinson's
disease. At present, there is no data to suggest that rasagiline or any other currentlyavailable prescription medication will slow down the progression of blepharospasm.
This is an issue worthy of study since over 50% of patients who develop dystonia of the
eyelid closing musculature (i.e., blepharospasm) will exhibit spread to the lower face,
jaw musculature, or neck within 5 years of disease onset.
Most published studies of oral medications for treatment of blepharospasm and other
cranial dystonias are compromised by small sample sizes (too few patients),
heterogeneous populations (mixtures of age-of-onset, distribution and etiology) and
otherwise faulty experimental designs. By current standards, proof that a particular
drug is effective for a specific medical condition demands two independent,
randomized, multicenter, double-blind, placebo-controlled studies. Clearly, NO study
of blepharospasm or other adult-onset focal/segmental dystonia has approached these
criteria. "Modern" clinical practice as it pertains to oral medications for
blepharospasm and other cranial dystonias has been driven by treatments developed
for other movement and neurological disorders, small published clinical series,
textbook chapters written by dystonia experts, and empiricism.
Most of the oral medications used to treat blepharospasm and other cranial dystonias
are listed in Table 1. Evaluating the effects of medications in individual patients must
be tempered by consideration of drug-drug interactions, drug dosage, etiology of
blepharospasm (primary versus secondary), the occurrence of spontaneous remissions,
and duration of treatment. Moreover, it is likely that blepharospasm is
biologically/genetically heterogeneous. Consequently, what works for one patient may
not work for another! Overall, however, a significant fraction of patients with
blepharospasm and/or other forms of cranial dystonia will benefit from
trihexyphenidyl, clonazepam, or baclofen.

The drugs presented in Table 1 are divided into major classes based on mechanism of
action. In general, drugs that work on the nervous system must be started at low
dosages and incremented slowly. Similarly, none of these medications should be
discontinued abruptly. If ineffective after an adequate test period, these and most
other medications for neurological conditions should be slowly tapered.
ANS=?
http://www.blepharospasm.org/blepharospasm-oral-medications1.html
--------------------------------------------------------A man comes with history of abnormal excessive blinking and grunting. He says he has
no control over his symptoms which have risen in frequency of late. This has started
affecting his social life making him depressed. Which of the following medications he
probably used
a) Carbamazepine
b) SSRIs
c) Risperidone
d) ?
ANS=C
Movement Disorders Associated with Risperdal and Invega
Several movement disorders are linked to the use of Risperdal and Invega, but the
three most mentioned are tardive dyskinesia, tardive dystonia and parkinsonism, all
of which are mentioned in Risperdal packaging. Its possible for one person to suffer
from all of these disorders at the same time.
Researchers dont understand why or how antipsychotic medications cause these
disorders. Some theories point to the way antipsychotics alter the levels of
neurotransmitters in the brain like serotonin and dopamine.
Dopamine is responsible for regulating body movement, and it is possible that by
altering dopamine levels, Risperdal, Invega and other antipsychotics can cause
unnatural and involuntary movements. This may cause abnormal stimulation of the
parts of the brain that signal muscles to move.
Tardive Dyskinesia
Tardive dyskinesia means late-developing abnormal movements. They are called
late developing because most symptoms appear after taking a drug for a period of
time. These uncontrolled movements are usually quick, jerky and repetitive. When
patients are asleep, the symptoms may subside, but grow much worse during
moments of stress or agitation.

Symptoms of tardive dyskinesia include:


Rapid blinking of eyes Lip smacking
TongueTWITCHING and protruding out of the mouth Raising eyebrows
Foot and toe tapping Body jerking
Jerky hand, finger and arm movements Facial grimacing, tic-like movements
Clinical studies show that among Risperdal users, tardive dyskinesia most affects older
women.
Tardive Dystonia
While tardive dyskinesia is characterized by quick, jerky, repetitive movements that
are more embarrassing than painful, tardive dystonia involves slower, painful, twisting
movements of the muscles. This disorder mostly affects young men.
Some of the symptoms of tardive dystonia are:
Difficulty speaking (dystonia), running out of breath mid-sentence Grinding of teeth
Involuntary flinging out arms or legs Distress
Disability Difficulty walking and breathing
Abnormal body postures
Tardive dyskinesia and dystonia can be similar in the way they present themselves,
and their symptoms may overlap.
Parkinsonism
In clinical trials, parkinsonism was listed as one of the most common adverse
reactions to Risperdal. When caused by a medicine, it is also referred to as secondary
parkinsonism. This helps differentiate the condition from Parkinsons disease.
Like tardive dyskinesia and dystonia, parkinsonism causes involuntary body
movements. Elderly users are most susceptible.
Symptoms of secondary parkinsonism resemble those of Parkinsons disease and
include:
Decrease in facial expressions Loss or weakness of movement
Soft voice Stiffness of torso, arms or legs
Tremors Memory loss
Difficulty controlling body movements Slow, shuffling gait
Stooped posture
There are no cures for any of these movement disorders, though doctors may
prescribe other drugs or suggest certain vitamins to help control symptoms. In many
cases, patients will have irreversible damage to receptors in the brain. Neurologists
who specialize in movement disorders may also be able to help.
http://www.drugwatch.com/risperdal/movement-disorders/
193) Jersey finger is caused by rupture of
a) Flexor digitorum profundus

b) Extensor digiti minimi


c) Flexor digitorum superficialis
d) Extensor indicis
C
194) Most metabolically active part in the bone is
a) Periosteum
b) Endosteum
c) Cancellous
d) Cortical
ANS=B
ref=Osteoporosis and Bone Densitometry Measurements
edited by Giuseppe Guglielmi
The most metabolically active component of bone is the endosteal surface of cortex with trabecula bone being next
most common active area .

Mesenchymal progenitors residing close to the bone surface are functionally distinct from those in
the central bone marrow.
Siclari VA1, Zhu J, Akiyama K, Liu F, Zhang X, Chandra A, Nah HD, Shi S, Qin L.
Author information
Abstract
Long bone is an anatomically complicated tissue with trabecular-rich metaphyses at two ends and
cortical-rich diaphysis at the center. The traditional flushing method isolates only mesenchymal
progenitor cells from the central region of long bones and these cells are distant from the bone
surface. We propose that mesenchymal progenitors residing in endosteal bone marrow that is close
to the sites of bone formation, such as trabecular bone and endosteum, behave differently from
those in the central bone marrow. In this report, we separately isolated endosteal bone marrow using
a unique enzymatic digestion approach and demonstrated that it contained a much higher frequency
of mesenchymal progenitors than the central bone marrow. Endosteal mesenchymal progenitors
express common mesenchymal stem cell markers and are capable of multi-lineage differentiation.
However, we found that mesenchymal progenitors isolated from different anatomical regions of the
marrow did exhibit important functional differences. Compared with their central marrow
counterparts, endosteal mesenchymal progenitors have superior proliferative ability with
reduced expression of cell cycle inhibitors. They showed greater immunosuppressive
activity in culture and in a mouse model of inflammatory bowel disease. Aging is a major
contributing factor for trabecular bone loss. We found that old mice have a dramatically
decreased number of endosteal mesenchymal progenitors compared with young mice.
Parathyroid hormone (PTH) treatment potently stimulates bone formation. A single PTH
injection greatly increased the number of endosteal mesenchymal progenitors, particularly
those located at the metaphyseal bone, but had no effect on their central counterparts. In
summary, endosteal mesenchymal progenitors are more metabolically active and relevant to
physiological bone formation than central mesenchymal progenitors. Hence, they represent a
biologically important target for future mesenchymal stem cell studies.
Copyright 2012 Elsevier Inc. All rights reserved.

195) Removal of vertebral disc is by all these methods except

a) Laminotomy
b) Laminectomy
c) Laminoplasty
d) Hemilaminectomy
C
196) In Osteoporosis which of these is seen
a) Normal calcium, Decreased ALP
b) Decreased calcium, Increased ALP
c) Normal calcium, Normal ALP
d) Decreased calcium, Decreased ALP
C
Lab Studies:
Levels of serum calcium, phosphate, and alkaline phosphatase usually are normal in primary
osteoporosis, although alkaline phosphatase levels may be elevated for several months after a
fracture.
Checking thyroid function tests is a good idea because abnormal values may point to a cause of
secondary osteoporosis.
Markers of bone turnover (both formation and resorption) may be elevated in high bone turnover
states (eg, early Type I osteoporosis) and may be useful in some cases in monitoring early response
to therapy. However, further study to determine their clinical utility in osteoporosis management is
needed. Some of these biochemical measures include the following:
Bone-specific alkaline phosphatase (bone formation)
Osteocalcin (bone formation)
Type I procollagen peptides (bone formation)
Urinary deoxypyridinoline and cross-linked N- and C-telopeptide of type I collagen (bone resorption)

197) Traction not used in lower limb


a) Gallows
b) Bryant
c) Dunlop
d) Perkin
C
1. Dunlops Traction <ul><li>Used for supracondylar and transcondylar fractures in children
</li></ul><ul><li>Used when closed reduction difficult or traumatic </li></ul><ul><li>Forearm
skin traction with weight on upper arm </li></ul><ul><li>Elbow flexed 45 degrees </li></ul>
2. 55. Olecranon Pin Traction <ul><li>Supracondylar/distal humerus fractures
</li></ul><ul><li>Greater traction forces allowed </li></ul><ul><li>Can make angular and
rotational corrections </li></ul><ul><li>Place pin 1.25 inches distal to tip
</li></ul><ul><li>Avoid ulnar nerve </li></ul>

198) A 42 yr old man is diagnosed to have irreparable tear of the rotator cuff. Treatment of
choice will be
a) Tendon transfer
b) Total Shoul replacement
C) reverse shoulder replacement
d) acromioplasty
ANS=C
Reverse Shoulder Arthroplasty
Author: Ujash Sheth
Topic updated on 06/02/15 12:54pm
Introduction

Use of a convex glenoid (hemispheric ball) and concave humerus (articulating cup) to reconstruct the glenohumeral
joint
o center of rotation (COR) is moved inferiorly and medialized
o allows the deltoid muscle to act on a longer fulcrum and have more mechanical advantage
o therefore, deltoid must be functional

History

popularized in Europe and now increasingly used in North America

Indications

Indications
o clinical conditions

CTA
pseudoparalysis

an inability to actively elevate the arm in the presence of free passive ROM and in the absence of a
neurologic lesion

occurs secondary to irreparable rotator cuff tear in setting of glenohumeral arthritis

antero-superior escape

when all other options have been exhausted

rheumatoid arthritis

only if glenoid boneSTOCK is sufficient


patient characteristics (in clinical conditions above)

low functional demand patients


physiological age >70
sufficient glenoid boneSTOCK
working deltoid muscle

non-union or mal-union of the tuberosity following trauma or prior arthroplasty

failed arthroplasty

where GT has poor potential for healing and bone quality is poor for primary repair

rotator cuff insufficiency 'equvialent'

humeral "escape" in subcutaneous tissue with hemiarthroplasty

acute 3 or 4-part proximal humerus fractures in the elderly

incompetent coracoacromial arch

intact axillary nerve

Contraindications
o deltoid deficiency (axillary nerve palsy)

o
o
o

bony acromion deficiency


glenoid osteoporosis
active infection

Biomechanics

Biomechanics
o the advantage of a reverse shoulder arthroplasty is that the center of rotation (COR) is moved inferiorly and medialized

allows the deltoid muscle to act on a longer fulcrum and have more mechanical advantage to substitute for the
deficient rotator cuff muscles to provide shoulder abduction

allows increased (but not normal) shoulder abduction

does not significantly help shoulder internal or external rotation


reverse shoulder arthroplasty can be combined with latissimus dorsi transfer to assist with external rotation

Surgical Technique

Approaches
o superolateral

lower incidence of postoperative instability

better preservation of active external rotation

lower risk of intraoperative scapular spine and acromion fractures


deltopectoral
better orientation of glenoid component
decreased risk of glenoid loosening and scapular notching
often used for revision surgery

Technique
o humeral preparation

humeral head typically osteotomized anywhere between 0 and 30 degrees of retroversion

long head of biceps is tenotomized

labrum is excised and capsule is released circumferentially

place baseplate as inferiorly as possible with an inferior tilt

mount glenosphere onto baseplate

ream and broach humerus similar to conventional TSA


glenoid preparation
accurate central guidewire placement is dictated by availability of the best boneSTOCK
screw fixation

more retroversion is gaining popularity as it may improve post-op external rotation

shown to decrease implant loosening and scapular notching

Postoperative Care
o patient placed in sling post-op

may allow use of arm for light ADLs (brushing teeth and eating)
sling discontinued at 3 weeks if subscapularis is NOT repaired, and 6 weeks if subscapularis is repaired

Outcomes

There are no high quality long-term outcome studies present in literature


Results are dependent on indication, with cuff tear arthropathy (CTA) having the best results
Some cases series' have noted 10 year survivability is approximately 90% for implant retention
Radiographic results deteriorate after 6 years and clinical results after 8 years

Complications

for baseplate

Scapular notching
o common

o
o
o

related to impingement by the medial rim of the humeral cup during adduction
increased risk with superiorly placed glenoid component, or insufficient inferior tilt of glenoid component on the native
glenoid

Dislocation
o reported rate between 2% - 3.4%
o usually anterior instability
o increased risk with

irreparable subscapularis (strongest risk)


proximal humeral bone loss
failed prior arthroplasty
proximal humeral nonunion
fixed glenohumeral dislocation preop
NOT related to condition of rotator cuff

Glenoid Loosening
o glenoid prosthetic loosening is most common mechanism of failure
o treat using staged procedure to fill glenoid cavity with autogenous bone and await incorporation with a hemiarthroplasty
o prior to reimplantation of a new glenosphere

Deep Infection
o susceptible to infection due to large subacromial dead space created by reverse prosthesis
o most common organisms include propionibacterium acnes and staphylococci

Acromion and Scapular Spine Fractures


Neuropraxia

Sirveaux Classification of Scapular


Notching
Grade 1

limited to scapular pillar

Grade 2

in contact with inferior screw of baseplate

Grade 3

beyond the inferior screw

Grade 4

extends under baseplate approaching central peg

http://www.orthobullets.com/sports/3076/reverse-shoulder-arthroplasty
ANS=D
199)

Quartile

In statistics, a quartile, a type of quantile, is three points that divide sorted data set into four equal
groups (by count of numbers), each representing a fourth of the distributed sampled population.
There are three quartiles: the first quartile (Q1), the second quartile (Q2), and the third quartile (Q3).
The first quartile (lower quartile, QL), is equal to the 25th percentile of the data. (splits off the lowest
25% of data from the highest 75%)
The second (middle) quartile or median of a data set is equal to the 50th percentile of the data (cuts
data in half)
The third quartile, called upper quartile (QU), is equal to the 75th percentile of the data. (splits off the
lowest 75% of data from highest 25%)

How we calculating quartiles?


We sort set of data with n items (numbers) and pick n/4-th item as Q1, n/2-th item as Q2 and 3n/4-th
item as Q3 quartile. If indexes n/4, n/2 or 3n/4 aren't integers then we use interpolation between
nearest items.
For example, for n=100 items, the first quartile Q1 is 25th item of ordered data, quartile Q2 is 50th
item and quartile Q3 is 75th item. Zero quartile Q0 would be minimal item and the fourth quartile Q4
would be the maximum item of data, but these extreme quartiles are called minimum resp. maximum
of set.

For explanation of answers please visit my page on facebook


https://www.facebook.com/dragonlives4ever
http://www.rxpgonline.com/postp2783159.html#2783159

Potrebbero piacerti anche